Sei sulla pagina 1di 202

Objectives

After studying this section you will be able to:


1. Understand the fundamental geometric concepts of point, line, and plane.
2. Describe the concepts of line segment, ray, and half line.
3. Understand the concepts of plane and space.
4. Describe the relation between two lines.
5. Describe the relation between a line and a plane.
6. Describe the relation between two planes.

A. POINT, LINE, AND PLANE


1. Introduction
In this chapter we will look at the fundamental concepts we need in order to begin our study
of geometry.
Definition

geometry
The word geometry comes from two Greek words, geo and metric, which together mean to
measure the earth. Geometry is now the branch of mathematics that studies space, shape,
area, and volume.
Nature displays an infinite array of geometric shapes,
from the smallest atom to the biggest galaxy. Snowflakes,
the honeycomb of a bees nest, the spirals of seashells,
spiders webs, and the basic shapes of many flowers are
just a few of natures geometric masterpieces.
The Egyptians and Babylonians studied the area and
volume of shapes and established general formulas.
However, the first real book about geometry was written by
a Greek mathematician, Euclid. Euclids book, The
Elements, was published in about 300 BC. It defined the
most basic concepts in geometry and proved some of their
properties.
Geometry as a science has played a great role in the
development of civilization. Throughout history, geometry
has been used in many different areas such as
architecture, art, house design, and agriculture.

Geometriy 7

The three most basic concepts of geometry are point, line,


and plane. Early mathematicians tried to define these
terms. In fact, it is not really possible to define them using
any other concepts, because there are no simpler
concepts for us to build on. Therefore, we need to
understand these concepts without a precise definition.
Let us look instead at their general meaning.

2. Point
When you look at the night sky, you see billions of stars, each represented as a small dot of
light in the sky. Each dot of light suggests a point, which is the basic unit of geometry.
Concept

point
A point is a position. It has no size, length, width, or thickness, and it is infinitely small.
We use a dot to represent a point. We name a point with a capital letter such as A, B, C, etc.

Natures Great Book is


written in mathematical symbols.
Galileo Galilei

All geometric figures consist of collections of points, and many terms in geometry are defined
using points.

3. Line
Concept

line
A line is a straight arrangement of points. It is the second fundamental concept of geometry.
There are infinitely many points in a line. A line has no width or thickness, and extends
without end in both directions.

Geometric Concepts

A line is usually named by any two of its points, or by a lower-case letter.


Look at the diagram. The line that passes through points A and B is written AB. We say it is
line AB. The line on the right is simply called line .

Let none unversed in


geometry enter here.
Plato

The arrows at each end of a line show that the line extends to infinity in both directions.
If any point C is on a line AB or a line d, we write C AB, or C d.

C
C AB

A, B, C d

Property

There exists exactly one line passing through any two distinct points.
By this property, a line is determined by two distinct points. However, remember that a line
consists of more than just two points. There are infinitely many points on a line.

4. Plane
Concept

plane
A plane is the third fundamental concept of geometry. A plane has length and width but no
thickness. It is a flat surface that extends without end in all directions.
A plane is suggested by a flat surface such as a table top, a wall, a floor, or the
surface of a lake. We represent a plane with a four-sided figure, like a piece of
paper drawn in perspective. Of course, all of these things are only parts of
planes, since a plane extends forever in length and in width.
We use a capital letter (A, B, C, ...) to name a plane. We write plane P, or (P),
to refer to a plane with name P.

10

Geometriy 7

We can now understand the meaning of the terms point, line, and plane without a formal
definition. We can use these undefined terms to define many new geometric figures and
terms.

5. Collinear Points
collinear points

Definition

Points that lie on the same line are called collinear points.
A

For example, in the diagram above, points A, B, and C lie on the same line d. Therefore A, B,
and C are collinear points. However, point P is not on line so M, P, and N are not collinear
points. We say that, M, P, and N are noncollinear points.

EXAMPLE

Look at the given figure.


a. Name the lines.

b. Write all the collinear points.


c. Give two examples of noncollinear points.
Solution

a. There are three lines, AC, CN, and SR.

c. The points A, N, C and the points M, T, N


are not on the same line. They are examples of noncollinear points.

Geometric Concepts

N
R

b. The points A, B, C, the points S, T, R, and


the points C, M, N are on the same line, so
they are collinear.

Now consider the three noncollinear points in


the figure on the right. Since we know that two
distinct points determine a straight line, we
can draw the lines AB, AC and BC passing
through A, B, and C. Therefore, there are three
lines that pass through three noncollinear points.

d
B
m
C
l

11

triwise points

Definition

If three points are noncollinear then they are also called triwise points.
When we say, n triwise noncollinear points,
we mean that any three of n points are
noncollinear.

A
B
E

For example, the diagram opposite shows five


triwise noncollinear points. Any set of three
points in the diagram is noncollinear.

Theorem

n ( n 1)
different lines pass through n triwise points.
2

EXAMPLE

Solution

How many different lines pass through each number of triwise noncollinear points?
a. 4

b. 5

c. 9

d. 22

a.

4 (4 1) 4 3
=
= 6 lines
2
2

b.

5 (5 1) 5 4
=
= 10 lines
2
2

c.

9 (9 1) 9 8
=
= 36 lines
2
2

d.

22 (22 1) 22 21
=
= 231
2
2

Check Yourself 1
1. Describe the three undefined terms in geometry.
A

2. Name the collinear points in the figure.

B
C

3. Look at the figure.


a. Name the lines.
b. Name all the collinear points.
c. Give two examples of noncollinear points.
12

A
F
D

C
E
B

Geometriy 7

4. How many different lines can pass through each number of triwise noncollinear points?
a. 8

b. 14

c. 64

d. 120

Answers
1. Point: A point is a position. It has no size, length, width, or thickness, and it is infinitely
small. Line: A line a straight arrangement of points. There are infinitely many points in a
line. A line has no width or thickness, and extends without end in both directions. Plane:
A plane has length and width but no
surface that extends without end in all directions.

thickness.

It

is

is

flat

2. The points A, B, C and the points D, B, E are collinear points.


3. a. The lines: AC, AB, DG b. The points A, E, B, the points D, B, G are collinear points c.
The points A, F, G, and the points D, B, C are non collinear points.
4. a. 28 b. 91 c. 2016 d. 7140

B. LINE SEGMENT, RAY, AND HALF LINE


1. Line Segment
Definition

line segment
The line segment AB is the set of points
consisting of point A, point B, and all the
points between A and B. A and B are called the
endpoints of the segment. We write [AB] to
refer to the line segment AB.

B
line segment
AB or [AB]

This definition describes one type of line segment: a closed line segment. There
are three types of line segment.

a. Closed Line Segment


A line segment whose endpoints are included in the
segment is called a closed line segment.

[AB] in the diagram is a closed line segment.


Physical model of a line
segment: a piece of string.

b. Open Line Segment


A line segment whose endpoints are excluded from the
segment is called an open line segment.

The line segment AB in the diagram is an open line segment and denoted by ]AB[.
We use an empty dot ( ) to show that a point is not included in a line segment.
Geometric Concepts

13

c. Half-Open Line Segment


open line segment.
A line segment that includes only one of its endpoints is called a half-o
A

half-open line segment AB


[AB[

EXAMPLE

half-open line segment AB


]AB]

Name the closed, open and half-open line segments in the figure on the right.

Solution Closed line segment: [AB]


Half-open line segments: [AC[, [BC[, and [BD[
Open line segment: ]CD[
Property

If C is a point between A and B, then

[AC]

[CB]

[AC] + [CB] = [AB].


A

Using this property, we can conclude that if


three points are collinear, then one of them is
between the other points.
A

[AB]

Point B is between the points A and C.

2. Ray
Definition

ray
The ray AB is the part of the line AB that contains point A and all the points on the line
segment that stretches from point A through point B to infinity. The ray AB is denoted by [AB.
In the diagrams, each ray begins at a point and extends to infinity in one direction. A is the
endpoint of [AB, and C is the endpoint of [CD.
A

B
ray AB, or [AB

14

C
[CD

Geometriy 7

B
half line AB
]AB

A half line extends to infinity in one direction. A half line is like a ray,
but it begins at an open endpoint.

C. PLANE AND SPACE


1. Plane
We can think of the floor and ceiling of a room as parts of horizontal planes. The walls of a
room are parts of vertical planes.

vertical planes

A point can be an element of a plane.

D
B

A
C

horizontal planes

In the diagram, point A is an element of plane P. We can write


A (P). Similarly, B (P), C (P), D (P), and E (P).

plane P: (P)

a. Coplanar Points
Definition

coplanar points
Points that are in the same plane are called coplanar points.
In the figure, points A, B, and C are all in the plane P. They
are coplanar points. Points K, L, and M are also coplanar
points. A, K, and M are not coplanar points, because they
do not lie in the same plane.

Geometric Concepts

A
C

P
K

15

b. Coplanar Lines
Definition

coplanar lines
Lines that are in the same plane are called coplanar lines.

For example, in the figure, the lines m and n are both in


the plane P. They are coplanar lines.

Theorem

For any three points, there is at least one plane that contains them. For any three noncollinear points, there is exactly one plane that contains them.
In the figure, the plane P is determined by the
noncollinear points A, B, and C.

A
B

2. Space
Definition

space
Space is the set of all points.
We have seen that lines and planes are defined by sets of points.
According to the definition of space, all lines and planes can be considered as subsets of space.

D. RELATION BETWEEN TWO LINES


1. Intersecting Lines
d

Two lines that intersect each other in a plane are called


intersecting lines, or concurrent lines.
In the figure on the left, line d and line l intersect each
other at point A. They are intersecting lines.
16

l
A

Geometriy 7

2. Parallel Lines

Two lines are parallel if they are in the same plane and
do not have a common point.

d l and d l =

In the figure on the left, line d and line l are parallel


lines. We write d l to show that lines d and l are parallel.

3. Coincident Lines
Two lines are coincident if each one contains all the
points of the other.

In the figure on the left, line d and line l are coincident


lines. We write d = l to show that lines d and l are
coincident.

d=l

4. Skew Lines
l

Two lines are skew if they are non-coplanar and they do


not intersect.
In the figure on the left, E and F are two non-parallel
planes. Hence, lines d and l are in different planes, and
since they do not intersect, they are skew lines.

EXAMPLE

d
F

In the figure there are three intersecting lines. Decide whether each statement is true or
false.
m C

a. point A is the intersection of l and d

b. point C is the intersection of d and l


c. point B is the intersection of l and m
Solution

a. True, since point A is the common point of l and d.


b. False, since point C is not a common point of d and l.
c. True, since point B is the common point of l and m.

Geometric Concepts

17

E. RELATION BETWEEN A LINE AND A PLANE


We have seen the different possibilities for the relation between two lines. Let us look at the
possible relations between a line and a plane.

1. The Intersection of a Line and a Plane


A line can intersect a plane at one point.

In the diagram on the left, the line d intersects the plane


E at point A.

d (E) = {A}

2. Parallelism of a Line and a Plane


A line can be parallel to a plane.
In the diagram on the left, there is no common point
between line d and plane E. They are parallel.

E
d (E) =

3. A Line Lies in a Plane


If at least two points of a line lie in a plane, then the line
lies in the plane. We write d (E) to show that line d lies
in plane E.
In the diagram, points A and B are in plane E, so the line
AB lies in the plane E.

d
B
A

A, B d
A, B (E)

d (E)

F. RELATION BETWEEN TWO PLANES


1. Parallel Planes
If two planes have no common point, they are called
parallel planes. We write (A) (B) to show that two
planes are parallel. The opposite walls of a room are an
example of parallel planes.

P Q
(P) (Q)

18

E
F
(E) (F)

Geometriy 7

2. Intersecting Planes
If two planes have only one common line, they are called
intersecting planes.

F
(E) (F) = d

3. Coincident Planes
If two planes have three noncollinear points in common,
they are called coincident planes. (P) and (Q) in the
figure are coincident planes. We write (P) = (Q) to show
that planes P and Q are coincident.

A
Q
P
A, B, C (P)
A, B, C (Q)

(P) = (Q)

4. Half Planes
A line in a plane separates the plane into two disjoint
regions that are called half planes. (E1) and (E2) in the
figure are half planes of (E).

half plane
(E1)

half plane
(E2)

E
boundary of two half planes
(E1) (E2) =
(E1) (E2) l = (E)

Geometric Concepts

19

EXERCISES

1 .1

1. Explain why the concepts of point, line, and plane

10. Name all the lines, rays, line segments, and half
lines in the given figure.

cannot be defined in geometry.

2. Draw five points on a piece of paper, and make

sure that no three are of them collinear. Draw all


the lines passing through these points. How many
lines can you draw?

4. At least how many points determine a line?


5. At least how many noncollinear points determine
a plane? Why?

H
L
E

3. Explain the difference between a ray and a half


line.

11. Write the meaning of the following.


a. [CD] b. [PQ[

c. ]AB[

d. [KL

e. ]MN

f. EF

12. Describe the intersection of the line and the

6. Give examples from daily life to illustrate the


concepts of point, line, and plane.

plane in each figure.


a.
b. m

7. Write words to complete the sentences.


a. A point has no __________ and no __________.

A
E

c.
C

b. Two points determine a ___________ .


c. Three noncollinear points determine a _____ .
d. Two lines that lie in different planes and do
not intersect are called ___________ lines.

8.

13. Write the coplanar points


in the given figure.

D
A

L
K

14. Draw a diagram to show that the intersection of


two planes can be a line.

Determine whether the statements are true or


false for the given figure.

15. Draw a diagram to show that the intersection of


three planes can be a point.

a. A, B, and C are collinear points


b. points D and E are not in the line l
c. B l

16. Look at the figure.

a. How many planes are


there?

d. E l
e. C, D, and E are noncollinear

S
DR

number of triwise noncollinear points?

b. Write the intersection


of the planes.

a. 5

c. How many lines pass through each point?

9. How many different lines can pass through each

20

b. 7

c. 21

d. 101

P
A

Q
B

Geometriy 7

1A

CHAPTER REVIEW TEST

1. Which concept is precisely defined in geometry?


A) point

B) line
D) space

C) plane

E) ____

5. According to the figure,

which statement is
false?

B
A

d
m

A) l d = {C}

B) l m = {A}

C) l d m = {A, B, C}

D) m d = {B}

E) ____

2. A plane has no
A) thickness.

B) length.

D) surface.

C) width.

E) ____

6. ABCD is a rectangle in a plane P. E is a point such


that E (P). How many planes are there that
include point E, with one or more of points A, B,
C, and D?
A) 7

3. A ray with an open endpoint is called


A) a line.

B) a half line.

C) a line segment.

D) an open line segment.


E) ____

B) 8

C) 9

D) 10

E) ?

7. How many lines do five points determine if no


three of the points are collinear?
A) 15

4. According to the figure,


which statement is true?

D
B E

B) 12

C) 10

E) ?

d
C

8. Space is

A) A, B and E are collinear points

A) the intersection of two planes.

B) l d = {B}

B) the set of all points.

C) C l

C) a subset of a plane.

D) D, B, and E are noncollinear points

D) a very large plane.

E) _

E) ?

Chapter Review Test 1A

D) 9

21

9. According to the figure,


which statement is
false?
A) C (E)

12. Let [AB] + [BC] = [AC], and [MN] + [NK] = [MK].

Which points are between two other points?


d

C
A

B) l (E) = {A}

A) A and M

B) B and M

C) C and K

D) B and N
E) ____

C) l d = {B}
D) l and d are skew lines
E) ?

10. Which figure shows ]AB?


A)
C)

B)
D)

E) ____

11. According to the figure,

which statement is false?


l

A) (P) l = l
B) (P) m = m

C) (P) n = n
D) l m n = {A}
E) ?
22

Geometriy 7

Objectives

After studying this section you will be able to:


1. Define the concept of angle and the regions an angle forms.
2. Measure angles.
3. Classify angles with respect to their measures.
4. Classify angles with respect to their positions.
5. Classify angles with respect to the sum of their measures.

A. REGIONS OF AN ANGLE
1. Angle
One of the basic figures in geometry is the angle.
A television antenna is a physical model of an angle.
Changing the length of the antenna does not change the
angle. However, moving the two antennae closer
together or further apart changes the angle.

Definition

angle
An angle is the union of two rays that have a common endpoint. The rays are called the sides
of the angle. The common endpoint is called the vertex of the angle.
Look at the diagram. [BA and [BC are the sides of the
angle. The vertex is the common endpoint B.
e

vertex

sid

The symbol for an angle is . We name the angle in the


diagram ABC, or CBA, and say angle ABC, or angle
CBA. We can also name angles with numbers or
lower-case letters, or just by their vertex.

side
B

Note
In three-letter angle names the letter in the middle must always be the vertex.
24

Geometriy 7

EXAMPLE

Solution

Name the angles in the diagrams.

a.

b.

c.

d.

a. AOB or BOA
c. 1

b. A

d. a

interior and exterior region of an angle

Definition

The region that lies between the sides (arms) of an angle is called the interior region of the angle.
The region that lies outside an angle is called the exterior region of the angle.

EXAMPLE

Answer the questions for the angle ABC on the right.


A

a. Which points are in the interior region of the angle?


b. Which points lie on the angle?

c. Which points are in the exterior region of the angle?


Solution

D
H

B
F

a. The points D and E are in the interior region of the angle.


b. The points A, B, C, and H lie on the angle.
c. The points G and F are in the exterior region of the angle.

B. MEASURING ANGLES
Angles are measured by an amount of
rotation. We measure this rotation in units
called degrees. One full circle of rotation is
360 degrees. We write it as 360.

Babylonian astronomers chose


the number 360 to represent one
full rotation of a ray back on to
itself.

B
360

We can show the size of an angle on a diagram


using a curved line between the two rays at the
vertex, with a number. When we write the size
of an angle, we write a lowercase m in front of
the angle symbol.

Why this number was chosen?


It is because 360 is close to the
number of days in a year and it is
divisible by 2, 3, 4, 5, 6, 8, 9, 10,
12, and many other numbers.

45

For example, mAOB = 45 means that angle AOB measures 45 degrees.


Look at some more examples of angle measures in the diagrams.

10

mA = 10

Angles

30

mB = 30

90

mC = 90

150

D
mD = 150

E
360

mE = 360

25

60

70

80

90 100

110

50

Notice that the symbol for a 90 angle is a small square at the vertex. A 90 angle
is also called a right angle in geometry.

120

130
90 80 70
110100
140
120
60
50
130
30
150
40
140
30
150
20
160
160
20
10
170
170
10

It is important to read angles carefully in geometry problems. For example, an


angle in a problem might look like a right angle (90). However, if it is not labelled
as a right angle, it may be a different size. We can only use the given information
in a problem. We calculate other information using the theorems in geometry.

40

180

180

10

protractor

Definition

The geometric tool we use to measure angles on paper is called a protractor.


A protractor has a semi-circular shape and a scale with units from 0 to 180.
To measure angles with a protractor, follow the three steps below.
60

70

80

90 100

50

110

60

120

120

40

180

110

130
90 80 70
110100
140
120
60
50
130
30
150
40
140
30
150
20
160
160
20
10
170
170
10

90 80 70
110100
140
120
60
50
130
30
150
40
140
30
150
20
160
160
20
10
170
170
10
180

90 100

50

130

40

80

70

10

180

180

10

1. Place the central hole (dot) of the protractor on the vertex of the angle.
2. Place the zero measure on the protractor along one side of the angle.
3. Read the measure of the angle where the other side of the angle crosses the protractors scale.
Notice that there are two semicircular scales of numbers on the protractor. If the angle
measure is smaller than 90 then we read the angle using the scale with the smaller number.
If the angle measure is greater than 90 then we use the scale with the larger number.

EXAMPLE

Read the protractor to find the measure of


each angle.
a. mAOB

b. mAOC

d. mAOE

e. mAOF

g. mCOF

h. mDOE

D
E

c. mAOD

80

90 100

50

110

120

130
90 80 70
110100
40
140
120
60
50
130
30
150
40
140
30
150
20
160
160
20
10
170
170
10

f. mBOC
F

180

26

60

70

O
3

B
A

180

10

Geometriy 7

Solution

a. mAOB = 22

b. mAOC = 68

c. mAOD = 90

d. mAOE = 140

e. mAOF = 175

f. mBOC = mAOC mAOB = 68 22 = 46

g. mCOF = mAOF mAOC = 175 68 = 107


h. mDOE = mAOE mAOD = 140 90 = 50
A

For example, let us use a protractor to draw an angle of 56.

1. Draw a ray.

C
A

2. Place the centre point of the protractor on the endpoint (A) of the ray.
Align the ray with the base line of the protractor.

C
A

56

3. Locate 56 on the protractor scale. Make a dot at that point and label it as C.
mBAC = 56

4. Remove the protractor and draw [AC.

After learning to how use a protractor we can easily draw and measure angles.

Check Yourself 1
1. Name the following angles.
a.

b.

c.

d.
3

2. Find the following sets of points in the figure.


a. L {X}
b. int L {X}
c. ext L L

X
T

M S

K
L
Y

d. int L ext L
e. int L {S}
f. L {T, S, }
g. int L {Z, Y}
h. ext L {Z, Y}
Angles

27

3. Measure each angle using a protractor.


a.

b.

c.
B

d.
C

4. Draw the angles.


a. 45

b. 83

c. 174

d. 180

e. 225

Answers
1. a. AOB b. A c. 3 d. b
2. a. b. {x} c. d. e. f. {S, K} g. h. {Z, Y}

C. TYPES OF ANGLE WITH RESPECT TO THEIR MEASURES


We can classify angles according to their measures.

1. Acute Angle
An angle that measures less than 90 is called an acute
angle.
The angles on the left are all examples of acute angles
because they measure less than 90.

25

45

80

2. Right Angle
An angle that measures exactly 90 is called a right angle.
The angles on the left are all examples of right angles
because they measure exactly 90. We use a special
square symbol at the vertex to show a right angle.

3. Obtuse Angle
An angle that measures between 90 and 180 is called
an obtuse angle.
The angles on the left are all obtuse angles.

Y
91

120

165
O

4. Straight Angle
An angle that measures exactly 180 is called a straight
angle. In the diagram, A is a straight angle.
28

180
A

Geometriy 7

5. Complete Angle
An angle that measures exactly 360 is called a complete
angle. In the diagram, E is a complete angle.

EXAMPLE

Solution

Classify the angles according to their a.


measure.

b.

180

a. 180 is a straight angle.

360

c.

360

125

d.

b. 360 is a complete angle.

e.
35

c. 125 is between 90 and 180, so


it is an obtuse angle.
d. 90 is a right angle.
e. 35 is less than 90, so it is an acute angle.

D. TYPES OF ANGLE WITH RESPECT TO THEIR POSITION


1. Adjacent Angles
adjacent angles

Definition

Adjacent angles are two angles in the same plane that have a common vertex and a common
side, but do not have any interior points in common.
In the diagram, the angles AOC and BOC have a
common vertex and a common side ([OC) with
non-intersecting interior regions.
Therefore, AOC and BOC are adjacent angles.

B
C

Theorem

If two angles are vertical then they are also congruent, i.e. they have equal measures.
EXAMPLE

Determine whether the pairs of angles are vertical or not,


using the figure.
a. a, b

b. a, c

c. d, a

d. b, d

l
c

Solution The lines l and k intersect at one point. Therefore,


a.
b.
c.
d.
Angles

b
d

a
k

a and b are not vertical angles,


a and c are vertical angles, because they are in opposite directions,
d and a are not vertical angles, and
b and d are vertical angles.
29

E. TYPES OF ANGLE WITH RESPECT TO THE SUM OF


THEIR MEASURES
1. Complementary Angles
complementary angles

Definition

If the sum of the measures of two angles is 90, then the


angles are called complementary angles.

Each angle is called the complement of the other angle.


For example, in the diagram opposite, ANB and CMD
are complementary angles, because the sum of their
measures is 90:

30
60
N

mANB + mCMD = 30 + 60 = 90.

2. Supplementary Angles
supplementary angles

Definition

If the sum of the measures of two angles is 180, then the angles are called supplementary
angles. Each angle is called the supplement of the other angle.
In the diagram, XYZ and MNO are supplementary
angles because the sum of their measures is 180:

40

mXYZ + mMNO = 40 + 140 = 180.

EXAMPLE

Solution

Find x if the given angles are


complementary.
a. If x and 2x are
complementary, then

M
X
O

Y N

a.

140

b.
3x+30
2x

2x+10

x + 2x = 90.
Therefore, x = 30.
b. 2x + 10 + 3x + 30 = 90
2x + 3x + 10 + 30 = 90
5x = 50
x = 10
30

Geometriy 7

EXAMPLE

Solution

Find x if the given angles are


supplementary.

a.

b.

a. If 2x and 4x are supplementary, then

4x

2x

2x+60

3x+50

2x + 4x = 180.
Therefore, x = 30.
b 2x + 60 + 3x + 50 = 180
2x + 3x + 60 + 50 = 180
5x + 110 = 180
5x = 70
x = 14

Check Yourself 2
b.

c.

15
3x+

4x

2x+

30

a.

1. Find x if the given angles are complementary.

2x+20
x 20

2. Find x if the given angles are supplementary.


a.

b.
6x

3x

c.
2x 10

4x+40

5x 12
2x
18

Answers
1. a. 18 b. 30 c. 9
2. a. 20 b. 25 c. 30

Angles

31

Objectives

After studying this section you will be able to:


1. Identify corresponding angles, alternate interior angles, and alternate exterior angles.
2. Identify interior angles on the same side of a transversal.
3. Describe the properties of angles with parallel sides.
4. Define an angle bisector.

A. CORRESPONDING ANGLES AND ALTERNATE ANGLES


Definition

supplementary angles
Let m and n be two lines in a plane. A third line l that intersects each of m and n at different
points is called a transversal of m and n.
In the diagram, line AB is a transversal of m and n.
Let us look at the types of angle formed in the figure of two parallel lines with a transversal.
Remember the notation for parallel lines: m n means that m is parallel to n.

1. Corresponding Angles
Definition

corresponding angles
In a figure of two parallel lines with a transversal, the
angles in the same position at each intersection are
called corresponding angles.

m//n
2
3

In the diagram, 1 and 5 are corresponding angles.


Also, the angle pairs 2 and 6, 3 and 7, and 4 and
8 are corresponding angles.

6
7

Property

Corresponding angles are congruent.


Therefore, in the diagram, m1
m2
m3
m4

=
=
=
=

m5,
m6,
m7, and
m8.

m//n
3

6
7

32

Geometriy 7

2. Alternate Interior Angles


Definition

alternate interior angles


In a figure of two parallel lines with a transversal, the interior angles between the parallel
lines on opposite sides of the transversal are called alternate interior angles.
m//n
In the diagram, the angles 4 and 6 are alternate
interior angles. Also, 3 and 5 are alternate interior
angles.

Property

Alternate interior angles are congruent.

m//n

Therefore, in the diagram,

m4 = m6, and m3 = m5.


6

3. Alternate Exterior Angles


Definition

alternate exterior angles


In a figure of two parallel lines with a transversal, the angles outside the parallel lines on
opposite sides of the transversal called alternate exterior angles.
l

m//n
In the diagram, the angles 1 and 7 are alternate
exterior angles. Also, 2 and 8 are alternate exterior
angles.

3
6
7

4
5

Property

Alternate exterior angles are congruent.

m//n
2

Therefore, in the diagram,


m1 = m7, and m2 = m8.
7

Angles

m
n

33

4. Interior Angles on the Same Side of a Transversal


alternate interior angles

Definition

In a figure of two parallel lines intersected by a


transversal, interior angles on the same side of the transversal are supplementary.

m//n

m
x

Therefore, in the diagram, mx + my = 180.


y

EXAMPLE

In the digaram, [BA DE.

100

Find mBCD.
30
D

Solution

B
100

80

30

B
100

30

30

If [BA [CF then ABC


and x are supplementary.
mABC + mx = 180
100 + mx = 180
mx = 80

EDC and y are


alternate interior
angles.
mEDC = my
my = 30

mBCD = mx + my
= 80 + 30
= 110

5. Angles with Parallel Sides


Theorem

The measures of two angles with parallel sides in the same direction are equal.
Proof

Consider the diagram on the right.

[OA [LK

1. AOB and LTB are corresponding angles.

[OB [LM
A

AOB LTB

mAOB = mKLM
34

2. LTB KLM (corresponding angles)


AOB KLM

M
O

Geometriy 7

EXAMPLE

Solution

In the figure, [AC [DF, [AB [DE, mCAB = 2x + 40,


and mFDE = 6x 20. Find mCAB.

C
D

mCAB = mFDE
2x + 40 =
40 + 20 =
60 =
15 =

F
6x 20
E

2x + 40

6x 20
6x 2x
4x
x

So mCAB = 70.

Theorem

The measures of two angles with parallel sides in opposite directions are equal.
Proof

1. KLM BPL (corresponding angles)


2. AOB BPL AOB KLM

[OA // [LM
[OB // [LK

Property

In the figure, if d k and B is the intersection of [BA and


[BC, then

mb = ma + mc.

a
b

EXAMPLE

10 In the figure, [AE [BF, mA = 40, and mB = 30.

Find mAOB.
Solution

mAOB = mOAE + mOBF


mAOB = 40 + 30
= 70

Angles

E
40
?

30
F

35

Property

In the figure, if d k and B is the intersection of [BA and


[BC, then

EXAMPLE

11 In the figure, AB CD, mAEF = 5x, mEFH = 3x, and

3x

mFHC + mFHD = 180 (supplementary angles)


mFHC = 180 2x
mAEF + mEFH + mFHC =
5x + 3x + 180 2x =
6x + 180 =
6x =
x=

E
5x

mFHD = 2x. Find x.


Solution

ma + mb + mc = 360.

2x
H

360
360
360
180
30

Property

In a figure such as the figure opposite, the sum of the


measures of the angles in one direction is equal to the
sum of the measures of the angles in the other direction.

A E

mx + my + mz = ma + mb + mc + md

AB CD

EXAMPLE

12 In the figure, [AE [DF, mEAB = 35, mBCD = 25,


and mABC = 4 mCDF. Find mABC.

Solution

35 + 25 = 4x + x
60 = 5x
x = 12

35

4x
B
25
x

Therefore, mABC = 48.


36

Geometriy 7

Two lines are called perpendicular lines if they intersect


at right angles. We write AB CD to show that two lines
AB and CD are perpendicular.

A
C

D
O
B

Property

In the diagram, if [OA [LK and [OB [LB then


mAOB + mNLB = 180.

K A
N
L

EXAMPLE

13 In the figure, [BA [FD, [BC [FE, [BA [FG,

mABC = 60, and mGFE = x. Find x.

Solution

mABC + mDFE = 180


60 + mDFE = 180
mDFE = 120

mGFD
mGFD + mGFE + mDFE
90 + mGFE + 120
90 + x + 120
x + 210
x

EXAMPLE

60

=
=
=
=
=
=

90
360
360
360
360
150

14 In the figure, [AC [ED, mABE = 23,


Let us draw a line [BF parallel to [ED.
mDEB + mEBF = 180 (interior angles on the same
118 + mEBF = 180 side of a transversal)
mEBF = 62
mBAC + mABF = 180
mBAC + 85 = 180
mBAC = 95

E
23

and mBED = 118, Find mBAC.

Solution

118

B
C

A
E
23
62

118

D
F

Angles

37

EXAMPLE

15 In the figure, [AD [CE, mDAB = 112,

and mBCE = 120.

Find mABC.

120

A
112

?
B

Solution

Let us draw a line [BK parallel to [AD.


mDAB + mABK = 180
112 + mABK = 180
mABK = 68

C
D

112
68

mBCE = mABC + mABK


120 = mABC + 68 mABC = 52

120

A
?
B

6. Bisector of an Angle
angle bisector

Definition

A ray that divides an angle into two congruent angles is called the bisector of the angle.
In the figure, [OB is the bisector of AOC:

mAOB = mBOC
=

EXAMPLE

1
AOC.
2

16 In the figure, [BE and [BF are the bisectors of ABD and
CBD respectively. Find mEBF.

Solution

a
a

D
F

mABD + mCBD = 180


mEBF =

mABD m CBD
+
= 90
2
2

Definition

The bisectors of two adjacent supplementary angles are perpendicular to each other.
38

Geometriy 7

EXAMPLE

17 In the figure, [OE is the bisector of FOD.

mBAD = a, mEBC = b, and mFCG = c.


Show that a + c = 2 b.

Angles

C
c
O

Let [OH AG.


mHOC = mGCF = c
mHOB = mCBE = b
mHOD = mBAD = a
Let mCOB = mBOA = x.
c+x=b x=bc b+x=a
b+bc=a
Therefore, a + c = 2 b.

Solution

F
E

a
A

39

EXERCISES

1. Using the given figure, find each set of


points.

P
M

K
N

a. O {P}
b. O {N}

c. O {K, O, M}

d. int O {P}

e. int O {N}

f. int O {K, O, M} g. ext O {N}


h. ext O {P}

i. O int O

j. O ext O

k. int O ext O O

4. Draw the angles.


a. 20

b. 35

c. 75

d. 120

e. 175

f. 210

g. 240

h. 330

5. Classify the types of angle.


a.

b.

c.
135

45

d.

e.
360

2. Find each set of


points for the given
figure.

B
A
H

a. ABC ACD

D F
G

b.

complementary.
a.

4x+30

15
x+

2x+25

c. ABC int CAD

c.

35
3x

b. int ABC CAD

6. Find x in each figure if the angles are

3x5

d. ext ABC CAD


e. ABC ext CAD

7. Find x in each figure if the angles are

3. Measure the angles using a protractor.


a.

b.

c.

d.

e.

f.

supplementary.
a.
3x

b.

2x+20

3x+15

c.
4x+25

125 + 2x

45 x

8. In the figure, m n,

g.

h.

i.

l is a transversal and
m7 = 115.
Find the measures.
a. m1

b. m2

c. m3

d. m4

e. m5

f. m6

3
4

7
8

g. m8
40

Geometriy 7

9. Given [BA [DE, find

mx.

10. In the figure,

x
E

[BA [ED,
[BC [EF,
mABC = 3x 30, and
mDEF = 4x 70.
Find x.

D
4x70

25

3x
40

Find x.

x
2x

50
x

l
C

18. In the figure,

d l.
Find mx.

105

17. In the figure, d l.

115

12. In the figure,

30

3x30

d l.
Find mx.

100

[BA [EF,
mBCD = 100,
mCDE = 25, and
mFED = 105.
Find mABC.

80

11. In the figure,

16. In the figure,

B
30

100

[BA [DE,
[BC [DF and
[BC [BD], and
mGDE = 40.
Find mABC.

A B

E
G

19. In the figure,


13. In the figure,

A
x

[BA [DE,
mBCD = 40, and
mCDE = 120.
Find mABC.

40

Find mx.

15. In the figure,


[BA [ED] and
[CD] [EF.
Find the relation
between x, y, and z.
Angles

120

14. In the figure,


[BA [FG,
mEFG = 120, and
mABC = 130.

[BC [DF,
[BA [DG, and
[ED] is the angle
bisector of mGDF.
Find mABC.

G
B

20 . In the figure,
x

F
B
130

2x
E

2x
4x
C

60

[AB [CD.
Find mAEC.

D
F

120

140
E

E
G

40

21 . In the figure,
AB CD.
Find mBFC.

70

41

CHAPTER REVIEW TEST

1. The complement of an angle x is 10 more than


three times mx. Find the measure of the bigger
angle.
A) 50

B) 60

C) 70

D) 80

2. The sum of the measures of the supplementary


and complementary angles of an angle x is 250.
Find mx.
A) 10

B) 20

C) 30

7. In the figure,

m n,
mKAB = 130, and
mLCD = 40.
Find mABC.
A) 100

B) 90

bisectors of two adjacent supplementary angles?


B) 60

C) 75

D) 90

4. In the figure, [OA [OB,

C) 80

D) 70

8. In the figure,

A) 40

3x

d l. Find x.

6x10
4x20

B) 30

C) 20

9. In the figure,

[AB] [BE.
Find mx.

D) 10
B

x
70
D

C
60
130 E

A) 30

b
a

a 2 . Find b.
=
b 3

B) 36

C) 54

D) 60

5. The ratio of two complementary angles is 2 . Find


the measure of the supplementary angle

of the smaller angle.


A) 170

mCOB = b, and

A) 30

mAOC = a,

x
C

40

D) 40

3. What is the measure of the angle between the

A) 45

A
130

B) 160

C) 150

D) 110

B) 40

C) 50

10. In the figure,

[BC [DE and


[BA [DF.
What is the relation
between mx and
my?

Find mx.

115

x 35

F
B

y
C

A) mx + my = 90

B) mx + my = 180

C) mx = my

D) mx my = 30

11. In the figure,


6. In the figure, l k.

D) 60

[BC is the angle


bisector of ABC.
Find mx.

x
85
125

B
C

A) 110
42

B) 100

C) 90

D) 80

A) 65

B) 55

C) 50

D) 45
Geometriy 7

Objectives

After studying this section you will be able to:


1. Define a triangle.
2. Name the elements of a triangle.
3. Describe the types of triangle accordin to sides.
4. Describe the types of triangle according to angles.

A. THE TRIANGLE AND ITS ELEMENTS


The roofs of many buildings have a triangular cross-section. A triangle makes a simple musical instrument, and many traffic signs have a triangular shape. These are just some examples of how triangles are used in the world around us.

Challenge!
Without lifting your pencil,
join the following four
points with three segments
to form a closed figure.

Making a Poster - Triangles

Activity

Make a poster to show how triangles are used in everyday life. You can take photographs,
make drawings or collect pictures from magazines or newspapers to show buildings,
designs, signs and artwork which use triangles.
In this section we will consider the main features of triangles and how we can use them to
solve numerical problems.

1. Definition
The word triangle means three angles. Every triangle has three angles and three sides.
Definition

triangle, vertex, side


A triangle is a plane figure which is formed by three line segments joining three noncollinear
points. Each of the three points is called a vertex of the triangle. The segments are called the
sides of the triangle.

The plural of vertex is


vertices.

44

We name a triangle with the symbol


followed by three capital letters, each
corresponding to a vertex of the triangle. We
can give the letters in any order, moving
clockwise or counterclockwise around the
triangle.

Geometriy 7

For example, we can refer to the triangle shown at the right as ABC. We can also call it BCA,
CAB, ACB, BAC or CBA. The vertices of ABC are the points A, B and C. The sides of ABC
are the segments AB, BC and CA.
interior and exterior angles of a triangle

Definition

Notice that a triangle is


defined as the union of
three line segments. Since
an angle lies between two
rays (not two line segments),
a triangle has no angles by
this definition. However,
we can talk about the
angles of a triangle by
assuming the existence
of rays: for example, the
rays AB and AC form
angle A of a triangle ABC.
A
a

In a triangle ABC, the angles BAC, ABC and


ACB are called the interior angles of the
triangle. They are written as A, B and C,
respectively. The adjacent supplementary
angles of these interior angles are called the
exterior angles of the triangle. They are
written as A, B and C, respectively.

A
B

We can refer to the sides of a triangle ABC by


using the line segments AB, BC and AC, or by
using the lower-case form of the vertex
opposite each side.

For instance, in ABC at the right:


a is the side opposite vertex A,

b is the side opposite vertex B, and


c is the side opposite vertex C.

EXAMPLE

Look at the figure.

a. Name all the triangles in the figure.


b. Name all the interior angles of MNE.
c. Name all the vertices of NEP.
d. Name all the sides of MNP.

e. Name all the exterior angles of ENM.


Solution

Triangles and Construction

a. MNE, NEP and MNP


b. M (or NME), MNE and MEN.

d. segment MP, segment PN and segment


NM

c. points N, E and P

e. E, N and M

45

perimeter of a triangle

Definition

The sum of the lengths of the three sides of a triangle is called the perimeter of the triangle.
We write P(ABC) to mean the perimeter of a triangle ABC.

Challenge!
Move exactly three
toothpicks in the following
arrangement to make five
triangles.

For instance, the perimeter of the triangle


ABC in the figure is
P(ABC) = BC + CA + AB = a + b + c.

A
b

EXAMPLE

Solution

In the figure, P(ABC) = P(DEF). Find x.

P(ABC) = P(DEF)

D
10

x+2

14

16

x + 2 + x +10 = 16 + 14 + x (given)
2x + 12 = x + 30

x = 18

Check Yourself 1
1. Three distinct points K, M and N lie on a line m, and a fourth point T is not on the line
m. Point T is joined to each of the other points. Find how many triangles are formed and
name each one.
D
F
C
2. Find and name all the triangles in the figure at the right.
G

3. Polygon ABCDE is a regular polygon and its diagonals are


shown in the figure. Name
Triangles in the world
around us

a. all the triangles whose three vertices lie on the polygon.


b. all the triangles which have exactly one vertex on the
polygon.

A regular polygon is a
polygon in which all sides
have the same length and
all angles are equal.

46

c. all the triangles which have two sides on the polygon.

D
N

C
L

K
A

d. the sides of all the triangles which do not have a side on the polygon.
4. The side AC of a triangle ABC measures 12.6 cm, and the two non-congruent sides AB and
BC are each 1 cm longer or shorter than AC. Find P(ABC).
Geometriy 7

5. Point X is on the side KN of a triangle KMN. Find the length


of MX if the perimeters of the triangles KXM, XMN and KMN
are 24, 18, and 30, respectively.
Answers
1. Three triangles are formed: KMT, MNT and TKN.
2. AEL, LEB, LBC, AKL, AGK, ALB, ABC, AFC,
ADF, AGL, ADC

The picture shows the food


triangle of different types of food.
Can you see what the different
regions mean?

3. a. ABC, BCD, CDE, DEA, EAB


b. BKL, CLM, DMN, ENP, APK
c. ABC, BCD, CDE, DEA, EAB

d. sides of BKL: BK, KL, BL; sides of CLM: CL, ML, CM; sides of DMN: DM, MN, DN;
sides of ENP: EN, NP, EP; sides of APK: AP, KP, AK
4. 37.8 cm

5. 6

2. Regions of a Triangle
Any given triangle ABC separates the plane which contains it into three distinct regions:
1. The points on the sides of the triangle form the triangle itself.
2. The set of points which lie inside the triangle form the interior of the triangle, denoted
int ABC.
3. The set of points which lie outside the
triangle form the exterior of the triangle,
denoted ext ABC.
The union of a triangle with its interior and
exterior region forms a plane. In the figure
opposite, the plane is called E. We can write

exterior

interior
B

E = int ABC ABC ext ABC.


EXAMPLE

Write whether each statement is true or false


according to the figure opposite.

a. Point T is in the interior of DFE.


b. M ext BDE
c. ADF BED =

d. ext FDE int FCE = FCE

e. Points T and K are in the exterior of DFE.


Solution
Triangles and Construction

a. false

b. true

c. false

d. false

e. true
47

Check Yourself 2
A

Answer according to the figure.

a. Name five points which are on the triangle.

J
N

b. Name three points which are not on the triangle.


c. Name two points which are in the exterior of the triangle.

S
C

d. What is the intersection of the line ST and the triangle ABC?


A physical model of a
triangle with its interior
region

e. What is the intersection of the segment NS and the exterior of the triangle ABC?
Answers
a. points A, B, C, T and S b. points J, L and N c. points J and L d. points S and T
e.

3. Auxiliary Elements of a Triangle


Three special line segments in a triangle can often help us to solve triangle problems. These
segments are the median, the altitude and the bisector of a triangle.

a. Median
Definition

median
In a triangle, a line segment whose endpoints are a vertex and the midpoint of the side
opposite the vertex is called a median of the triangle.
In the figure, the median to side BC is the
line segment AD. It includes the vertex A and
the midpoint of BC.

Va

Auxiliary elements are


extra
or
additional
elements.

We usually use the capital letter V to indicate


the length of a median. Accordingly, the
lengths of the medians from the vertices of a
triangle ABC to each side a, b and c are
written as Va, Vb and Vc, respectively. As we
can see, every triangle has three medians.
48

Va
Vb
B

Vc
D

Geometriy 7

EXAMPLE

Name the median indicated in each triangle and indicate its length.
a.

b.

Solution

c.

a. median MD, length Vm


b. median TE, length Vt
c. median PF, length Vp

Paper Folding - Medians

Activity

1. Follow the steps to construct the median of a triangle by paper folding.

Take a triangular piece of paper


and fold one vertex to another
vertex.
This
locates
the
midpoint of a side.

Fold the paper again from the


midpoint to the opposite vertex.

DM is the median of EF.

2. Cut out three different triangles. Fold the triangles carefully to construct the three
medians of each triangle. Do you notice anything about how the medians of a triangle
intersect each other?

Definition

centroid of a triangle
The medians of a triangle are concurrent. Their common point is called the centroid of the
triangle.

Triangles and Construction

49

Concurrent lines are lines


which all pass through a
common point.

The centroid of a triangle is the center of gravity of the triangle. In other words, a triangular
model of uniform thickness and density will balance on a support placed at the centroid of
the triangle. The two figures below show a triangular model which balances on the tip of a
pencil placed at its centroid.

b. Angle bisector
Definition

triangle angle bisector


An angle bisector of a triangle is a line segment which bisects an angle of the triangle and
which has an endpoint on the side opposite the angle.
In the figure, AN is the angle bisector which
divides BAC into two congruent parts.
We call this the bisector of angle A
because it extends from the vertex A.
Since AN is an angle bisector, we can write
m(BAN) = m(NAC).
We usually use the letter n to indicate the
length of an angle bisector in a triangle.
Hence the lengths of the angle bisectors of a
triangle ABC from vertices A, B and C are
written nA, nB and nC, respectively. As we can
see, every triangle has three angle bisectors.

50

A
a a
nA

A
nA
nB
B

nC
N

Geometriy 7

Paper Folding - Angle Bisectors

Activity

Follow the steps to explore angle bisectors in a triangle.


1. Cut out three different triangles.
2. Fold the three angle bisectors of each triangle as shown below.
3. What can you say about the intersection of the angle bisectors in a triangle?

Folding the angle bisector of A.

Definition

The inscribed circle of a


triangle is a circle which
is tangent to all sides of
the triangle.

AN is the angle bisector of A.

BM is the angle bisector of B.

incenter of a triangle
The angle bisectors in a triangle are concurrent
and their intersection point is called the
incenter of the triangle. The incenter of a
triangle is the center of the inscribed circle of
the triangle.

A
L

N
C
O is the incenter of DABC

As an exercise, try drawing a circle centered at the incenter of each of your triangles from
the previous activity. Are your circles inscribed circles?
We have seen that nA, nB and nC are the bisectors of the interior angles of a triangle ABC. We
can call these bisectors interior angle bisectors. Additionally, the lengths of the bisectors of
the exterior angles A, B and C are
written as nA, nB and nC respectively. These
K
bisectors are called the exterior angle
bisectors of the triangle.
nK
In the figure at the right, segment KN is the
exterior angle bisector of the angle K in
KMT and its length is nK.
Triangles and Construction

51

excenter of a triangle

Definition

The bisectors of any two exterior angles of a


triangle are concurrent. Their intersection is
called an excenter of the triangle.

An escribed circle of a
triangle is a circle which is
tangent to one side of the
triangle and the extensions
of the other two sides.

EXAMPLE

In the figure, ABC is a triangle and the bisectors


of the exterior angles A and C intersect at
the point O. So O is an excenter of ABC. In
addition, O is the center of a circle which is
tangent to side AC of the triangle and the
extensions of sides AB and BC of the triangle.
This circle is called an escribed circle of
ABC.

A
T

C
S

As we can see, a triangle has three excenters and three corresponding escribed circles.

Find all the excenters of KMN in the figure


by construction.

N
M

Solution

To find the excenters, we first construct the


bisector of each exterior angle using the
method we learned in Chapter 1. Then we
use a straightedge to extend the bisectors
until they intersect each other.

E2
K
E1

The intersection points E1, E2 and E3 are the


excenters of KMN.

N
M

E3

52

Geometriy 7

c. Altitude
altitude of a triangle

Definition

An altitude of a triangle is a perpendicular line segment from a vertex of the triangle to the
line containing the opposite side of the triangle.
In the figure, AH is the altitude to side

BC because AH is perpendicular to BC.


ha

In a triangle, the length of an altitude is called a height of the triangle.


The heights from sides a, b and c of a triangle
ABC are usually written as ha, hb and hc,

respectively. As we can see, every triangle has


three altitudes.

hb
B

EXAMPLE

Name all the drawn altitudes of all the


triangles in the figure.

ha

hc
C

Solution

Triangles and Construction

There are eight triangles in the figure. Let us look at them one by one and name the drawn
altitudes in each.
53

D
altitude AK

altitude BK

altitudes BK and CK

altitudes
AK and BK

D
altitude DK

Activity

C A

K
C

D
altitude CK

altitudes CK and DK

D
altitudes
AK and DK

Paper Folding - Altitudes

To fold an altitude, we fold a triangle so that a side matches up with itself and the fold
contains the vertex opposite the side.

Cut out three different triangles. Fold them carefully to construct the three altitudes of
each triangle. What can you say about how the altitudes intersect?
54

Geometriy 7

orthocenter of a triangle

Definition

The altitudes of a triangle are concurrent. Their common point is called orthocenter of the
triangle.
Since the position of the altitudes of a
triangle depends on the type of triangle, the
position of the orthocenter relative to the
triangle changes. In the figure opposite, the
orthocenter K is in the interior region of the
triangle. Later in this chapter we will look
at two other possible positions for the
orthocenter.

hb
B

K
ha

hc

H
C
K is the orthocenter of DABC

Once we know how to draw an altitude of a triangle, we can use it to find the area of the
triangle.
area of a triangle

Definition

The area of a triangle is half the product of the length of a side (called the base of the
triangle) and the height of the altitude drawn to that base. We write A(ABC) to mean the
area of ABC.
For example, the area of ABC in the figure

BC AH a h
=
. Area is usually
is A( ABC ) =
2
2

expressed in terms of a square unit.


B

C
a

EXAMPLE

Find the area of each triangle.


a.

b.

12 cm

4 cm
B 3 cm H

Solution

7 cm

a. A( ABC ) =

BC AH
2

10 4
2
= 20 cm2

Triangles and Construction

2 cm
T

c.

6 cm

5 cm

8 cm

(Definition of the area of a triangle)


(Substitute)
(Simplify)
55

FT DE
2
5 14
=
2
= 35 cm2
KM MN
c. A( KMN ) =
2
6 8
=
2
= 24 cm2

b. A( DEF ) =

(Definition of the area of a triangle)


(Substitute)
(Simplify)
(Definition of the area of a triangle)
(Substitute)
(Simplify)

perpendicular bisector of a triangle

Definition

In a triangle, a line that is perpendicular to a side of the triangle at its midpoint is called a
perpendicular bisector of the triangle.

The picture below hangs


straight when the hook
lies on the perpendicular
bisector of the pictures
top edge.

In the figure, HN, DN and EN are the


perpendicular bisectors of triangle ABC.
Perpendicular bisectors in a triangle are
always concurrent.

A
E

D
N

circumcenter of a triangle

Definition

The intersection point of the perpendicular bisectors of a triangle is called the circumcenter
of the triangle. The circumcenter of a triangle is the center of the circumscribed circle of the
triangle.
The circumscribed circle
of a triangle is a circle
which passes through all
the vertices of the triangle.

EXAMPLE

Find the circumcenter of each triangle by construction.


a.

56

b.

c.

Geometriy 7

First we construct the perpendicular bisector of each side of the triangle. Their intersection
point is the circumcenter of the triangle.

Solution

a.

b.

c.

Perpendicular Bisector of a Triangle

Activity
There are three main faculties on
a university campus. The university
wants to build a library on the
campus so that it is the same
distance from each faculty.
1. Make a geometric model of the
problem.
2. Find the location of the library in
the picture opposite.

As an exercise, draw three more triangles on a piece of paper and construct their
circumcenters. Check that each circumcenter is the center of the inscribed circle.

Check Yourself 3
1. Name the auxiliary element shown in each triangle using a letter (n, h or V) and a vertex
or side.
a.

b.

Triangles and Construction

d.

N
B

c.

C N

e.

P
H

f.

M
N

57

2. In a triangle MNP, the altitude NT of side MP and the median MK of side NP intersect at
the point R.
b. Name two altitudes of MTN.

a. Name all the triangles in the figure formed.

3. In a triangle DEF, EM is the median of side DF. If DE = 11.4, MF = 4.6 and the perimeter
of DEF is 27, find the length of side EF.
4. In a triangle KLM, LN is the altitude of the side KM. We draw the angle bisectors LE and
LF of angles KLN and MLN respectively. If the angles between the angle bisectors and the
altitude are 22 and 16 respectively, find m(KLM).
A

5. In the figure, A(ABH) = A(AHC). Find x.

10

6. Write one word or letter in each gap to

H
A

make true statements about the figures.

a. Point O is a(n) __________ .

b. Segment ________ is a median.

c. Point _______ is an excenter.

d. Segment ________ is an altitude.

e. Point B is a(n) _____________.

f. Segment ER is a(n) __________

___________.

V
Z

M
T

g. Point _________ is a circumcenter.

h. Line TM is a(n) __________


__________.
i. Point ________ is a centroid.
Answers
1. a. nB b. hp c. Vx d. Vl e. hn f. nL

2. a. MNK, MKP, MNT, NTP, MRT, MNR, RNK, MNP


3. 6.4

4. 76

6. a. incenter
g. M
58

b. NT, TM

5. 5

b. ET

c. K

d. AB (or BC)

h. perpendicular bisector

e. orthocenter (or vertex)

f. angle bisector

i. G
Geometriy 7

B. TYPES OF TRIANGLE
Some triangles are given special names according to the lengths of their sides or the
measures of their angles.

1. Types of Triangle According to Sides


A triangle can be called scalene, isosceles or equilateral, depending on the lengths of its sides.
Definition

scalene triangle
A triangle is called scalene if all of its sides
have different lengths. In other words, a
scalene triangle has no congruent sides.

A
b

a b c, so DABC is a scalene triangle

Euler Lines

Activity
The Euler line of a triangle is the line which passes through
the orthocenter, circumcenter and centroid of the triangle.

centroid

orthocenter

circumcenter

Draw a scalene triangle and find its Euler line using


Euler line
Dynamic
geometry
software is a powerful tool
for studying geometric
concepts.
Geometry
programs allow us to
change and manipulate
figures, so that we can
explore and experiment
with
geometrical
concepts instead of just
memorizing them.

a. a ruler and set square.

b. a compass and straightedge.

c. dynamic geometry software.

Which method was easier?


Definition

isosceles triangle
A triangle is called isosceles if it has at least
two congruent sides.

B
C
a
b = c, so DABC is isosceles

Triangles and Construction

59

In an isosceles triangle, the congruent sides


are called the legs of the triangle. The third
side is called the base of the triangle.

vertex
angle

A
legs

The two angles between the base and the


legs of the triangle are congruent. They are
called the base angles of the triangle.

base

The angle opposite the base is called the


vertex angle.

base angles
B

Golden Triangles

Activity
The head of this knee
hammer forms an isosceles
triangle.

Line segments AB and


BC are in the golden
ratio if
a+ b a
= ,
a
b
=

1+ 5
1.6180339...
2

A golden triangle is a triangle in which the


ratio of the length of the legs to the length
of the base is the golden ratio. The angle
between the two legs of a golden triangle is
always 36.

36

golden
triangle
72

72

To construct a golden triangle, first draw a


square ABCD and mark the midpoint E of
AB. Find the point F on the extension of AB
by making EF = EC. Then find G by making
AF = AG and BA = BG. Finally, draw FG
and AG. Then BGF is a golden triangle.

The sides of the Great Pyramid


of Giza are golden triangles.
F

E
D

1. Construct a golden triangle using a straightedge and compass.


2. Repeat the construction using dynamic geometry software.
3. In both constructions, check the measures of the interior angles.

EXAMPLE

Solution

Segment EM is a median of an isosceles


triangle DEF with base DF. Find the length of
EM if the perimeter of EMF is 65 and the
perimeter of DEF is 100.

Let us draw an appropriate figure.

In the figure opposite,


in DEM, a + b + x = 65,

(1)

in DEF, 2(a + b) = 100. So a + b = 50.

(2)

Substituting (2) into (1) gives us 50 + x = 65; x = 15. So EM = 15.


60

Geometriy 7

EXAMPLE

10

In KMN, K . Given that KN is 4 cm


less than MN and MK is 2 cm more than
three times KN, find the perimeter of KMN.

x4

Solution

We begin by drawing the figure opposite.


If MK = x then KN = x 4. Also, MK = MN
because K N.
Also, we are given

MN = 3KN + 2
x = 3(x 4) + 2
10 = 2x
5 = x.

Since P(KMN) = 3x 4, P(KMN) = (3 5) 4 = 11 cm.

EXAMPLE

11

In ABC opposite, O is the intersection point


of the bisectors of the interior angles of the
triangle. Given that OE BC, OD AB,
AD = 4 cm, DE = 5 cm and EC = 6 cm, find
P(EOD).

A
4
D
5
O

E
6

Solution

Let us join points A and C to O. We know


from the question that OA and OC are the
bisectors of A and C, respectively.

A
4

Since OD AB,
m(OAB) = m(AOD).

(Alternate Interior
Angles Theorem)

So ODA is an isosceles triangle and


AD = OD = 4 cm. (1)

E
6
C

Similarly, since OE BC,


m(EOC) = m(OCB). (Alternate Interior Angles Theorem)
So EOC is also an isosceles triangle and OE = EC = 6 cm. (2)
By (1) and (2), P(EOD) = OE + OD + DE = 6 + 4 + 5 = 15 cm.
Triangles and Construction

61

equilateral triangle

Definition

A triangle is called equilateral if it has three


congruent sides.

a
How many equilateral
triangles can you see in
the figure below?

In an equilateral triangle, all of the interior


angles are congruent and measure 60.
Notice that an equilateral triangle is also an
isosceles triangle, but an isosceles triangle is
not always equilateral.

A
60

60

You will probably see two


triangles, one on top of the
other. This is actually an
optical illusion, though,
as the white triangle is
not actually drawn.

60

Toothpick Triangles

Activity

Find six toothpicks and try to do each thing below. Some things may not be possible.
Can you explain why?
1. Make one equilateral triangle with six toothpicks.
2. Make two equilateral triangles with six toothpicks.
3. Make three equilateral triangles with six toothpicks.
4. Make four equilateral triangles with six toothpicks.

EXAMPLE

12

Solution

The three sides of a triangle measure 5n + 8, n+12 and 3n+10 with n N. Which value of
n makes this triangle equilateral?
If the triangle is equilateral, all the sides must be congruent.
So 5n + 8
5n + 8
4n
n

=
=
=
=

n + 12 = 3n + 10. Let us solve the first equality to find n:


n + 12
4
1.

If we substitute 1 for n, the side lengths become (5 1) + 8 = 13, 1 + 12 = 13 and


(3 1) + 10 = 13. So the triangle is equilateral when n = 1.
62

Geometriy 7

Check Yourself 4
A

1. In ABC opposite, DE BC and point O is the incenter of the


triangle. If BD = 6 and EC = 4, find DE.
O

2. The perimeter of an isosceles triangle is 18.4 and its base measures 4 units more than the
length of one leg. Find the length of a leg of this triangle.
3. The sides of an isosceles triangle have lengths in the ratio 4 : 5 : 5. Find the length of the
base of the triangle if its perimeter is 28.
4. The perimeter of an isosceles triangle is 22.8. An equilateral triangle is drawn such that one
side is congruent to the base of the isosceles triangle. If the perimeter of the equilateral
triangle is 24.6, find the length of one leg of the isosceles triangle.
5. In an isosceles triangle NTM, MN = NT, MN = 35, TN = 4x +15 and MT = 40 x2. Find
MT.
6. In the figure, all triangles are equilateral,
AG = 24.12 cm and AC = 3CE = 2EG. Find
The picture shows a puzzle
the perimeter of each triangle.
called the Three Companions
Puzzle. Get your own and
try to free one of the triangles
from the string. Can you do 7.
it?

F
C

The three sides of a triangle measure 3a,


a+10 and 6a 15. Which value of a makes the triangle equilateral?

8. Construct an isosceles and an equilateral triangle.


Answers
1. 10

2. 4.8

3. 8

4. 7.3

5. 15

6. 12.6 cm, 6.3 cm, 4.2 cm

7. 5

2. Types of Triangle According to Angles


A triangle can be called acute, right or obtuse, depending on the measures of its angles.
Definition

acute triangle, right triangle, obtuse triangle


A triangle is called an acute triangle if all its angles are acute.
A triangle is called a right triangle if it has a right angle.
A triangle is called an obtuse triangle if it has an obtuse angle.

Triangles and Construction

63

Challenge!
Try to change the
equilateral triangle in the
figure so that it points
upwards by moving only
three balls.

Then try to make the


triangle in this figure
point downwards by using
the least number of balls
possible.

In a right triangle, the sides adjacent to the


right angle are called the legs of the triangle.
The side opposite the right angle is called the
hypotenuse of the triangle.

A
legs

hypotenuse

Note
Notice that a triangle can be only one of obtuse, acute or right.
Triangles
Obtuse

Acute

Scalene

EXAMPLE

13

Solution

EXAMPLE

14

Isosceles

Scalene

Equilateral

Right

Isosceles

Name all the right triangles in the figure.

Scalene

There are four smaller right triangles (ABK,


BKC, CKD and DKA) and four larger
triangles (ABC, BCD, CDA and DAB).

Classify each triangle according to its side lengths and angle measures.
a.

b.

c.
80

80

45

Solution
64

Isosceles

45

a. isosceles right triangle

70

30

b. scalene acute triangle

50

50

c. isosceles acute triangle


Geometriy 7

Tangram

Activity

Tangram is a fun puzzle and a good way to exercise your brain. The name comes from
tan, which means Chinese, and gram, which means diagram or arrangement. The
puzzle first appeared in China thousands of years ago, and it is now known all over the
world. There are seven pieces in a tangram set: five triangles, one square and one
parallelogram. The challenge of the puzzle is to use the seven pieces together to make
different shapes. You must use all the pieces, and they must all touch but not overlap.

All seven tangram pieces are made up of right triangles with this shape:

The first tangram challenge is to make a square with all seven pieces. The solution is
shown below.
1

2
6
4
7

Find a tangram set, or copy the figure above to make your own.
1. Make one right triangle using all of the pieces.
2. Can you make an obtuse triangle by using all of the pieces?
3. Can you make an acute triangle by using all of the pieces?

EXAMPLE

15

Draw a right triangle and divide it using


a. two parallel lines which are perpendicular to one of the legs.
b. two parallel lines which are not perpendicular to legs.
c. two perpendicular lines to create two more right triangles.
d. two intersecting lines which are not perpendicular to each other to create two more right
triangles.

Triangles and Construction

65

Solution

EXAMPLE

16

a.

b.

d.

Draw each triangle and use a set square to find its orthocenter. Write the orthocenter as an
intersection of lines or line segments.
a. acute scalene

Solution

c.

b. right scalene

c. obtuse scalene

Remember that the orthocenter of a triangle is the intersection point of its altitudes. We draw
the altitudes in each triangle by using a set square.
A

a. orthocenter K,
E

K = AF BD EC

30

60

K
B

b. orthocenter A,

D
C

A = AB CA AD

a 30-60 set square

c. orthocenter T,

45
45

C
A

T = AT BT TC
B

a 45 set square

Ealier in this chapter we said that the position of the orthocenter of a triangle depends on
the type of triangle. One position is in the interior of the triangle. Can you see what the other
two possible positions are, after studying the example above? How do they correspond to the
types of triangle shown?
66

Geometriy 7

Check Yourself 5
1. Classify each triangle according to its angle measures.
a.

b.

A
40

c.

d.

K
20

80
50

70

60

60

60

120

2. Name all the right triangles in the figure.

This sharks fin forms a


right triangle with the
water.

e.

C
E
A

3. At most how many of each type of angle can one triangle have?
a. acute angle

b. right angle

c. obtuse angle

(Hint: Try to draw a suitable figure for each case using a protractor.)
4. Draw a right triangle and divide it using
a. two intersecting lines which are perpendicular to each other.
b. two intersecting lines which are not perpendicular to each other, to make three more
right triangles.
5. Construct a right isosceles triangle.
Answers
1. a. right triangle
d. obtuse triangle
How many triangles?

b. acute triangle

c. acute triangle

e. obtuse triangle

2. DKB, KAB, KBC, KCD, KDA


3. a. three b. one c. one
4. a.

Triangles and Construction

b.

67

EXERCISES

3 .1

A. The Triangle and Its Elements

8. In a triangle ABC, two points different to A and B


on the side AB are joined to the vertex C by line
segments. Similarly, three points different to B
and C on side BC are joined to the vertex A by line
segments. How many regions inside the triangle
are formed by the intersection of these segments?

1. Find and name all the


triangles in the figure.

K
F

2. How many triangles can be formed by joining any


three points D, E, F and G if no three of the given
points are collinear? Name each triangle.
6
of AC, AB = BC and
5
AC = 15 cm. Find P(ABC).

9. In an isosceles triangle DEF, DF is the base


and FT is a median. Given that P(DEF) = 23 cm
and P(EFT) is 1 cm more than the perimeter of
triangle DTF, find DF.

3. In a triangle ABC, AB is

18
cm, MN is 75% of
5
KM and KN is 0.1 cm more than KM. Find

4. In a triangle KMN, KM =
P(KMN).

which is 2.6 cm less than the sum of the lengths


of the other sides. Find the perimeter of this
triangle.

6. Answer according to

the figure.

E
F
A

b. Name a point which is in the interior of


ADC.
c. What is the intersection of ABC and ADC?

as in the figure and


construct each element
separately, using a compass
and straightedge.
a. ha

5. The side AC of a triangle ABC measures 12.8 cm,

a. Name four
collinear points on
ABC.

10. Draw three triangles ABC

b. Va

c. nA

11. Draw four triangles KMN as in


the figure and find each
point separately using
only a compass and
straightedge.

a. centroid

b. incenter

c. orthocenter

d. circumcenter

12. Repeat question 11 with a protractor and ruler.


13. Find the excenters of the triangle in question 11
by using a protractor and ruler.

d. What is the intersection of ABC and int MAC?

14. The sides AB, BC and AC of a triangle ABC measure


7. Draw four figures to show how two triangles can
intersect to form a four-sided, five-sided,
six-sided and three-sided polygon.
68

13, 14 and 15 units respectively. Given that the


length of the altitude to side BC is 12, find the
lengths of the remaining altitudes.
Geometriy 7

B. Types of Triangle

21. Write always, sometimes or never to make true


statements.

15. Look at the figure and


name
a. an isosceles triangle.
b. three right triangles.

a. If a triangle is isosceles then it is ______________


equilateral.

F
60
B

30

c. an obtuse isosceles triangle.

b. If a triangle is equilateral then it is ___________


isosceles.
c. If a triangle is scalene then it is ______________
isosceles.

d. an acute triangle.

d. If a triangle is obtuse then it is _______________


isosceles.

e. an equilateral triangle.

16. Find the circumcenter of a right triangle using a


ruler and protractor.

17. State whether each type of triangle is possible or


not.

e. An obtuse triangle is __________________ a right


triangle.
f. In a triangle DEF, if DE EF then DF is
________________ perpendicular to EF.
g. A scalene triangle ________________________ has
an acute angle.

a. an isosceles acute triangle


b. a right equilateral triangle

h. If a triangle has two complementary angles


then it is ____________________ a right triangle.

c. a scalene acute triangle


d. an obtuse isosceles triangle
e. an obtuse equilateral triangle

18. The sides of a triangle measure 2x + 8, 3x 6,

22. In each case, draw a triangle with the given

and 12 + x.

property.

a. Find the value(s) of x that make(s) the triangle


isosceles.

a. All three angle bisectors are medians.

b. Which value(s) of x make(s) the triangle


equilateral?

19. The sum of the lengths of the legs of an


isosceles right triangle is 22 cm. Find the area of
this triangle.

20. Complete the table showing the location (in the


interior, on the triangle or in the exterior) of the
intersection of the segments or lines for each type
of triangle.
Perpendicular
bisectors
Acute
triangle
Right
triangle
Obtuse
triangle

Angle
bisectors

Medians

b. No altitude is a median.
c. Only one angle bisector is the perpendicular
bisector of a side.
d. Only one altitude is in the interior region of
the triangle.
e. The medians, altitudes and angle bisectors
coincide.
f. Exactly one of the three altitudes is also a
median.

Line containing
the altitudes

23. Divide any right triangle using two lines so that

a.

b.

c.

d.

e.

f.

g.

h.

the figure contains a total of

i.

j.

k.

l.

a. five right triangles.

Triangles and Construction

b. six right triangles.


69

Objectives

After studying this section you will be able to:


1. Identify congruent triangles
2. Construct a circle
3. Construct congruent segments
4. Find the midpoint of a segment
5. Construct perpendicular lines and parallel lines
6. Construct congruent angles and an angle bisector
7. Construct atriangle tram given information
8. Desctibe and use the properties of isosceles, equilateral and right triangles.
9. Describe and use the triangle Angle Bisector Theorem.

A. THE CONCEPT OF CONGRUENCE


In the previous section we studied triangles and their features and properties. In this section
we will look at possible relations between two or more triangles.
If we are given two triangles, how can we compare them? We might notice that they are the
same size and shape. This important relation in geometry is called congruence. Let us start
our study of congruence with a general definition of congruence in figures and polygons.

1. Congruent Figures
The world around us is full of objects of various shapes and sizes.
If we tried to compare some of these objects we could put them
in three groups:
objects which have a different shape and size,
objects which are the same shape but a different size, and
objects which are the same shape and size.
The tools in the picture at the right have different shape and size.
The pictures below show tools which have the same shape but different size. In geometry,
figures like this are called similar figures. We will study similar figures in Chapter 3.

Congruence is a basic
geometric relationship.

70

Geometriy 7

Factories often need to


produce many parts
with exactly the same
size and shape.

The pictures below show objects which are the same size and shape. In this section, we will
study figures which have this property.

congruent figures

Definition

Figures that have the same size and shape are called congruent figures. We say A is congruent
to B (or B is congruent to A) if A and B are congruent figures.
The pictures at the bottom of the previous page show some examples of congruent objects.
The pictures below show two more examples. In these two examples there is only one piece
left to fit in the puzzle. Therefore, without checking anything, we can say that each piece and
its corresponding place are congruent.

Congruence in nature:
the petals of this flower
are congruent.

Making a Poster - Congruent Figures

Activity

Make a poster to show congruent figures in everyday life. You can take photos, draw
pictures or collect pictures from magazines or newspapers that show buildings, designs,
signs and artwork with congruent parts.

EXAMPLE

17

Solution

Which piece is congruent to the empty space?


a.

b.

c.

d.

If we compare the vertices and sides, we can easily see that only c. fits into the space.

Triangles and Construction

71

Congruent Dissections

Activity

When you learned common fractions, you probably learned them by working with
figures divided into congruent parts. Often the figures are circles and rectangles, as
these are the easiest to divide into any number of congruent parts.
Dividing (also called dissecting) a figure into congruent parts can also be a puzzle. As an
example, can you see how to dissect the first figure below into two congruent pieces?
Answer:

Now try the two puzzles below. The answers are at the back of the book.
1. Dissect each figure into four congruent pieces.

A car has many


congruent parts.

2. The polygon below left can be dissected into four congruent polygons, as shown in the
figure below right. There is also a way to divide this polygon into five congruent
polygons. Can you find it?

72

Geometriy 7

We can think of congruent figures as figures that are exact copies of each other. In other
words, we can put congruent figures one on top of the other so that each side, angle and
vertex coincides (i.e. matches perfectly).
corresponding elements or parts

Definition

The points, lines and angles which match perfectly when two congruent figures are placed
one on top of the other are called corresponding elements or corresponding parts of the
congruent figures.

Sometimes we need to
move or modify a figure
to see that it is congruent
to another figure. The
basic changes that we
can make to a figure are
reflection
(flipping),
rotation (turning) and
translation (sliding). We
will study these in
Chapter 3.

We can see that by definition, corresponding parts of congruent figures are congruent. We
can write this in a shorter way as CPCFC.
You are already familiar with congruent segments (segments that have equal lengths) and
congruent angles (angles that have equal measures). In the rest of this section we will look
at congruent figures which are made up of segments and angles. These figures are polygons
and especially triangles.

2. Congruent Triangles
congruent triangles

Definition

Two triangles are congruent if and only if their corresponding sides and angles are congruent.
We write ABC DEF to mean that ABC and DEF are congruent.

Challenge!
Remove five toothpicks
to make five congruent
triangles.

In the figure below, ABC and DEF are congruent because their corresponding parts are
congruent. We can write this as follows:
A D

AB DE

B E

and

C F

BC EF
AC DF.

We can show this symbolically in a figure as follows:


A

Triangles and Construction

DABC

DDEF

73

EXAMPLE

18

Solution
A short history of the symbol:
Gottfried Wilhelm
Leibniz
(1640-1716)
introduced for congruence
in an unpublished manuscript in
1679.
In 1777,
Johann Friedrich
Hseler
(1372-1797)
used

(with the tilde reversed).


In 1824,
Carl Brandan
Mollweide
(1774-1825)

used the modern symbol for


congruence in Euclids Elements.

Given that MNP STK, state the congruent angles and sides in the two triangles without
drawing them.
The figure at the right shows how the
vertices of each triangle correspond to each
other. Because MNP STK and CPCTC
(corresponding parts of congruent triangles
are congruent), we can write
M S
N T
P K

DMNP

DSTK

M corresponds to S
N corresponds to T
P corresponds to K

MN ST
and

NP TK
PM KS.

As we can see, the order of the vertices in congruent triangles is important when we are
considering corresponding elements. Any mistake in the ordering affects the correspondence
between the triangles.
If two triangles are congruent then we can write this congruence in six different ways. For
instance, if ABC is congruent to DEF, the following statements are all true:
ABC DEF
ACB DFE
BAC EDF
BCA EFD
CAB FDE
CBA FED.

EXAMPLE

19

Solution

EXAMPLE

20

Complete each statement, given that PRS KLM.


a. PR _____

b. _____ K

c. _____ SP

d. S _____

e. ML _____

f. L _____

a. PR KL

b. P K

c. MK SP

d. S M

e. ML SR

f. L R

Decide whether or not the two triangles in


the figure are congruent and give a reason for
your answer.

4
60

30

74

Geometriy 7

Let us calculate the missing angles:

Solution

m(C) = 60

(Triangle Angle-Sum Theorem in ABC)

m(M) = 30

(Triangle Angle-Sum Theorem in KMN)

Now we can write the congruence of corresponding parts:


AB KM

(Given)

BC KN

(BC = KN = 4)

AC MN

(AC = MN = 8)

A M

(m(A) = m(M) = 30)

B K

(m(B) = m(K) = 90)

C N

(m(C) = m(N) = 60)

Therefore, ABC MKN by the definition of congruent triangles.

EXAMPLE

21

ABC EFD is given with AB = 11 cm, BC = 10 cm and EF + ED = 19 cm. Find the


perimeter of EFD.
Since ABC EFD, AB = EF, BC = FD and
AC = ED by the definition of congruence.
So by substituting the given values we get
11 = EF, 10 = FD and AC = ED.

Solution

Since we are given that EF + ED = 19 cm,


we have 11 + ED = 19 cm; ED = 8 cm.

11

11

10

10

So P(EFD) = EF + ED + FD = 11 + 8 + 10 = 29 cm.

Check Yourself 6
1. KLM XYZ is given. State the corresponding congruent angles and sides of the
triangles.
2. State the congruence JKM SLX in six different ways.

What would happen if


the blades of this ships
propellor or these wheels
were not congruent?

3. Triangles KLM and DEF are congruent. P(KLM) = 46 cm, the shortest side of KLM
measures 14 cm, and the longest side of the DEF measures 17 cm. Find the lengths of
all the sides of one of the triangles.
4. Triangles DEF and KLM are congruent. If DE = 12.5 cm, EF = 14.4 cm and the perimeter
of the triangle KLM is 34.6 cm, find the length of the side DF.

Triangles and Construction

75

5. Two line segments KL and AB bisect each other at a point T. If AL = 7 and the lengths of
the segments KL and AB are 22 and 18 respectively, find the perimeter of KTB.
Answers
1. KL XY

K X

LM YZ

L Y

KM XZ

M Z

2. PKM SLN, KMP LNS, MPK NSL,


PMK SNL, KPM LSN, MKP NLS

3. 14 cm, 15 cm, 17 cm

4. 7.7 cm

5. 27

B. CONSTRUCTIONS
In this section we will construct geometric figures using only two instruments,
a straightedge and a compass.

straightedge

1. Basic Constructions
We use a straightedge to construct a line,
ray, or segment when two points are given.
A straightedge is like a ruler without numbers.

compass

r
compass
point

r
O

We use a compass to construct an arc or a


circle, given a point O and a length r
(a radius).

pencil
point

Construction 1

Constructing a congruent segment.


Given [AB],
A

construct [CD] such that [CD] [AB].

C
Use a straightedge to draw
a line.
Mark a point C on the line.

A
Set your compass
to the length of [AB].

Using C as the center,


draw an arc intersecting
line [CD]. Label the point of
intersection D.

Result: [CD] [AB].


76

Geometriy 7

Construction 2

Finding the midpoint of a given segment.


Given [AB],
B

construct M such that such that [AM] [MB].


X
X

A
B

Y
Y
Using any radius greater
than 1 |AB| , draw an
2
arc with center A.

Draw an arc with the same


radius and center B.
Label the points of
intersection of the arcs
X and Y.

Draw [XY].
Mark and name
the intersection point M.

Result: M is the midpoint of [AB].


Construction 3

Constructing a perpendicular to a line at a given point on the line.


Given point M on the line l,
l

construct [MN] l.
N

Using M as the center and


any radius, use a compass to
draw arcs intersecting
l at A and B.

Using centers A and B


and a radius greater
than |MA|, draw two arcs
and find the intersection point N.

Draw [MN].

Result: [MN] l.
Triangles and Construction

77

Construction 4

Constructing a perpendicular to a given line through a point outside the given line.
N

Given line l and a point N outside the line,


construct [MN] l.

M
Using N as a center,
draw an arc that intersects
l at two points A and B.

Use A and B as centers to draw arcs


with the same radius that intersect
at a point M.

Draw [MN].

Result: [MN] l.
Construction 5

Constructing a congruent angle.

Given A, construct A such that A A.


A

B
C

Use a straightedge to draw a ray.


Name its initial point A.
Using a compass at center A,
draw BC. Keep the same
radius and draw an arc which
intersects the ray from A at point B.

B
Use |BC| as a radius and
center B to draw an
arc which intersects the
first arc at point C.

B
Draw [AC.

Result: AA.
78

Geometriy 7

Construction 6

Constructing a parallel to a line through a point outside the line.


N

Given line l with point N which is not on l,


construct a line through N which is parallel to l.

l
k
R

k
N

N
l

Q
l

At N, construct RNQ
congruent to NMP.

Draw a line k which intersects


line l at point M, and passes
through point N.

Result: l t.

Construction 7

Constructing an angle bisector.


C

Given CAB, construct the bisector of CAB.


A

C
D

Draw an arc BC with


center A.

With B and C as centers and a


radius greater than 1 |BC|,
2
draw arcs intersecting at D.

B
Draw [AD.

Result: [AD bisects A


Triangles and Construction

79

EXAMPLE

22

a. Construct two congruent line segments.

d. Construct an isosceles triangle.

b. Construct an obtuse angle and bisect it.

e. Construct an equilateral triangle.

c. Construct two parallel line segments.


Solution

a.

B
A
Use a straightedge to
draw [AB].
Set the compass at the
points A and B.

A
Draw another line l.
Choose any point on
line l and label it A.

b.

l
A
B
Use the radius |AB| and set
the compass point at A. Draw
an arc intersecting l.
Label the point of intersection
B. Now [AB] [AB]

B
C
Draw any obtuse angle ABC. Use B as the
center, and draw an arc AC. Next, draw two arcs,
one with center A and the other with center C.

B
C
Label the point D where the two
arcs intersect. Draw [BD.
[BD is the angle bisector of ABC.

c. Look at construction 6.
d.

l
A
B
Draw a line segment [AB]. Use any radius greater
1
than 2 | AB| and draw two arcs with centers A
and B. Name the intersection point C.

e.

A
B
Draw the triangle ABC.

|AC| = |BC|, so the triangle is


isosceles.
C

l
A
B
Use a straightedge to draw [AB]. Next, open
the compass to |AB| and draw two arcs, one
with center A and the other with center B.

80

l
A
B
Label the intersection point C. Draw [AC]
and [BC]. All the sides have equal length,
so ABC is an equilateral triangle.

Geometriy 7

Practice Problems 7
1. Construct a 30 angle. (Hint: construct a 60 angle and bisect it.)
2. Construct a right triangle with legs which are congruent to [AB]
and [CD] in the figure.

3. Construct a right triangle whose legs are in the ratio 2:1.


4. Construct a line segment and divide it into four equal parts.
Answers
C

1. By constructing equilateral triangles:

30

2.

Use a straightedge
to draw [AB]

Open your compass


more than [MA] and
draw two arcs, one
with center M, te
other with center N.
Draw a line from A
to the point of
intersection.

Open your compass


to [CD] and draw an
arc with center A.
Label the point of
intersection D. Draw
[AD] and [DB.

3.

Find the midpoint of


[AD] (see construction 2)

B A

B A

Draw a perpendicular
line to [AB] from A.
(see construction 3)

Open your compass


to as [AC] then draw
[AD] and [DB].

4.

Find the midpoint of [AB].

Triangles and Construction

Find the midpoint of [AC]


and the midpoint of [CB].

Label the new points D and


E.

81

2. Constructing Triangles
We can construct basic geometric figures using only a straightedge and a compass. However,
to construct triangles we need a compass, a ruler and a protractor. We use the ruler to
measure the sides of triangle, and the protractor to draw the angles.
We have seen that a triangle has six basic elements: three angles and three sides. To
construct a triangle, we need to know at least three of these elements, and one of these three
elements must be the length of a side. Let us look at the possible cases.

Note
In any triangle, the sum of any two given angles is less than 180 and the sides satisfy the
triangle inequality.

a. Constructing a Triangle from Three Known Sides


Let us construct ABC,

B a C

where |AB| = c, |BC| = a, and |AC| = b, given that

a < b < c.

C
B

Construction 1

Draw a line d.

Construction 2

Locate point A on d.

Construction 3

Open the compass as much as length c and put the


sharp point of the compass on A. Then draw an arc.

Name the intersection point B.

Construction 4

Again open the compass as much as length b and put the


sharp point on A. Then draw an arc on the upper side of
d.
82

Geometriy 7

Construction 5

Finally, open the compass as much as length a and put


the sharp point on B. Then draw an arc which intersects
the other arc drawn before. Name the intersection point
C.

C
c

Construction 5

After determining the point C, draw [AC] and [BC]. The


result is the constructed triangle.

a
c

Note
Remember that in a triangle, side a is opposite A, side b is opposite B, and side c is
opposite C. When we talk about side b we mean the side opposite B, or the length of this
side.

EXAMPLE

23

Construct ABC given |AB| = 10 cm, |BC| = 8 cm, and |AC| = 6 cm.

Solution

C
8 cm

6 cm
A

10 cm

Draw a line and


locate points A and B.

10 cm

8 cm

6 cm

C is the intersection point


of the two arcs.

10 cm

Connect the vertices.

b. Constructing a Triangle from Two Known Angles and a Known Side


Let us construct the triangle ABC, where A, B, and the side c are given.
Construction 1

Draw a line d.

Construction 2

Locate point A on the line.


Triangles and Construction

83

Construction 3

Using a protractor, take the point A as a vertex and draw


a ray [AX to construct A.

X
d

Construction 4

Using a compass, locate the point B on d such that


|AB| = c.

X
c

Construction 5

Using a protractor, take the point B as vertex and draw a


ray [BY to construct B. Label the intersection point of
[AX and [BY as C. The construction is complete.

24

EXAMPLE

Construct ABC given mB = 40, mC = 70, and |BC| = 12 cm.

Solution

40
B

70 C

40

40
B

Mark the point


B and draw B.

Y
A

C
12 cm
Using a compass
and ruler, find C.

12 cm
Draw C to find
the point A on [BX.

3. Constructing a Triangle from Two Known Sides and a Known


Angle
Finally, let us construct ABC given
|AB| = c, |BC| = a and the known angle B.

A
B

c
a

B
C

Construction 1

Draw a line d and use a compass to locate the points B


and C such that |BC| = a.
84

Geometriy 7

Construction 2

Use a protractor to construct B and the ray [BX.

Construction 3

Use a compass or ruler to locate the point A on [BX such


that |AB| = c.

c
a

Construction 4

Join the points A and C. The result is the constructed


triangle.

EXAMPLE

25

Construct ABC given |BC| = 5 cm, |AB| = 10 cm, and mB = 70o.

Solution

A
10 cm
70
B 5 cm C
Locate the points B and
C and draw B.

Triangles and Construction

70

A
10 cm
70

B 5 cm C

B 5 cm C

Locate A on [BX.

Join A and C.

85

Practice Problems 8
1. State the things you need to know in order to construct a triangle.
2. Draw an equaliteral triangle with sides 6 cm long.
3. Construct ABC given a = 5 cm, b = 4 cm and c = 2 cm.
4. Construct ABC given a = 7 cm, b = 6 cm and c = 8 cm.
5. Construct DEF given d = 6 cm, e = 8 cm and f = 10 cm.
6. Construct ABC given mA = 40o, mB = 65o and |AB| = 10 cm.
7. Construct KLM given mM = 45o, mL = 70o and |ML| = 7 cm.
8. Construct PQR given mR = 40o, mQ = 60o and |RQ| = 4 cm.
9. Construct MNP given mM = 30o, mN = 65o and |MN| = 15 cm.
10.Construct ABC given mB = 90o, |AB| = 5 cm and |BC| = 12 cm.
11.Construct PQR given mQ = 80o, |PQ| = 7 cm and |QR| = 4 cm.
12.Construct GHK given mH = 50o, |GH| = 6 cm and |HK| = 9 cm.
13.Construct XYZ given mY = 110o, |XY| = 3 cm and |YZ| = 5 cm.
14.Can you draw a triangle from only three given angles?

C.

ISOSCELES, EQUILATERAL AND RIGHT TRIANGLES


Isosceles, equilateral and right triangles are useful triangles because they have many special
properties. If we can identify one or more of these triangles in a figure then we can often use its
properties to solve a geometric problem. In this section we will look at some fundamental
theorems about isosceles, equilateral and right triangles, and some useful additional properties.

1. Properties of Isosceles and Equilateral Triangles


a. Basic Properties
Theorem

Isosceles Triangle Theorem


If two sides of a triangle are congruent then the angles opposite these sides are also
congruent.

Proof

Let us draw an appropriate figure.


Given: AB = AC
Prove: B C
B

86

Geometriy 7

Let AN be the bisector of A.


Statements

Reasons

1. AB AC

1. Given

2. BAN CAN

2. Definition of an angle bisector

3. AN AN

3. Reflexive property of congruence

4. ABN ACN

4. SAS Congruence Postulate

5. B C

5. CPCTC

Converse of the Isosceles Triangle Theorem

Theorem

If two angles in a triangle are congruent then the sides opposite these angles are also
congruent.
Proof

Let us draw an appropriate figure.


Given: B C
Prove: AB AC
We begin by drawing the bisector AN, and
continue with a paragraph proof.
Since AN is the angle bisector,

BAN CAN.
It is given that B C.
By the reflexive property of congruence, AN AN.
So ABN ACN by the AAS Congruence Theorem.
Since CPCTC, we have AB AC.
EXAMPLE

26

Solution

In a triangle DEF, T DF such that DT = DE. Given m(EDT) = 40 and m(DEF) = 85,
find m(TEF).
Let us draw an appropriate figure.

Since DE = DT, DET is an isosceles triangle.

40

So by the Isosceles Triangle Theorem,


m(DET) = m(DTE).
So by the Triangle Angle-Sum Theorem in
DET,
m(EDT) + m(DET) + m(DTE) = 180

T
F

40 + 2m(DET) = 180
m(DET) = 70.
So m(TEF) = m(DEF) m(DET) = 85 70 = 15.
Triangles and Construction

87

Corollary of the Isosceles Triangle Theorem

Corollary

If a triangle ABC is equilateral then it is also equiangular. In other words,


if a = b = c then m(A) = m(B) = m(C).

Corollary of the Converse of the Isosceles Triangle Theorem

Corollary

If a triangle ABC is equiangular then it is also equilateral, i.e. if m(A) = m(B) = m(C)
then a = b = c.

EXAMPLE

27

In the figure, ABC and DEF are equilateral


triangles. If BF = 17 cm and EC = 3 cm, find
AB + AH + DH + DF.

A
D

Solution

In the figure,
m(HCE) = 60 and m(HEC) = 60.

(ABC and DEF are equilateral)

So in HEC,
m(H) + m(E) + m(C) = 180

(Triangle Angle-Sum Theorem)

m(H) = 60.
So HEC is equiangular.

(m(C) = 60, m(E) = 60, m(H) = 60)

Therefore HEC is equilateral.

(By the previous Corollary)

So HE = HC = EC = 3 cm.
Let a and b be the lengths of the sides of ABC and DEF, respectively.
In ABC, AB = a, BE = a 3 and AH = a 3. (EC = 3 cm, given)
In DEF, DF = b, CF = b 3 and DH = b 3. (EC = 3 cm, given)
So

A
D
a

60

= 2(a + b) 6. (1)

Moreover, BF = a 3 + 3 + b 3 (Segment

Addition Postulate)
17 = a + b 3
20 = a + b. (2)

AB + AH + DH + DF = a + a 3 + b 3 + b

60
B

a3

60
60 60

E 3 C

60
b
60
b3

Substituting (2) into (1) gives us AB + AH + DH + DF = 34 cm.


88

Geometriy 7

Practice Problems 9
1. In a triangle ABC, the interior angle bisector at the vertex A makes an angle of 92 with
the side opposite A and has the same length as one of the remaining sides. Find all the
angles in ABC.
A

2. In the figure, CE is the angle bisector of C, HD BC and


HD = 5 cm. Find the length of AC.

E
H

3. In the figure, DCE is an equilateral triangle and DC = BC.


m( A )
1
is given. Find m(A).
=
m( B) 13

E
D

Answers
1. 8, 84 and 88

2. 10 cm

3. 5

b. Further properties
Properties 6

1. For any isosceles triangle, the following statements are true.


a. The median to the base is also the angle bisector of the vertex and the altitude to the
base.
b. The altitude to the base is also the angle bisector of the vertex and the median to the
base.
c. The angle bisector of the vertex is also the altitude and the median to the base.
In other words, if AB = AC in any triangle ABC then nA = Va = ha.
2. In an equilateral triangle, the medians, angle bisectors and altitudes from the same
vertex are all the same, i.e., ha = nA = Va, hb = nB = Vb, and hc = nC = Vc.
Moreover, all of these lines are the same length:
ha = hb = hc = nA = nB = nC = Va = Vb = Vc.
In other words, if AB = BC = AC in ABC then ha = hb = hc = nA = nB = nC = Va = Vb = Vc.
3. If AB = AC in any triangle ABC then nB = nC, Vb = Vc and hb = hc.
4. If ha = nA or ha = Va or nA = Va in ABC then ABC is isosceles.
Triangles and Construction

89

5. a. If ABC is an isosceles triangle with


AB = AC, P BC, E AC, D AB,
PE AB and PD AC,
then PE + PD = b = c.

b. If P, E and D are any three points on


the sides of an equilateral triangle ABC
such that PE and PD are parallel to
two distinct sides of ABC, then
PE + PD = AB = BC = AC.

E
B

6. a. In any isosceles triangle ABC with


AB = AC, the sum of the lengths of two
lines drawn from any point on the base
perpendicular to the legs is equal to
the height of the triangle from the
vertex B or C. In other words, if
AB = AC, P BC, H AC, D AB,
PH AC and PD AB, then
PH + PD = hc = hb.

D
B

b. In any equilateral triangle ABC, the sum of the lengths of two lines drawn from any
point on any side perpendicular to the other sides is equal to the height of the triangle
from any vertex. In other words, if AB = BC = AC, P BC, H AC, D AB, PH AC
and PD AB, then PH + PD = ha = hb = hc.
7. In any equilateral triangle ABC,
if P int ABC and points D, E and F
lie on the sides of ABC such that
PE AB, PD AC and PF BC, then
PE + PF + PD = AB.

A
D
P

90

Geometriy 7

8. In any equilateral triangle ABC,


if P int ABC and points D, E and F lie
on the sides of ABC such that PE AB,
PD AC and PF BC,
then PD + PE + PF = AH where
AH BC.

A
D
E
P

EXAMPLE

28

Solution

An isosceles triangle TMS has base TS which measures 8 cm and perimeter 32 cm. The
perpendicular bisector of leg TM intersects the legs TM and MS at the points F and K,
respectively. Find the perimeter of TKS.
Let us draw an appropriate figure.

Since hk = Vk in TKM, TKM is isosceles


by Property 6.4.
So TK = KM.

29

Solution

6
F

(1)

By the Segment Addition Postulate,


MK + KS = MS.
(2)

EXAMPLE

Since TMS is isosceles and


P(TMS) = 32 cm,
TM = MS = 12 cm.

(3)

So P(TKS) =
=
=
=

(By (1))
(By (2))
(By (3))

TK + KS + ST
KM + KS + ST
MS + ST
12 + 8 = 20 cm.

6
T

In the figure,
AB = AC,
PD AC and
PE AB.
Given AB + AC = 32 cm, find PD + PE.
Since AB + AC = 32 cm and
AB = AC, we have AB = AC = 16 cm.
So by Property 6.5b, PD + PE = AB = 16 cm.

Triangles and Construction

K
S

A
E
D

91

EXAMPLE

30

Solution

In the figure opposite, H, A and B are


collinear points with CH HA,
PE AC, PD AB and AB = AC.
PE = 6 cm and HC = 10 cm are given.
Find the length of PD.

10

By Property 6.6a, PE + PD = CH.

So 6 + PD = 10 and so PD = 4 cm.
EXAMPLE

31

In the equilateral triangle ABC shown


opposite, PD BC, PE AB and PF AC.
PE = 6 cm, PF = 5 cm and
P(ABC) = 42 cm are given.
Find the length of PD.

A
E
6
D

P
5

Solution

EXAMPLE

32

Solution

42
=14 cm.
3
By Property 6.7, PD + PE + PF = AB. So PD + 6 + 5 = 14 and so PD = 3 cm.

Since ABC is equilateral and its perimeter is 42 cm, AB =

Prove that in any isosceles triangle, the median to the base is also the angle bisector of the
vertex and the altitude to the base.
A

Look at the figure.


Given: AT is a median and AB = AC.
Prove: AT bisects A and is an altitude of
ABC.
We will write a two-column proof.
B

Proof:
Statements

92

Reasons

1. AC AC

1. Given

2. ABC ACB

2. Isosceles Triangle Theorem

3. BT TC

3. AT is a median.

4. ABT ACT

4. By 1, 2, 3 and the SAS Congruence Postulate

5. m(TAB) = m(TAC)

5. By 4 (CPCTC)

6. AT bisects A

6. Definition of angle bisector

7. m(ATB) = m(ATC)

7. By 4 (CPCTC)

8. m(ATB) = m(ATC) = 90

8. Linear Pair Postulate

9. AT is also an altitude of ABC

9. Definition of altitude

Geometriy 7

Practice Problems 10
A

1. In the figure, AD and BE are the interior angle


bisectors of A and B, respectively.
AC = BC and BE + AD = 12 cm are given.
Find the value of BE AD.

2. In an equilateral triangle ABC, H BC, N AC, BN is the interior angle bisector of B,


and AH is the altitude to the side BC. Given BN = 9 cm, find AH.
A

3. In the figure, BH = HC,


m(HAC) = 20 and
m(BCD) = 30.
Find m(BDC).

20

?
B

30

C
A

4. In the figure, AB = AC = 13 cm,


PE AC, PF AB and PE = 4 cm.
Find the length of PF.

E
4
B

5. In the figure, AB = AC, PE HB,

C
H

PF AC and BH HC.
Given CH = 12 cm,
find the value of PE + PF.

12

C
A

6. In the figure, AB = BC,


PD = x + 3, PH = 3x 1, and
AE = 14. Find the value of x.

D
P

5. 12 cm

6. 3

Answers
1. 36
Triangles and Construction

2. 9 cm

3. 80

4. 9 cm

93

2. Properties of Right Triangles


a. The Pythagorean Theorem
The Pythagorean Theorem is one of the most famous theorems in Euclidean geometry, and
almost everyone with a high school education can remember it.
Theorem

Pythagorean Theorem
In a right triangle ABC with m(C) = 90,
the square of the length of the hypotenuse
is equal to the sum of the squares of the
lengths of the legs, i.e.
2

c =b +a.
C

Proof

There are many proofs of the Pythagorean


Theorem. The proof we will give here uses
the dissection of a square. It proves the

Pythagorean Theorem for the right triangle


ABC shown opposite.

Imagine that a large square with side length


a + b is dissected into four congruent right
triangles and a smaller square, as shown in
the figure. The legs of the triangles are a and
b, and their hypotenuse is c. So the smaller
square has side length c.

a
b

We can now write two expressions for the area S of the larger square:
ab
2
2
S= 4
+ c and S = (a + b) .
2

Since these expressions are equal, we can write

ab
2
2
4
+ c = ( a + b)
2

2ab + c2 = a2 + 2ab + b 2
c2 = a2 + b 2 .
This concludes the proof of the Pythagorean Theorem.
94

Geometriy 7

Try the following activity to discover two more proofs of the Pythagorean Theorem.

The Pythagorean Theorem

Activity

1. Cut out a square with side length c, and cut out four identical right triangles with
hypotenuse c and legs a and b. Place the four triangles over the square, matching the
hypotenuses to the sides and leaving a smaller square uncovered in the centre. Try
to obtain the Pythagorean Theorem by considering the area of the original square
and the areas of the parts that dissect it.

2. Cut out or construct two squares with sides a and b. Try


to dissect these squares and rearrange their pieces to
form a new square. Then use the areas of the original
squares and the new square to write the Pythagorean
Theorem. (Hint: Start by cutting out two right triangles
with legs a and b.)
EXAMPLE

33

In the figure, ST SQ. Find x and y.

S
13

Solution

First we will use the Pythagorean Theorem


in SKT to find x, then we can use it in
SKQ to find y.
SK 2 + KT 2 = ST2
x2 + 122 = 132

x
T

12

(Pythagorean Theorem in SKT)


(Substitute)

x + 144 = 169
x2 = 25
x = 5 is not an answer
because the length of a
segment cannot be negative.
So the answer is x = 5.
From now on we will
always consider only
positive values for lengths.

Triangles and Construction

x=5
SK2 + KQ2 = SQ2
2

5 +y =6

y2 =36 25

(Simplify)
(Positive length)
(Pythagorean Theorem in SKQ)
(Substitute)
(Simplify)

y = 11
95

EXAMPLE

34

Solution

In the figure,
PT = TS = KS,
PM = 4 cm and KM = 3 cm. Find ST.

Let PT = TS = KS = x.

So SM = KM KS = 3 x and TM = PM PT = 4 x.
In TMS,
Quadratic formula
The roots x1 and x2 of
the quadratic equation
ax2 + bx + c = 0 are
x1,2 =

b b2 4ac
.
2a

TS2 = TM2+ MS2


2

(Pythagorean Theorem)

x = (4 x) + (3 x)
2

(Substitute)
2

x = 16 8x + x + 9 6x + x

(Simplify)

x 14x + 25 = 0
x1, 2 = (7 24) cm

(Quadratic formula)

Since 7 + 24 is greater than 3 and 4 which are the lengths of the sides, the answer is
x = |ST| = 7 24 cm.
EXAMPLE

35

In the figure,
m + k = 3 n.
Given A(KMN) = 30 cm2,

find the value of n.


M

Solution

m + k = 3 n (1)

(Given)

A(KMN) = 30 cm2
km
= 30
2

(Given)

k m = 60

(Definition of the area of a triangle)

(2)

In KMN, n2 = k2 + m2

(Pythagorean Theorem)

n2 = (k + m)2 2km

(Binomial expansion: (k+ m)2 = k2 + 2km + m2)

n2 = (3n)2 2 60

(Substitute (1) and (2))

8n = 120

(Simplify)

n = 15
n = 15 cm.
Theorem

Converse of the Pythagorean Theorem


If the square of one side of a triangle equals the sum of the squares of the other two sides,
then the angle opposite this side is a right angle.

96

Geometriy 7

Proof

We will give a proof by contradiction.


Suppose the triangle is not a right triangle,
and label the vertices A, B and C. Then there
are two possibilities for the measure of angle
C: either it is less than 90 (figure 1), or it is
greater than 90 (figure 2).

a
figure 2

A
c

a
figure 1

By the Pythagorean Theorem in BCD,


BD2 = a2 + b2 = c2, and so BD = c.

However, in figure 3 we have

Let us draw a segment DC CB such that


DC = AC.

So ACD is isosceles (since DC = AC) and


ABD is also isosceles (AB = BD = c). As a
result, CDA CAD and BDA DAB.

c
b

b
a
figure 3

figure 4

m(BDA) < m(CDA) and m(CAD) < m(DAB), which gives m(BDA) < m(DAB) if
CDA and CAD are congruent. This is a contradiction of BDA DAB. Also,
in figure 4 we have m(DAB) < m(CAD) and m(CDA) < m(BDA), which gives
m(DAB) < m(BDA) if CAD and CDA are congruent. This is also a contradiction.
So our original assumption must be wrong, and so ABC is a right triangle.

EXAMPLE

36

Solution

In the triangle ABC opposite, K AC and


AN is the interior angle bisector of A.
AB = 16 cm, AN = 13 cm, AK = 12 cm and
NK = 5 cm are given. Find the area of
ABN.

H
16

N
13

Let us draw an altitude NH from the vertex


N to the side AB.

5
K

12

To find the area of ABN we need to find NH, because A( ABN ) =


as 16 cm.

Triangles and Construction

NH AB
and AB is given
2

132 = 122 + 52, so m(NKA) = 90.

(Converse of the Pythagorean Theorem)

Also, NH = NK = 5 cm.
NH AB 5 16
So A( ABN ) =
=
= 40 cm 2 .
2
2

(Angle Bisector Theorem)


(Substitution)
97

We can use the Pythagorean Theorem to prove other theorems. Here is one example.

Theorem

Carnots Theorem
If a triangle ABC with P int ABC has
perpendiculars PK, PN, and PT drawn to the
sides BC, AC and AB respectively, then
AT 2 + BK 2 + CN 2 = AN2 + BT 2 + CK2.

A
N

T
P

Proof

Let us join the point P to the vertices A, B


and C and use the Pythagorean Theorem in
the six right triangles that are created.

A
N

T
P

In ATP, AT2 + TP2 = AP2.

(Pythagorean Theorem)

In ANP, AN + NP = AP ,

(Pythagorean Theorem)

AT2 + TP2 = AN2 + NP2. (1)

(Transitive property of equality)

In BKP, BK2 + KP2 = BP2.

(Pythagorean Theorem)

In BTP, BT + TP = BP ,

(Pythagorean Theorem)

BK + KP = BT + TP . (2)

(Transitive property of equality)

In CNP, CN2 + NP2 = CP2.

(Pythagorean Theorem)

In CKP, CK2 + KP2 = CP2,

(Pythagorean Theorem)

CN + NP = CK + KP . (3)

(Transitive property of equality)

From (1), (2) and (3) we get


AT2 + TP2 + BK2 + KP2 + CN2 + NP2 = AN2 + NP2 + BT2 + TP2 + CK2 + KP2 (Addition property
of equality)
AT 2 + BK 2 + CN 2 = AN2 + BT2 + CK2.

98

(Simplify)

Geometriy 7

EXAMPLE

37

Find the length x in the figure.

Solution

AT2 + BK2 + CN2


x2 + 42 + (2x)2
5x2
x2
x

=
=
=
=
=

AN2 + BT2 + CK2


22 + 42 + 62
40
8
22

2x

(Carnots Theorem)
(Substitute)
(Simplify)

Practice Problems 11
1. Find the length x in each figure.
a.

A Pythagorean triple is a
set of three integers a, b
and c which satisfy the
Pythagorean Theorem.
The smallest and
best-known Pythagorean
triple is (a, b, c) = (3, 4, 5).

b.

D
x

d.
6
L

N
5
x

17
10

24

7
E
9

e.

17

c.

f.

x
15

15

x
N 19 K

2. In the figure, TN = NK, ST = 12 cm

12

and SN = 69 cm. Find the length of TK.


12
69
T

3. In a right triangle ABC, m(A) = 90, AB = x, AC = x 7 and BC = x + 1. Find AC.


Answers
1. a. 15 b. 25 c. 9 d. 53 e. 20 f. 10
Triangles and Construction

2. 10 cm

3. 5 cm
99

b. Further properties

Activity

Paper Folding - Median to the Hypotenuse

Fold a corner of a sheet of paper, and cut along the fold to get a right triangle.
Label the vertices A, B and C so that m(B) = 90.
Fold each of the other two vertices to match point B.
Label the point T on the hypotenuse where the folds intersect.
What can you say about the lengths TA, TB and TC? Repeat the steps with a different
right triangle, and see if your conclusions are the same.
Properties 7

1. The length of the median to the hypotenuse of a right triangle is equal to half of the length
of the hypotenuse.
2. a. In any isosceles right triangle, the length of the hypotenuse is 2 times the length of
45-9
90 Triangle Theorem.)
a leg. (This property is also called the 45-4
b. In any right triangle, if the hypotenuse is 2 times any of the legs then the triangle is
45-9
90
a 45-45-90 triangle. (This property is also called the Converse of the 45-4
Triangle Theorem).
3. In any 30-60-90 right triangle,
a. the length of the hypotenuse is twice the length of the leg opposite the 30 angle.
b. the length of the leg opposite the 60 angle is 3 times the length of the leg opposite
60-9
90 Triangle Theorem.)
the 30 angle. (These properties are also called the 30-6
4. In any right triangle,
a. if one of the legs is half the length of the hypotenuse then the angle opposite this leg
is 30.
b. if one of the legs is 3 times the length of the other leg then the angle opposite this
60-9
90
first leg is 60. (These properties are also called the Converse of the 30-6
Triangle Theorem.)
A
5. The center of the circumscribed circle of
any right triangle is the midpoint of the
hypotenuse of the triangle.

100

r
B

Geometriy 7

EXAMPLE

38

In the figure,
m(BAC) = 90,
m(C) = 60 and
BD = DC.
Find BC if AD = 2x + 3 and AC = 6x 1.

C
60
6x 1

D
2x + 3

Solution

Since AD is a median and the length of the median to the hypotenuse of a right triangle
1
is equal to half the length of the hypotenuse, AD = BC.
2
By the Triangle Angle-Sum Theorem in ABC, m(B) = 30.
By the 30-60-90 Triangle Theorem, AC =
hypotenuse.

1
BC because m(B) = 30 and BC is the
2

So by the transitive property of equality, AC = AD, i.e. 6x 1 = 2x + 3 and so x = 1.


Finally, BC = 2 AC = 2 AD = 2 (2x + 3) = 10.

EXAMPLE

39

Solution

In the figure at the right, find m(ADC) if


m(BAC) = 90,
m(BAD) = 2x,
m(ACB) = 3x and
BD = DC.

A
2x
3x
B

1
BC.
2
So AD = BD = DC. Hence DCA and BDA are isosceles triangles.

Since AD is a median, by Property 7.1 we have AD =

Since DCA is isosceles, m(DAC) = m(ACD) = 3x.


Additionally, m(BAC) = m(BAD) + m(DAC) by the Angle Addition Postulate.
So 2x + 3x = 90 and x = 18.
By the Triangle Angle-Sum Theorem in DCA, m(ADC) + 3x + 3x = 180.
So m(ADC) = 180 (6 18), i.e. m(ADC) = 72.
Triangles and Construction

101

EXAMPLE

40

Solution

One of the acute angles in a right triangle measures 16. Find the angle between the
bisector of the right angle and the median drawn from the same vertex.
Let us draw an appropriate figure. We need
to find m(NAT).

A
45

According to the figure,


AN is the angle bisector, AT is the
median, and m(BAC) = 90.

16

m(ACB) = 16 by Property 5.3.


Property 5.3:

Since AT is median to hypotenuse, AT = CT = BT.

In any triangle ABC, if


m(B) > m(C) or
m(B) < m(C) then
ha < na < Va.

So ATC is isosceles.
Therefore, by the Isosceles Triangle Theorem, m(TAC) = m(ACT) = 16.
Since AN is an angle bisector and m(BAC) = 90, m(NAC) = 45.
So m(NAT) = m(NAC) m(TAC)= 45 16 = 29.

EXAMPLE

41

In the figure, AB AC and AH BC.

Given m(C) = 30 and BH = 2 cm, find the


length of HC.
30
B

Solution

In ABC, since m(C) = 30,


A

m(B) = 60.
In ABH, since m(B) = 60,

4 30

m(BAH) = 30.
In ABH, by Property 7.3,
AB = 2 BH = 2 2 = 4 cm.
This set square is in the
form of a 30-60-90
triangle.

In ABC, again by Property 7.3,

60

60
B

30
H

C
8

BC = 2 AB = 2 4 = 8 cm.
So HC = BC BH = 8 2 = 6 cm.

102

Geometriy 7

EXAMPLE

42

Find the value of x in the figure.

60

10

Solution

Let us draw an altitude from C to AB.

45

In BHC,

BC = 2 BH

AB = AH + HB

Solution

60

(Segment Addition
Postulate)
(Substitute)
(Simplify)

In AHC,
AC = 2 AH
AC = 2 4
AC = x = 8.

43

(Substitute)
(Simplify)

10 = AH + 6
AH = 4.

EXAMPLE

62

(45-45-90 Triangle
Theorem)

62 = 2 BH
BH = 6.

This set square is in the


form of 45-45-90 right
triangle.

45
B

62

(30-60-90 Triangle Theorem)


(Substitute)
(Simplify)

Construct a 30 angle by using the Converse of the 30-60-90 Triangle Theorem.


We will construct a right triangle in which one leg is half of the hypotenuse. Then by the
Converse of the 30-60-90 Triangle Theorem, the angle opposite this leg will measure 30.
1.

2.

3.

4.

5.

Triangles and Construction

7.

A
r

6.

B
r r Kr Cr
2 2
2
2

B
n

r K r C
2
2

30

A
r
B

r K r C
2
2

103

1. Draw a line m.
2. Construct a perpendicular to the line at any point. Name the line n and label the
intersection point of these lines B.
3. Draw an arc with center B and any radius r. Label the intersection point C of this arc and
the line m.
4. Construct the midpoint K of the segment BC and draw the line k perpendicular to m
through this point.
5. Draw an arc with center K and radius BC. Label the intersection point A of the arc and line n.
6. Join A and K to make AK = 2 KB.
7. By the Converse of the 30-60-90 Triangle Theorem, m(BAK) = 30.
EXAMPLE

44

Solution

Construct the circumscribed circle of a given right triangle.


1.

2.

3.

1. Label ABC with m(A) = 90 and m(B) > m(C).


2. Construct the perpendicular bisector of each side and label their point of intersection M.
3. Draw a circle with center M and radius MB. This is the circumscribed circle.

Activity

Unsolved Problem - Kobon Triangles

The Kobon triangle problem is an unsolved problem in


geometry which was first stated by Kobon Fujimura. The
problem asks: What is the largest number of non-overlapping
triangles that can be produced by n straight line segments?
This might seem like a simple question, but nobody has yet
found a general solution to the problem.
The mathematician Saburo Tamura proved that the largest
integer below n(n 2) / 3 is an upper bound for the number of
non-overlapping triangles which can be produced by n lines. If
the number of triangles is exactly equal to this upper bound,
the solution is called a perfect solution. Perfect solutions are
known for n = 3, 4, 5, 7, 9, 13 and 15, but for other n-values
perfect Kobon triangle solutions have not been found.

A perfect Kobon solution with


15 lines and 65 triangles
(T. Suzuki, Oct. 2, 2005)

The perfect Kobon solution for five lines creates 5 (5 2) / 3 = 5 triangles. Can you find
this solution?
Make Kobon patterns with seven lines. Can you find the perfect solution for seven lines?
104

Geometriy 7

Check Yourself 12
1. In an isosceles right triangle, the sum of the lengths of the hypotenuse and the altitude
drawn to the hypotenuse is 27.3. Find the length of the hypotenuse.
C

2. In the figure, ABC is a right triangle with


m(ABC) = 90 and CF = FE, and CE is the

angle bisector of C. If m(ADB) = 102, find

102

the measure of CAB.


A

3. One of the acute angles in a right triangle measures 48. Find the angle between the
median and the altitude which are drawn from the vertex at the right angle.
4. In a triangle ABC, m(B) = 135, AC = 17 cm and BC = 82 cm. Find the length of AB.
5. In a right triangle, the sum of the lengths of the hypotenuse and the shorter leg is 2.4.
Find the length of the hypotenuse if the biggest acute angle measures 60.
A

6. In the figure,
m(C) = 60,
HC = 4 cm and
DH = 23 cm. Find the length AC = x.

x
23

60
4

H
A

7. ABC in the figure is an equilateral triangle with


DH BC,

x
D

BH = 5 cm and
HC = 3 cm.
Find the length AD = x.
B

8. The distance from a point to a line k is 10 cm. Two segments non-perpendicular to k are
drawn from this point. Their lengths have the ratio 2 : 3. Find the length of the longer
segment if the shorter segment makes a 30 angle with k.
9. CAB is a right triangle with m(A) = 90 and m(C) = 60, and D is the midpoint of
hypotenuse. Find the length of the hypotenuse if AD = 3x + 1 and AC = 5x 3.
10.The hypotenuse of an isosceles right triangle measures 18 cm. Find the distance from the
vertex at the right angle to the hypotenuse.
Answers
1. 18.2
Triangles and Construction

2. 22

3. 6

4. 7 cm

5. 1.6

6. 5 cm

7. 2 cm

8. 30 cm

9. 14 10. 9 cm
105

D. THE TRIANGLE ANGLE BISECTOR THEOREM


Theorem

Triangle Angle Bisector Theorem


1. The bisector of an interior angle of a

triangle divides the opposite side in the


same ratio as the sides adjacent to the
angle. In other words, for a triangle ABC
and angle bisector AN,

AB BN
=
.
AC CN

2. In any triangle ABC, if the bisector AN of

the exterior angle A cuts the extension


of side BC at a point N, then
B

AB BN

.
AC CN

Proof of 1

We begin by drawing two perpendiculars NK


and NL from N to the sides AB and AC
respectively, then we draw the altitude
AH BC.

L
K

AH BN
BN
A( ABN )
2
=
=

(1) (Definition of the area of a triangle and simplify)


A( ANC )
AH NC CN
2
Now let us find the same ratio by using the sides AB and AC and the altitudes NK and NL.

Since N is the point on the angle bisector, by the Angle Bisector Theorem we have NK = NL.
NK AB
AB
A( ABN )
2
=
=

AC
A( ANC )
NL AC
2

106

AB BN
=
AC CN

(2) (Definition of the area of a triangle and simplify)

(By (1), (2) and the transitive property of equality)

Geometriy 7

Proof of 2

We begin by drawing two perpendiculars NK


and NL from N to the extension of the sides
AB and AC respectively, then we draw the
altitude AH BN.

AH BN
BN
A( ABN )
2
=
=

A( ACN )
AH CN CN
2

(1) (Definition of the area of a triangle and simplify)

Now let us find the same ratio by using the sides AB and AC, and the altitudes NK and NL.
Since N is the point on the angle bisector, by the Angle Bisector Theorem we have NK = NL.
NK AB
AB
A( ABN )
2
=
=

AC
A( ACN )
NL AC
2
AB BN
=

AC CN

EXAMPLE

45

Solution

(By (1), (2) and the transitive property of equality)

Find the length x in the figure.


12 6
=
8
x
3 6
=
2 x
x=4

Triangles and Construction

(2) (Definition of the area of a triangle and simplify)

(Triangle Angle Bisector

12

Theorem)
B

107

EXAMPLE

46

Solution

Find the length x in the figure.


A

12
4
=
12+ x x

x
4

(Triangle Angle Bisector


Theorem)

12

3
1
= ; 3x =12+ x ; x = 6.
12+ x x

Properties 8

1. In any triangle ABC, if AN is an interior


angle bisector then

AN2 = (AB AC) (BN NC).

2. In any triangle ABC, if AN is an


exterior angle bisector then

47

Solution

AN2 = (BN CN) (AB AC).

EXAMPLE

Find the length x in the figure.

By the Triangle Angle Bisector Theorem,


6 3
=
y 2
y = 4.
By Property 8.1,

x2 = 6 4 3 2 = 18
x = 32.
108

Geometriy 7

EXAMPLE

48

In the figure, m(CAB) = 2 m(ABC).


Given that AC = 4 cm and AB = 5 cm, find
the length of BC.

Solution

Let AD be the bisector of angle A.


Then m(B) = m(DAB) = m(DAC), since m(CAB) = 2 m(ABC).
So DAB is an isosceles triangle. Let AD = DB = x. If BC = a then CD = a x.
By the Triangle Angle Bisector Theorem in BAC,
5
x
=
4 ax
5(a x) = 4x
5a
. (1)
x=
9
Now we can use Property 8.1:

ax
D
4

x
A

x2 = 5 4 x(a x)
x2 = 20 ax + x2
ax = 20. (2)
Substituting (1) into (2) gives
5a
a
= 20 ; a2 = 36 ; a = 6 cm.
9

Check Yourself 13
1. In a triangle ABC, N is on side BC and AN is the angle bisector of A. If AB = 8 cm,
AC = 12 cm and BC = 10 cm, find AN.
2. In a triangle KMN, points M, Z, N and T are collinear, KZ is the angle bisector of the
interior angle K, and KT is the angle bisector of the exterior angle K. If MZ = 5 cm and
ZN = 3 cm, find NT.
3. MP is the angle bisector of the interior angle M of a triangle KMN. Find MN if
KP : PN = 3 : 4 and KM = 15 cm.
4. In a triangle ABC, point D is on side BC and AD is the bisector of the angle A. Find BD if
BA : AC = 5 : 3 and BD + DC = 8 cm.
5. ET is an angle bisector in a triangle DEF. Find the length of ET if DE = 14, EF = 21 and
DF = 15.
Answers
1. 62 cm
Triangles and Construction

2. 12 cm

3. 20 cm

4. 5 cm

5. 415
109

EXERCISES

3. 2

A. The Concept of Congruence

5. Two triangles ABC and CMN are congruent to


each other with AB = 8 cm, CN = 3 cm, and
CM = AC = 6 cm. Find BC and MN.

1. Find two pairs of congruent figures in each


picture. Draw each pair.
a.

b.

B. Constructions
6. Construct each angle.
a. 15

b. 105

c. 75

d. 37.5

e. 97.5

7. a. Construct an isosceles triangle with base 5 cm


and perimeter 19 cm.

2.

70
A

D
C

10 b
(x+10)

20y 3

M
a+2
M

(n+10)
E

b. Construct an equilateral triangle with perimeter


18 cm.

c. Try to construct a triangle with sides of length


2 cm, 3 cm and 6 cm. What do you notice?
Can you explain why this is so?

7
80
P

10y+7

In the figure, polygon ABCDE is congruent to


polygon KLMNP. Find each value, using the
information given.
a. x

b. y

c. n

d. a

d. Construct a triangle ABC with side lengths


a = 5 cm and c = 7 cm, and m(B) = 165.
e. Construct a right triangle ABC in which
m(C) = 90, b = 5 cm and the hypotenuse
measures 7 cm.

e. b

f. Construct ABC with angles m(C) = 120


and m(B) = 45, and side b = 5 cm.

3. ADE KLN is given. List the congruent


corresponding segments and angles of these
triangles.

g. Construct an isosceles triangle PQR with


vertex angle m(Q) = 40 and side QP = 3 cm.

4. A triangle ABC is congruent to a second triangle


KMN. Find the unknown value in each statement,
using the properties of congruence.
a. AC = 5m 25, KN = 11 m

8. In each case, construct a triangle using only the

three elements given.

b. m(BCA) = 45 v, m(MNK) = 2v 15,

a. a, b, Vc

b. a, b, hc

c. m(B) = 18, m(M) = u 4,

c. a, b, nC

d. ha, hb, hc

d. BA = 22x 30, MK = 3 2x

e. A, a, ha

f. Va, Vb, Vc

110

Geometriy 7

C. Isosceles, Equilateral and Right


Triangles
9.

14. In the figure,

KMN is an isosceles triangle with base KN. The


perpendicular bisector of the side MK intersects
the sides MK and MN at the points T and F,
respectively. Find the length of side KN if
P(KFN) = 36 cm and KM = 26 cm.

ABC is an equilateral
triangle, PE AC,
PD BC, and PF AB.
Given P(ABC) = 45,
find the value of
PD + PE + PF.

15. In the figure,


10 . In the figure,
AB = BC,
AD = DB
and BE = EC.
If DC = 3x + 1 and
AE = 4x 1,
find the length x.

BH = HC,
AB = DC and
m(B) = 54.
Find m(BAC).

A
D

11. In a triangle DEF, m(E) = 65 and m(F) = 15,


and DK is an angle bisector. Prove that DEK is
isosceles.

m(LMK) = 70 and m(KLM) = 80.


O int KLM and KO = LO = MO are given.
Find m(OKM), m(OML) and m(OLK).

13. In the figure,

ABC is an equilateral
3
P
triangle.
E
PE BC, PD AC,
5
PE = 3 and
PD = 5 are given.
B
D
C
Find the length of one side of the equilateral
triangle.
Triangles and Construction

?
54
B

16. In the figure,


BE = EC and
AD bisects the interior
angle A.
Given AB = 12 and
AC = 7, find the
length of ED.

A
12

O is the center of the


inscribed circle of
ABC, OB = CD,
AB = AC and
m(ECD) = 20.
Find the measure of
BEC.

7
D

17. In the figure,


12. In a triangle KLM, m(LKM) = 30,

E
O

D
20

18. In a right triangle ETF, the perpendicular


bisector of the leg ET intersects the hypotenuse at
the point M. Find m(MTF) if m(E) = 52.

19. In triangle DEF, DE = EF, m(DEF) = 90 and


the distance from E to DF is 4.8 cm. Find DF.
111

20 . TF is the hypotenuse of a right triangle TMF, and


the perpendicular bisector of the hypotenuse
intersects the leg MF at the point K. Find
m(KTF) given m(MTF) = 70.

21.

25. In the figure,

AB = AC,
AD = DB and
CD = 8 cm.
Find the length of DB.

8
D

26. In a triangle ABC, BC = 72, m(BAC) = 30 and


m(BCA) = 45. Find the length of the side AB.
15

In the figure, PMN is a right triangle, MH PN


MH
and m(N) = 15. Find
.
PN
(Hint: Draw the median to the hypotenuse.)

22 . In the figure,
?
A
BC = 12,
m(BAC) = 90,
m(ADC) = 90 and
60
m(ABC) = 60.
B
If AC is the angle
12
bisector of C, find the length of AD.

23. In the figure,

28. In the figure,

112

m(AHC) = 90,
m(B) = 30 and
HC = 1 cm.
Find the length of BH.

30. In the figure,

60
A

N
150 33
2

m(BAC) = 90,

m(M) = 150,
LM = 2 and
MN = 33.
Find the length of KL.

29. In the figure,

m(A) = m(B) = 60,


AD = 7 and
BC = 4.
Find DC.

30

24. In the figure,

ABC measures 45. The altitude drawn from the


obtuse angle B divides the opposite side into two
segments of length 9 and 12. Find the length of
the side BC.

m(A) = 30 and
AB = AC = 16 cm.
Find the value of
PH + PD.

27. The larger acute angle A in an obtuse triangle

C
4
60
B

m(DBA) = 30,
m(ABC) = 60 and
AD = 4. Find the
length of BC.

30
?

D 4

1 C

30
60
B

Geometriy 7

31. ABC in the figure is

an equilateral triangle.
BH = 5 cm and
HC = 3 cm are given.
Find the length
AD = x.

36. In a right triangle KLM, m(KLM) = 90 and the


perpendicular bisector of the leg LM cuts the
hypotenuse at the point T. If m(LMK) = 20,
find m(TLK).

32. In the figure,

37. In the figure,

m(C) = 90,
m(BAD) = m(DAC),
BD = DA and
DC = 3 cm. Find the
length of BD.

m(A) = 90,
m(ADC) = 60,
m(B) = 30 and
BD = 4 cm.
Find the length of AD.

60

39. In the figure,

ABC is an equilateral
E
triangle, PE AC and
PD AB.
PD = 5 cm,
PE = 3 cm and
D
P(ABC) = 48 cm are B
given. Find the length of PH.

24

In the figure, NK is the bisector of the interior


angle N and NL = LK. m(NMP) = 90 and
m(P) = 24 are given. Find m(PSM).

m(BAE) = m(DAE) = 60,


CB = 6 cm and
CE = 3 cm.
Find the length of CD.

30
C

35. In the figure,

Triangles and Construction

?
N

34. In the figure,

E
B

ABC is an equilateral
triangle, BD = 4 cm
and AE = 6 cm. Find
the length of EC.
B

30

38.
33. In the figure,

ABC is an isosceles
triangle, AB = AC,
m(BAC) = 30 and
2PE = PD = 4.
Find AC.

60 60

B
6
C 3 E

40. In the figure,

3
?

5
C

AB = BC
m(ADB) = 30 and
AC = 6.
Find the length of CD.

D
30

113

41. The base and a leg of an isosceles triangle


measure 14 cm and 25 cm respectively. Find the
height of the altitude drawn to the base.

42. A rectangular opening in a wall has dimensions


21 cm by 20 cm. Can an empty circular service
tray with a diameter of 28 cm pass through the
opening?

43. Two ships are in the same port. One starts to


travel due west at 40 km/h at 3 p.m. Two hours
later the second ship leaves port, traveling due
south at 60 km/h. How far apart will the ships be
at 7 p.m.?

48. In the figure,

ED = AC,
BD = DC and
m(C) = 63. Find
m(EDC).

angle of a right triangle divides the right angle in


the ratio 13 : 5. Find the smallest angle in this
triangle.

45. In a triangle DEF, m(DEF) = 120, DE = 25

63

?
B

49. In the figure,

AB = AC = 18 cm,
PH = 5 cm and
PD = 4 cm.
Find m(HPD).

5
B

50. In the figure,


44. The median drawn from the vertex at the right

ABC is an equilateral
triangle, AH = ED,
AE = EC and
CD = 4 cm.
Find the length of
AB.

and DF = 8. Find the length of the side EF.

51. In the figure,


46. In the figure,

BD = DC and
BC = 12.
Find the length of AD.

?
B

47. In the figure,


AD = EB,
CD = DB and
m(DEB) = 80. Find
the measure of ACB.

114

AB = AC,
AH = HB,
m(A) = 120 and
PB = 8 cm. Find
the length of CP.

?
P
8

120
A

52. In the figure,

C
?

AT = DB = DC and
m(A) = 36.
Find m(DBT).

80
A

36

T
E
B

Geometriy 7

53. A line m divides a segment KN with the ratio 2 : 3


at the point M. Find the distances from the points
K and N to m if KN = 40 and the angle between
m and the segment KN is 30.

59. In the figure,

NL = LK and
K
NK is the bisector
L
120
of N. If
S
m(NMP) = 90 N
and m(MSP) = 120, what is m(P)?

54. In a triangle KMN, T is the intersection point of


the three angle bisectors and the distance from T
to the side MN is 4. Find the distance from T to
the vertex K if m(K) = 60.

55. Prove that in a right triangle ABC with m(A) = 90


and acute angles 15 and 75, the altitude to the
1
hypotenuse measures
of the length of the
4
hypotenuse.

60. In the figure,


BE = EC, BD AC,
m(A) = 45,
m(D) = 30 and
DC = 6. Find the
length x.

D
A

45

30
E

D. The Triangle Angle Bisector Theorem


56. In the figure,

AC BD,
CE = 2AB and
m(C) = 15.
Find m(CAF).

61. In a triangle ABC, D AB and CD is the interior

15

angle bisector of C. Given AC = 9 cm, BD = 8 cm,


AD = m and BC = n, find the value of m n.

?
A

62. In a triangle ABC, D lies on the side BC and AD is


57. Prove that if AH is an
altitude and AD is a
median of a triangle
ABC then
|b2 c2| = 2a HD.

the interior angle bisector of A. If AC = BD,


AB = 9 cm and DC = 4 cm, find the length of BD.

63. In a triangle ABC, D lies on the side AC and BD is


B

58. In the figure,

ABC is an equilateral
triangle, m(BCE) = 15,
EF = FC, DF EC and
AD = 2 cm are given.
Find AE.

Triangles and Construction

2
D

64. In a triangle ABC, points C, A and D are collinear

E
F
B

the interior angle bisector of B. If AD = 5 cm,


DC = 6 cm and the sum of the lengths of the sides
a and c is 22 cm, find the value of a.

15

and C, B and E are also collinear. BD is the angle


bisector of EBA. If AC = AD and AB = 6 cm,
find the length of BC.
115

65. In a triangle EFM, points F, N and M are collinear

71. In a triangle ABC, D AB, E BC and AE and CD

and EN is the interior angle bisector of E.


If EN = 6, NM = 3 and FN = 4, find the length
of EF.

are the angle bisectors of A and C respectively,


intersecting at the point K. If AD = 4, DB = 6 and
AK
AC = 8, find the value of
.
KE

66. In a triangle ABC, D lies on the side BC and AD is


the interior angle bisector of A. If BD = 3 cm,
DC = 4 cm and AB + AC = 14 cm, find the length
of AD.

67. In a triangle ABC, D lies on the side BC and AD is


the interior angle bisector of A. If AB = AD = 12 cm
and AC = 16 cm, find the length of BD.

68. In a triangle DEF, E, F and K are collinear and

72. In a triangle ABC, points D, B and C are collinear


and AD is the angle bisector of A. If AB = 2m,
AC = 2m + 6, BC = 2m + 2 and DB = 28, find
the value of m.

73. In the figure, AN and BE


are the angle bisectors of
A and B respectively.
If AB = 4,
AC = 6 and BC = 5,
find the length of BE.

4
?
B

DK is the exterior angle bisector of the angle D. If


DE = 4 cm, DF = 3 cm and EF = 2 cm, find DK.

BE are the angle


bisectors of A and
B, respectively,
AE 3
= and
ED 2
B
AC = 12 cm.
Find the length of segment DC.

12

74. In the triangle


69. In the figure, AD and

ABC at the right,


AD is the angle
bisector of A
N
4
2
and BN is the
angle bisector of
m
3
D
B
C
B. DB = m,
BC = 3, AB = 4 and NC = 2 are given. Find the
value of m.

70. In a triangle ABC, points B, C and D are collinear

75. In a triangle ABC, points D, B and C are collinear

and AD is the angle bisector of A. If CD = 3 BC


and AB = 12 cm, find the length of AC.

and AD is the angle bisector of A. If AB = 12


and BD = 4 BC, find the length of AC.

116

Geometriy 7

Objectives

After studying this section you will able to


1. Describe and use relations between angles
2. Describe and use relations between angles
In this section we will look at some basic rules related to the basic and auxiliary elements of
a triangle.

Angles in a Triangle

Activity
Do the following activities and then try to

A. RELATIONS
BETWEEN ANGLES
find a common conjecture.
1. Cut out three identical triangles and
label their vertices K, M and N. Draw a
straight line and place the triangles
along the line as in the diagram at the
far right. What can you say about the
sum of K, M and N?

It is indeed wonderful
that so simple a figure
as the triangle is so
inexhaustible in
properties.
How many as yet
unknown properties of
other figures may there
not be?
August Crelle
(1780-1856),
civil engineer and
mathematician

Triangles and Construction

K
M
N

M
N

MN K

2
2. Cut out an acute triangle, a right triangle

and an obtuse triangle. Number the


1
3
1 2 3
angles of each triangle 1, 2 and 3 and
1
tear them off. Finally, put the three
angles of each triangle adjacent to each

other to form one angle as in the


1
2
2
figures at the far right. What can you
1
say about the sum of the angles in each

triangle?
2
3
1 2 3
3. Cut out a triangle ABC and label its
longest side BC. Fold the triangle so that A lies on the fold line and C lies on BC. Label
the intersection T of BC and the fold line. Unfold. Now fold the paper so that A, B and
C coincide with T. How does this activity support the results of activities 1 and 2 above?

117

Triangle Angle-S
Sum Theorem

Theorem

The sum of the measures of the interior angles of a triangle is 180.


Proof

Given: ABC

A
4 2 5

Prove: m(1) + m(2) + m(3) = 180


We begin by drawing an auxiliary line DE
through A, parallel to BC. Then we continue

with a two-column proof.


Statements

An auxiliary line is a
line which we add to a
figure to help in a proof.

EXAMPLE

49

Solution

Reasons

1. 1 4; 3 5

1. Alternate Interior Angles Theorem

2. m(DAE)=m(4)+m(2)+m(5)=180

2. Angle Addition Postulate, by the definition of straight angle

3. m(1) + m(2) + m(3) = 180

3. Substitute 1 into 2

In the figure, points A, B, F and E, B, C are


respectively collinear. Given that C and F
are right angles, m(E) = 40 and m(A) = ,
find the value of .

A
a
E

40

B
F

In EFB by the Triangle Angle-Sum Theorem,


m(E) + m(F) + m(B) = 180

40 + 90 + m(B) = 180

m(B) = 50.
By the Vertical Angles Theorem,
m(EBF) = m(ABC)
50 = m(ABC).

40

50

In ABC by the Triangle Angle-Sum Theorem,


m(A) + m(B) + m(C) = 180
+ 50 + 90 = 180
= 40.
118

Geometriy 7

50

EXAMPLE

In a triangle ABC, m(A) = 3x 10, m(B) = 2x + 20 and m(C) = 5x.


Find the value of x.
m(A) + m(B) + m(C)
3x 10 + 2x + 20 + 5x
10x + 10
x

Solution

=
=
=
=

180
180
180
17

(Triangle Angle-Sum Theorem)

Activity
Complete the table for the figure at the right, using the
Triangle Angle-Sum Theorem and the Linear Pair Postulate.
m(4)

m(3)

75

55

m(2)

63
77

m(1)

m(3) + m(4)
1

135

46
39

85

What do you notice about the values in the last two columns of the table?

Theorem

Triangle Exterior Angle Theorem


The measure of an exterior angle in a triangle is equal to the sum of the measures of its two
nonadjacent interior angles.

Proof

Given: ABC

Prove: m(1) = m(3) + m(4)


1 2
The two interior angles
which are not adjacent
to an exterior angle in a
triangle are sometimes
called remote angles.
exterior
angle

remote
angles

Triangles and Construction

Statements
1. m(1) + m(2) = 180
m(2) = 180 m(1)

Reasons
1. Linear Pair Postulate

2. m(2) + m(3) + m(4) = 180

2. Triangle Angle-Sum Theorem

3. 180 m(1) + m(3) + m(4) = 180

3. Substitute 1 into 2.

4. m(1) = m(3) + m(4)

4. Simplify.

119

EXAMPLE

51

Solution

In a triangle ABC, AB AC and m(B) = 136. Find m(C).


A

m(B) = m(A) + m(C)


136 = 90 + m(C)
136

46 = m(C)

EXAMPLE

52

In the figure, AB = BD, AD = DC

C
A

and m(DAC) = 35. Find m(B).

35
B

Solution

m(DCA) = m(DAC)

(Base angles of an isosceles triangle)

= 35
m(BDA) = m(DAC) + m(DCA)

(Triangle Exterior Angle Theorem)

= 35 + 35
= 70
m(BAD) = m(BDA)

(Base angles of an isosceles triangle)

= 70
m(B) + m(BAD) + m(BDA) = 180

(Triangle Angle-Sum Theorem)

m(B) + 70 + 70 = 180
m(B) = 40
EXAMPLE

53

In a triangle KMN, D lies on side KM. Decide whether each statement about the figure is
possible or impossible. If it is possible, sketch an example.
a. Triangles KDN and MDN are both acute triangles.
b. Triangles KDN and MDN are both right triangles.
c. Triangles KDN and MDN are both obtuse triangles.
d. Triangle KDN is obtuse and triangle KNM is acute.

Solution

a. impossible

b. possible

c. possible
N

d. possible
N

N
10

100

120

20

30

50
120

50

55
80

45

Geometriy 7

Triangle Exterior Angle-S


Sum theorem

Theorem

The sum of the measures of the exterior angles of a triangle is equal to 360.
Proof

Given: ABC

Prove: m(A) + m(B) + m(C) = 360

We will give a flow-chart proof.


B

m(A) = m(B) + m(C)

Triangle Exterior Angle Theorem


m(B) = m(A) + m(C)
Triangle Exterior Angle Theorem

m(A)+m(B)+m(C) =

m(A)+m(B)+m(C) =

= 2(m(A)+m(B)+m(C))
Addition Property of Equality

=2 180 = 360
Triangle Angle-Sum Theorem

m(C) = m(A) + m(B)


Triangle Exterior Angle Theorem

EXAMPLE

54

In the figure, m(TCA) = 120,


m(KAB) = 5x and m(PBC) = 7x.

K
5x

a. Find the value of x.

b. Find m(BAC).
P

Solution

a. m(A) + m(B) + m(C)


5x + 7x + 120
12x
x

=
=
=
=

b. m(KAB) + m(BAC) = 180


(5 20) + m(BAC) = 180
m(BAC) = 80
Triangles and Construction

360
360
240
20

120

B
7x

(Triangle Exterior Angle-Sum Theorem)

(Linear Pair Postulate)

121

EXAMPLE

55

In the figure,
m(A) = m,
m(B) = n and m(C) = k.

A
m
n

Find the value of m if m + n + k = 280.


P

Solution

m(A) + m(A) = 180

(Linear Pair Postulate)

m(A) = 180 m
180 m + n + k = 360

(Triangle Exterior Angle-Sum Theorem)

n + k = 180 + m (1)
Also,

m + n + k = 280

m + 180 + m = 280

180 m

(Given)
(Substitute (1))

2m = 100
m = 50.

n
P B

C
k

Check Yourself 14
1. The two acute angles in a right triangle measure 0.2x + 6.3 and 3.8x 2.7. Find x.
2. The measures of the interior angles of a triangle are in the ratio 4 : 6 : 8. Find the degree
measures of these angles.
3. The vertex angle of an isosceles triangle measures 42. An altitude is drawn from a base
angle to one of the legs. Find the angle between this altitude and the base of the triangle.
4. In an isosceles triangle, the angle between the altitude drawn to the base of the triangle
and one leg of the triangle measures 16 less than one of the base angles of the triangle.
Find the measure of the vertex angle of this triangle.
5. Two points E and F are drawn on the extension of the side MN of a triangle MNP such
that point M is between the points E and N and point N is between points M and F.
State which angle is the smallest angle in EPF if EM = MP, NF = NP, m(PMN) = 30
and m(PNM) = 40.
122

Geometriy 7

6. In a triangle DEF, point M lies on the side DF such that MDE and DEM are acute
angles. Decide whether each statement about the figure is possible or impossible. If it is
possible, sketch an example.
a. FME is an acute triangle.
b. FME is a right isosceles triangle.
c. FME and DME are both acute triangles.
d. DME is equilateral and EMF is isosceles.
e. DME is isosceles and DEF is isosceles.

7. In the figure, KLN is an isosceles triangle in a plane,

m(KLN) = 120, and L is the midpoint of the segment KM.


A point P is taken in the same plane such that MP = KL. Find
the measure of LPM when

120

a. the distance between N and P is at its maximum.

b. the distance between N and P is at its minimum.

8. One of the exterior angles of an isosceles triangle measures 85. Find the measure of the
vertex angle of this triangle.

9. State whether each triangle is a possible or impossible figure. If it is possible, sketch an


example. If it is impossible, give a reason why.
a. A triangle with two obtuse exterior angles.
b. A triangle with one acute exterior angle.
c. A triangle with two right exterior angles.
d. A triangle with two acute exterior angles.

10.Find the value of x in each figure, using the information given.


a.

b.

A
70

Triangles and Construction

105
x

115

80

c.

d.

72

x
N

40

3x
P

D
x

60
E

123

Answers
1. 21.6 2. 40, 60 and 80 3. 21 4. 74 5. PEF
6. a. possible

b. possible

65

40 45

20
60

40

7. a. 30 b. 60

c. not possible

F D

e. possible

60 30
30

50

55

d. possible

60 60

45

30

30

F D

30

8. 95

9. a. possible

b. possible
100
130 50
30

150

15

c. impossible because the third exterior angle would be 180


d. impossible because the third exterior angle would have to be more than 180
10. a. 45 b. 25 c. 27 d. 80
So far we have looked at some basic properties of angles. Now we will study some other
useful and important properties.
Properties 3

1. For any triangle, the following statements are true:


a. The measure of the angle formed by the bisectors of two interior angles of the triangle
is 90 more than half of the third angle, i.e. in the figure,
m( BAC )
A
m(BKC) = 90 +
.
S
2
b. The measure of the angle formed by
the bisectors of two exterior angles of a
triangle is 90 minus half of the third
angle, i.e. in the figure,
m( BAC )
.
m(BTC) = 90
2
c. The measure of the angle which is
formed by the bisector of one interior
angle and the bisector of a second
exterior angle is the half the measure
of the third interior angle.

90 +

a
2

a
2

90 a
2
T

We can refer to properties 1a, 1b and 1c as the Angle Bisector Relations Theorem.
124

Geometriy 7

2. In any triangle, the measure of the angle


formed by the altitude and the angle
bisector which both extend from the
same vertex is equal to the half the
absolute value of the difference of the
other two angles of the triangle.

A
x

H
x=

|m(B) m(C)|
2

3. In any triangle KLM, if N is any point in


the interior of KLM then

a. m(LNM) = m(LKM) + m(KLN)


+ m(KMN).

b. m(KNM) = m(KLM) + m(LKN)


+ m(LMN).

c. m(KNL) = m(KML) + m(MLN) + m(MKN).

EXAMPLE

56

The triangle ABC at the right has incenter O.


Find m(AOC).

O
80
B

Solution

Since the incenter is the intersection of the


angle bisectors, both AO and CO are bisectors.

C
A

By Property 3.1a,
m(AOC)= 90 +

m( B)
80
= 90 +
=130 .
2
2

O
80
B

EXAMPLE

57

In the figure, K is the intersection point of


the bisectors of the exterior angles at
vertices A and B with m(A) = 120
and m(B) = 40. Find m(BKA).

120
40
B

Triangles and Construction

125

Solution

(Triangle Angle-Sum Theorem in ABC)

m(A) + m(B) + m(C) = 180


120 + 40 + m(C) = 180
m(C) = 20

(1)

m( C )
m(BKA)= 90
2
20
= 90
2
= 80

EXAMPLE

58

(Property 3.1b)
(Substitute (1))

Find the value of x in the figure.

3x

x
E

Solution

m(C) + m(C) = 180


m(C) = 180 3x
m(AEB)=
x=

(Linear Pair Postulate)


(1)

m( C )
2

(Property 3.1c)

180 3 x
2

(Substitute (1))

5x = 180
x = 36

EXAMPLE

126

59

In the figure at the right,


AN is an angle bisector,
m(ANC) = 100 and
m(B) = 2m(C).
Find the value of x.

100
B

x
C

Geometriy 7

Solution

m(C) = x is given, so m(B) = 2x.


A

Let us draw the altitude AH BC.


Since ANC is an exterior angle of AHN,

10

m(HAN) + m(AHN) = m(ANC)


B

m(HAN) = 100 90 = 10
| m( B) m( C )|
2
| 2x x|
10 =
2
x
10 =
2
x = m(C) = 20.

m(HAN) =

EXAMPLE

60

Solution

100

2x

m(HAN) + 90 = 100

(Property 3.2)

One of the acute angles in a right triangle measures 20. Find the angle between the altitude
and the angle bisector which are drawn from the vertex of the right angle of the triangle.
Let us draw an appropriate figure. In the
figure at the right, A is the right angle, AN
is the angle bisector and
m(NAB) = m(NAC) = 45.
Let m(C) = 20, then m(B) = 70 and
m(HAB) = 20.

A
20

45
25

70
B

20

Therefore, m(HAN) = m(NAB) m(HAB) = 45 20 = 25. This is the required angle


measure.
Note that we can also solve this example by using Property 3.2. This is left as an exercise for
you.

EXAMPLE

61

In a triangle KLM, prove that if N is a point in the interior of KLM then


a. m(LNM) = m(LKM) + m(KLN) + m(KMN),
b. m(KNM) = m(KLM) + m(LKN) + m(LMN) and
c. m(KNL) = m(KML) + m(MLN) + m(MKN).

Triangles and Construction

127

Solution

Let us draw an appropriate figure.


Given: N is an interior point of KLM

Prove: m(1) = m(2) + m(3) + m()

3+4

Proof:

L
Let us extend segment MN through N and
label the intersection point T of ray MN and segment KL.

m(LTN) = m(3) + m(4)

(Triangle Exterior Angle Theorem)

m(1) = m(2) + m(3) + m(4)

(Triangle Exterior Angle Theorem)

This means m(LNM) = m(LKM) + m(KLN) + m(KMN).


The proofs of b. and c. are similar. They are left as an exercise for you.

EXAMPLE

62

Solution

ABC is an equilateral triangle and a point D int ABC such that AD DC and
m(DCA) = 42. Find m(BAD).
A

Let us draw an appropriate figure.


m(B) = 60

( ABC is equilateral)

m(BCD) = 60 42
= 18

D
60

(m(BCA) = 60)
B

42
C

m(ADC) = m(B) + m(BAD) + m(BCD) (Property 3.3)


90 = 60 + m(BAD) + 18
m(BAD) = 12
128

Geometriy 7

Check Yourself 15
1. Each figure shows a triangle with two or more angle bisectors. Find the indicated angle
measures in each case.
a.

b.

c.
P

50
S
x

S
x

40
Q

x=?

d.

70
x=?

e.

f.

70
Q

2x
R

x
T

x=?

K
3x+70

M
x y = ? (in terms of p)

x=?

y=?

2. In the triangle ABC at the right, AN is an angle bisector


and AH is an altitude. Given m(C) m(B) = 36, find
m(HAN).

3. A student draws the altitude and the angle bisector at the vertex of the right angle of a right
triangle. The angle between them is 18. Find the measure of the larger acute angle in the
right triangle.
4. Find the value of x in the figure.

4x

105

2x

Answers
1. a. 110
Triangles and Construction

b. 80

c. 35

d. 40

e. p

f. 80

2. 18

3. 63

4. 15
129

B. RELATIONS BETWEEN ANGLES AND SIDES


longer side opposite larger angle

Theorem

If one side of a triangle is longer than another side of the triangle then the measure of the angle
opposite the longer side is greater than the measure of the angle opposite the shorter side. In
other words, if two sides of a triangle have unequal lengths then the measures of the angles
opposite them are also unequal and the larger angle is opposite the longer side.
Proof

Given: ABC with AB > AC


Prove: m(C) > m(B)
K

3
1

2
C

We begin by locating K on AB such that AK = AC. We then draw CK and continue with a twocolumn proof.
Statements

EXAMPLE

63

Reasons

1. AB > AC

1. Given

2. AKC is isosceles

2. Definition of isosceles triangle (AK = AC)

3. 3 2

3. Base angles in an isosceles triangle are congruent.

4. m(ACB) = m(2) + m(1)

4. Angle Addition Postulate

5. m(ACB) > m(2)

5. Definition of inequality

6. m(ACB) > m(3)

6. Substitution property

7. m(3) > m(B)

7. Triangle Exterior Angle Theorem

8. m(ACB) > m(B)

8. Transitive property of inequality

Write the angles in each triangle in order of


their measures.

a.
A

Solution

b.

a. Since 7 > 5 > 3, m(A) > m(B) > m(C).


b. Since 5 = 5 > 4, m(E) = m(F) > m(D).

130

Geometriy 7

larger angle opposite longer side

Theorem

If two angles in a triangle have unequal measures then the sides opposite them have unequal
lengths and the longer side is opposite the larger angle.
Proof

Given: ABC with m(B) > m(C)


Prove: AC > AB

Trichotomy property
For any two real numbers
a and b, exactly one of
the following is true:
a < b, a = b, a > b.

We will give a proof by contradiction in


paragraph form.
According to the trichotomy property, exactly
one of three cases holds: AC < AB, AC = AB
or AC > AB.

Let us assume that either AC = AB or AC < AB and look for a contradiction.


If AC < AB then m(B) < m(C) by the previous theorem. Also, if AC = AB then
m(B) = m(C) by the definition of an isosceles triangle.
In both cases we have a contradiction of the fact that m(B) > m(C). That means that
our assumption AC AB must be false. By the trichotomy property, it follows that AC > AB.

EXAMPLE

64

Order the sides of triangle in the figure


according to their length.

A
(2x + 40)

Solution

m(A) + m(B) + m(C)


2x + 40 + 20 + 3x 10
5x
x

=
=
=
=

180
180
130
26

(3x 10)

20

So m(A) = (2 26) + 40 = 92 and m(C) = (3 26) 10 = 68.


Since m(A) > m(C) > m(B), by the previous theorem we have a > c > b.

EXAMPLE

65

In the figure at the right, KN = KM.


Prove that KT > KM.

K
4

1
T

Triangles and Construction

3
M

131

Solution

Given: KN = KM
Prove: KT > KM
Proof:
Statements

EXAMPLE

66

Solution

Reasons

1. KN = KM

1. Given

2. 2 3

2. Base angles of isosceles triangle KNM

3. m(2) = m(1) + m(4)

3. Triangle Exterior Angle Theorem

4. m(2) > m(1)

4. By 3

5. m(3) > m(1)

5. Substitute 2 into 4.

6. KT > KM

6. By the previous theorem

Prove that in any triangle ABC, a + b + c > ha + hb + hc, where ha, hb and hc are the altitudes
to the sides a, b and c, respectively.
A

Given: ABC with altitudes ha, hb and hc


Prove: (a + b + c) > (ha + hb + hc)

D
E

Proof:
B

Look at the figure. By the previous theorem,

in right triangle BCD, BC > BD, i.e. a > hb;

(1)

in right triangle AEC, AC > CE, i.e. b > hc;

(2)

in right triangle ABH, AB > AH, i.e. c > ha.

(3)

Adding inequalities (1), (2) and (3) gives (a + b + c) > (ha + hb + hc).
EXAMPLE

67

Find the longest line segment in the figure


using the given angle measures.

62

63
B

Solution

59

60

55

61
C

In ABC, m(B) > m(A) > m(C) so AC > BC > AB. (1) (By the previous theorem)
In ADC, m(C) > m(A) > m(D) so AD > CD > AC. (2) (By the previous theorem)
Combining (1) and (2) gives us AD > DC > AC > BC > AB. So AD is the longest segment in
the figure.

132

Geometriy 7

Check Yourself 16
1. Write the measures of the angles in each triangle in increasing order.
a.

b.

A
4

10
M

c.

2. Write the lengths of the sides of each triangle in increasing order.


a.

b.

80
60

40

c.

50

80

40

70

50

3. Find the longest line segment in each figure using the given angle measures.
a.

b.

A
60 62

65

60

c.

P
60

60

E
67 59 61

61

60
B

D
57 66

Answers
1. a. m(B) < m(A) < m(C) b. m(M) < m(P) < m(N) c. m(N) < m(K) < m(M)
2. a. c < b < a

b. n = m < k

c. k < s = t

3. a. CD

b. PK

Activity

c. BC

Triangle Inequality

For this activity you will need a piece of string and a ruler.
Cut the string into eight pieces of different lengths. Measure the lengths and label or
mark each piece with its length.
Take any three pieces of string and try to form a triangle with them.
Make a table to note the lengths of the pieces of string and whether or not they formed
a triangle.
Repeat the activity until you have two successes and two failures at making a triangle.
Look at your table. Which lengths of string together made a triangle? Which lengths
didnt make a triangle? What conjecture can you make about the sides of a triangle?
Triangles and Construction

133

Properties 4

Triangle Inequality Theorem


In any triangle ABC with sides a, b and c, the following inequalities are true:
|b c| < a < (b + c),
|a c| < b < (a + c),
|a b| < c < (a + b).
The converse is also true. This property is also called the Triangle Inequality Theorem.

EXAMPLE

68

Solution

Is it possible for a triangle to have sides with the lengths indicated?


a. 7, 8, 9

We can check each case by using the Triangle Inequality Theorem.


a. |9 8| < 7 < (8 + 9)

EXAMPLE

69

c. 1 , 1 , 1
2 3

b. 0.8, 0.3, 1

c. This is impossible,
since

b. |0.8 0.3| < 1 < (0.8 + 0.3)

|8 9| < 8 < (7 + 9)

|1 0.3| < 0.8 < (1 + 0.3)

|7 8| < 9 < (7 + 8).

|1 0.8| < 0.3 < (1 + 0.8).

This is true, so by the


Triangle
Inequality
Theorem this is a
possible triangle.

This is true, so by the


Triangle Inequality Theorem
this is a possible triangle.

1<

Find all the possible integer values of x in the


figure.

1 1
+ .
2 3

A
10

5
B

x
4

7
D

Solution

In ABC, |10 5| < x < (10 + 5)


5 < x < 15.
In DBC, |7 4| < x < (7 + 4)
3 < x < 11.

(Triangle Inequality Theorem)


(1)
(Triangle Inequality Theorem)
(2)

The possible values of x are the elements of the common solution of inequalities (1) and (2),
i.e. 5 < x < 11.
So x {6, 7, 8, 9, 10}.
134

Geometriy 7

EXAMPLE

70

Find the greatest possible integer value of m


in the figure, then find the smallest possible
integer value of n for this case.

A
9

m
B
6
D

Solution

In ABD, |9 6| < m < (9 + 6)

(Triangle Inequality Theorem)

3 < m < 15.


So the greatest possible integer value of m is 14.
In ADC, |8 m| < n < (m + 8)

(Triangle Inequality Theorem)

|8 14| < n < (14 + 8)

(m = 14)

6 < n < 22.


So when m = 14, the smallest possible integer value of n is 7.

Properties 5

1. In any triangle ABC,


a. if m(A) = 90 then

b2 + c2 = a < ( b + c).

b. if m(A) < 90 then |b c| < a <


c. if m(A) > 90 then

b2 + c2 .

b2 + c2 < a < (b + c).

2. In any triangle ABC, if P int ABC

then (BP + PC) < (BA + AC).


P

3. In any triangle ABC, if m(B) > m(C) or m(B) < m(C) then ha < nA < Va.
A

ha

Triangles and Construction

nA

Va

Va
N

nA

ha

135

EXAMPLE

71

Solution

In a triangle ABC, m(A) > 90, c = 6 and b = 8. Find all the possible integer lengths of a.
Since m(A) > 90,

b2 + c2 < a < (b + c) by Property 5.1.

Substituting the values in the question gives

82 +6 2 < a < (8 + 6), i.e.

10 < a < 14. So a {11, 12, 13}.

EXAMPLE

72

Solution

In the triangle ABC shown opposite,

P int ABC, AB = 10, AC = 8 and BC = 9.


Find the sum of all the possible integer values of PB + PC.

10

In PBC, BC < (BP + PC) by the Triangle


Inequality Theorem.
So 9 < BP + PC.

(1)

In ABC, (PB + PC) < (AB + AC) by Property 5.2.


So PB + PC < 10 + 8.

(2)

Combining (1) and (2) gives 9 < (PB + PC) < 18.
So the possible integer values for PB + PC are 10, 11, 12, 13, 14, 15, 16 and 17.
The required sum is therefore 10 + 11 + 12 + 13 + 14 + 15 + 16 + 17 = 108.

EXAMPLE

73

Solution

Prove that the sum of the lengths of the medians of a triangle is greater than half of the
perimeter and less than the perimeter.
Let us draw an appropriate figure.

Given: ABC with centroid G


Remember!
The centroid of a
triangle is the point of
intersection of its
medians.

136

Prove:

a+ b+ c
< ( Va + Vb + Vc ) < ( a + b + c).
2

Proof:
We need to prove two inequalities.

Geometriy 7

a+ b+ c
< Va + Vb + Vc : We will use the Triangle Inequality Theorem three times.
2
c
In CEB, (CE + EB) > BC, i.e. (Vc + ) > a.
(Triangle Inequality Theorem)
2

Proof that

a
) > b.
2

In ADC, (AD + DC) > AC, i.e. (Va +


In ABF, (BF + FA) > AB, i.e. (Vb +

b
) > c.
2

(Triangle Inequality Theorem)


(Triangle Inequality Theorem)

So (Vc + Va+ Vb +

c
a
b
+
+ ) > (a + b + c).
2
2
2

(Addition Property of Inequality)

So ( Va + Vb + Vc ) >

a+ b+ c
.
2

(Subtraction Property of Inequality)

(1)

Proof that Va + Vb + Vc < a + b + c:

For the second part of the inequality, let us


draw another figure as shown at the right

and extend the median AD through D to a


point K such that
AD = DK.

(2)

a
2

Then join K and B. Now,


BD = DC

Va

a
2

(AD is a median)

m(BDK) = m(ADC) (Vertical angles)


AD = DK.

Va

(By (2))
B

So by the SAS Congruence Postulate,


DBK DCA and so

a
2

|BK| = |CA| = b.
Then, in ABK,
2Va < b + c.

a
2

Va

(3) (Triangle Inequality


Theorem)

By considering the other medians in a similar way


we get 2Vb < (a + c) and 2Vc < (a + b).

(4)

Adding the inequalities from (3) and (4) side by side gives us
2(Va + Vb + Vc) < 2(a + b + c). So (Va + Vb + Vc) < (a + b + c).
Finally, by (1) and (5),
Triangles and Construction

(5)

a+ b+ c
< ( Va + Vb + Vc ) < ( a + b + c) as required.
2
137

Remark

The result we have just proved does not mean that for a given triangle, the sum of the
medians can be anything between the half perimeter and full perimeter of the triangle. This
is because the lengths of the medians are directly related to the lengths of the sides. As we
will see in the next chapter, once we know the lengths of the three sides of a triangle then
we can calculate the lengths of its medians. Their sum is a fixed number.

EXAMPLE

74

Two towns K and N are on the same side of the river Nile. The residents of the two towns
want to construct a water pumping station at a point A on the river. To minimize the cost of
constructing pipelines from A to K and N, they wish to locate A along the Nile so that the
distance AN + AK is as small as possible. Find the corresponding location for A and show that
any other location requires a path which is longer than the path through A.
l
K

l
A

A
X

figure 1

figure 2

figure 3

Solution Let M and N be as in figure 1, and let be a line representing the river. Then we can use the
following method to locate A:
1. Draw a ray from N perpendicular to , intersecting at point B.
2. Locate point C on the extension of NB such that NB = BC.
3. Draw KC.
4. Locate A at the intersection of KC and , as shown in figure 2.
Now we need to show that A is really the location which makes AN + AK as small as
possible. Figure 3 shows an alternative location X on l. Notice that in KXC, (KX + XC) > KC
by the Triangle Inequality Theorem. So (KX + XC) > (KA + AC) (1) by the Segment Addition
Postulate. Since AB NC and NB = BC, NXC is isosceles with XC = NX (2). By the same
reasoning, NAC is isosceles with NA = AC (3). Substituting (2) and (3) into (1) gives us
(KX + XN) > (KA + AN). So A is the best location for the station.
EXAMPLE

75

Prove that for any triangle ABC,


if P int ABC and x, y and z are as shown
in the figure then

(x + y + z) < (a + b + c) < 2(x + y + z), i.e.


a+ b+ c
< ( x + y + z) < ( a + b + c).
2
138

c
x
B

P
a

b
y
C

Geometriy 7

Solution

In ABP, c < (x + z).

(Triangle Inequality Theorem)

In APC, b < (y + z).

(Triangle Inequality Theorem)

In BPC, a < (x + z).

(Triangle Inequality Theorem)

So (a + b + c) < 2(x + y + z). (1)

(Addition property of inequality)

Also, (x + y) < (c + b),

(Property 5.2)

(y + z) < (a + c) and

(Property 5.2)

(x + z) < (b + a).

(Property 5.2)

So (x + y + z) < (a + b + c)

(2)

(Addition property of inequality)

As a result, (x + y + z) < (a + b + c) < 2(x + y + z), (By (1) and (2))


or equivalently,

a+ b+ c
< ( x + y + z) < ( a + b + c).
2

Remark

The example that we have just seen shows an application of triangle inequality. But the result
we obtained does not mean that the value of x + y + z can be any number less than
a + b + c. In other words, the maximum value of x + y + z may be a lot less than a + b + c.
In fact, the maximum value of x + y + z is always less than the sum of the lengths of the
two longer sides of the triangle, because as the interior point moves towards one of the
vertices, two distances increase but the third distance decreases. When this interior point
reaches the vertex point, the distance to this point becomes zero and the sum of the distances
becomes the sum of the two sides which include this vertex. So the maximum value of
x + y + z will always be less than the sum of the length of the two longer sides.

Check Yourself 17
1. Two sides of a triangle measure 24 cm and 11 cm respectively. Find the perimeter of the
triangle if its third side is equal to one of other two sides.
2. Determine whether each ratio could be the ratio of the lengths of the sides of a triangle.
a. 3 : 4 : 5

b. 4 : 3 : 1

c. 10 : 11 : 15

d. 0.2 : 0.3 : 0.6

3. The lengths of the sides DE and EF of a triangle DEF are 4.5 and 7.8. What is the greatest
possible integer length of DF?
4. The base of an isosceles triangle measures 10 cm and the perimeter of the triangle is an
integer length. What is the smallest possible length of the leg of this triangle?
5. In an isosceles triangle KLM, KL = LM = 7 and m(K) < 60. If the perimeter of the
triangle is an integer, how many possible triangle(s) KLM exist?
6. In a triangle ABC, AB = 9 and BC = 12. If m(B) < 90, find all the possible integer
lengths of AC.
Answers
1. 59 cm

2. a. yes b. no c. yes d. no

3. 12

4. 5.5 cm

5. six triangles

6. AC {4, 5, 6, 7, 8, 9, 10, 11, 12, 13, 14}


Triangles and Construction

139

EXERCISES

3 .3

A. Relations Between Angles

3. The two acute angles in a right triangle measure

1. Each figure below shows triangles with two or


more congruent sides. Find the value of x in each
figure, using the information given.
a.

b.

4. In a triangle ABC, the angle bisector of the interior

A
48

x
B

D
T

c.

D
E
m(BAC) = x

d.

MN intersect each other at a point P. Find


m(KPN) if m(KNM) = 72 and m(NKM) = 64.

9
2x
TM = TN

e.

f.

15

x
T

g.

x
N

P
M

h.

52

T
S
P

P
x

6. In the triangle MNP

opposite,
MS = MP,
26
ST = TP,
N
m(M) = 94 and
m(N) = 26. Find m(MST).

30

angle C makes an angle of 40 with the side AB.


Find the angle between the bisector of the exterior
angle C and the extension of the side AB.

5. In a triangle KMN, the altitudes to sides KM and

M
x

x
2x
+5) and ( 25) respectively. Find x and
4
3
the measures of these angles.
(

measure of the biggest angle in the triangle. The


measure of the third angle is half the measure of
the biggest angle. Find the measures of all three
angles.
140

94

?
T

7. In the triangle ABC at


the right,
BD = BE = BC and
segment EB bisects
B.
If m(ACD) = 18,
find m(ABC).

the right,
AB = AD = BE,
m(A) = 114 and
m(B) = 60. Find
m(EDC).

A
D
F

E
18

8. In the triangle ABC at


2. An angle in a triangle measures 20 less than the

A
D

114

60
B

Geometriy 7

9. In the figure,

14. x, y and z are the exterior angles of a triangle.

PQ = PS = PR and
m(SPR) = 24. Find
m(SQR).

Determine whether each ratio is a possible ratio


of x : y : z.

24

a. 2 : 3 : 5
?

10. In the figure opposite,


m(NMT) = 16,
m(TMP) = 44,
m(P) = 38 and
m(SNT) = 22.
Find m(TSN).

b. 1 : 2 : 3
c. 6 : 11 : 19
d. 12 : 15 : 21

M
16

15. Find the value of x in

44

145

38
N

T
22

a triangle DEF intersect at the point T. Find the


measure of DEF if its measure is one-third of
m(DTF).

12. a, b and b are the measures of the interior angles


of an isosceles triangle such that a and b are
integers and 24 < b < 38. Find the smallest
possible value of a.

16. In an isosceles triangle KMN, the bisectors of the


base angles K and M intersect each other at a
point T. Prove that m(KTM) = m(K).

B. Relations Between Angles and Sides


17. Each figure shows the lengths of two sides of a
triangle. Write an interval for the possible length
of the third side.
a.

Draw an appropriate figure for each of the


following, if it is possible.
a. FME is obtuse
b. FME is equilateral
c. DME is equilateral and DEF is isosceles
d. DME is isosceles and EMF is isosceles
e. DME is isosceles and DEF is equilateral

b.

A
8

13. In a triangle DEF, point M is on the side DF and


MDE and DEM are acute angles.

130

11. The bisectors of the interior angles D and F in

Triangles and Construction

140

the figure.

K
3

12

c.

M
4

18. For each figure, state the interval of possible


values for the length x.
a.
6
B

b.

A
x
5

10
7

4
M
D
7

c.

12

x
8

x
5

141

19. A triangle ABC has sides a, b and c with integer


lengths. How many triangles can be formed such
that b = c and a b = 18?

20. In the figure, a, b


and c are integers.
Calculate the smallest
possible value of
a + b + c, using the
information given.

F
6

m(A) > 90 and


10
6
m(C) > 90.
B
D
If AB = 6 cm,
AD = 10 cm,
5
12
BC = 12 cm and
C
CD = 5 cm, find
the sum of the all the possible integer lengths of
the side BD.

23. In the triangle ABC

12

26. In each case determine whether it is possible for


a triangle to have sides with the lengths given.
a. 13, 9, 5

b. 5, 5, 14

c. 8, 8, 16.1

d. 17, 11, 6

e. 0.5, 0.6, 1

f. 18, 18, 0.09

27. A triangle has side lengths 2x + y, 2y + 3x and 2x.


Which one is bigger: x or y?

28. For each figure, order the numbered angles


according to their size.
a.

b.

at the right,
AD = 9 cm,
x
y
BD = 6 cm,
9
DC = 8 cm,
B
C
AC = x cm and
8
6
D
AB = y cm.
Find the sum of the smallest and largest possible
integer values of x + y.
142

AB = 8 cm,
6
BC = 12 cm,
D
CD = 6 cm and
4
DA = 4 cm.
A
Find the number
of possible integer lengths of AC.

AB = 8 cm,
10
AC = 10 cm,
8
BD = 3 cm,
2x + 1
CD = 7 cm and
B
3
BC = 2x + 1 cm.
7
D
Find the sum of
all the possible integer values of x.

25. In the figure,

AC = 9 cm,
BC = a,
c
AB = c and
m(BAC) > 90.
B
Find the smallest
possible value of a + c if a, c .

21. In the figure,

22. In the figure,

24. In the figure,

20

18
1

n1
2

n
3

n+1

20

c.

d.

1
17

4.20

15
1

16
2

82
20

Geometriy 7

29. Determine whether each statement is true or


false.
a. In a triangle ABC, if the measure of A is 57
and the measure of B is 64 then the shortest
side of ABC is AB.
b. In a triangle KMN, if the measure of K is 43
and the measure of M is 47 then the shortest
side of KMN is KM.

31. A student has five sticks, each with an integer


length. He finds that he cannot form a triangle
using any three of these sticks. What is the shortest
possible length of the longest stick, if
a. the lengths of the sticks can be the same?
b. all the sticks have different lengths?
(Hint: Use the Triangle Inequality Theorem.)

c. In a triangle ABC, if B is an obtuse angle and


AH BC then HA < AB.
d. If an isosceles triangle KTA with base KA has
TA < KA then the measure of T is always less
than 90.

32. How many distinct isosceles triangles have


integer side lengths and perimeter 200 cm?

e. An angle bisector in an equilateral triangle is


shorter than any of the sides.
f. All obtuse triangles are isosceles.

33. How many triangles can be drawn if the length of

g. Some right triangles are equilateral.

the longest side must be 11 units and all side


lengths must be integer values?

34. In the figure,

30. State the longest line segment in each figure.


a.

b.
A

(x 8)

B
10
20

(x + 10) 110

c.

20

65

80

Triangles and Construction

5
8

?
12
9

D
A

d.

70 70

35. In a triangle ABC, AB = 8 cm, BC = x and AC = y.

60

AD = 5 cm,
AB = 12 cm,
BC = 9 cm and
DC = 8 cm.
If m(A) > 90 and
m(C) < 90, find
all possible integer
values of BD.

10

If m(A) > 90 and x, y , find the


smallest possible value of x + y.
143

36. How many distinct triangles have integer side

38.

lengths and perimeter 11?

130

y
M

37. Prove each theorem.


a. Hinge Theorem:
If two sides of one triangle are congruent to
two sides of another triangle, and if the included
angle of the first triangle is larger than the
included angle of the second, then the third
side of the first triangle is longer than the third
side of the second.

a. 130, x

b. y, 90

c. y, x

d. KM, MN

39.

A
y
M
30

b. Converse of the Hinge Theorem:


If two sides a and b of one triangle are
congruent to two sides d and e of another
triangle, and if the third side of the first
triangle is longer than the third side of the
second, then the angle between a and b is
larger than the angle between d and e.

a. (x 10), (y + 20)
b. MB + MC, AB + AC

40. In a triangle DTF, m(D) = 90 and m(F) < 45.


a. 2 m(T), m(D)
b. FT, 2DF
c. FD, DT

Instructions for questions 38 to 42


Each question gives two quantities to be compared,
separated by a comma. In each case, use the figure or
extra information to compare the quantities. Write

d. m(F), 2m(T)

41. In a triangle DEF, m(D) > m(E) = m(F).

A if the first quantity is greater than the second,

a. m(D), 60

B if the first quantity is smaller than the second,

b. m(E), 60

C if the quantities are equal, or


D if the extra information is not enough for you to
be able to compare the quantities.
All variables represent real numbers. Figures are
generally not drawn to scale.
144

42. In a triangle DEF, m(E) = 120 and EF > DE.


a. 120, 3 m(D)
b. 2 m(E), 3 m(D)
Geometriy 7

Objectives

After studying this section you will be able to:


1. Find the distance from a point to a line.
2. Find the distance between two parallel lines.

A. DISTANCE FROM A POINT TO A LINE


Theorem

Proof

distance from a point to a line


Let A(x1, y1) be a point and d: ax + by + c = 0 be a line, then the distance from A to the
line d is
| ax1 + by1 + c |
.
l=
a2 + b 2
Let the distance of A(x1, y1) to the line

A(x1, y1)

d: ax + by + c = 0 be l = AH.

Take C(x2, y2) = AD d. x2 = x1 and y2 = CD


C is a point on the line ax + by + c = 0, so
a
O

So we have the coordinates of C, C( x1 ,

C(x1, |CD|)

ax1 + b CD + c = 0
b CD = a x1 c
a
c
CD x1 .
b
b

a
c
x1 ).
b
b

Now, is the inclination of d and = m(CBD) = m(CAH) (angles with perpendicular sides).
In the right triangle ACH, cos =

AH
AC

and AH = AC cos ...(1)

Now, lets find the equivalent expressions for AC and cos.


a
c
a
c
AC = AD CD = y1 ( x1 ), so AC = y1 x1 ...(2)
b
b
b
b
We know sec2 = 1 + tan2
a
1
so cos =
, and tan = m
2
b
1+ tan
so cos =

Triangles and Construction

1
a
1+( )2
b

1
a
1 ( )2
b

...(3)

145

Substituting (2) and (3) in (1),


l = AH = AC cos , and since l is the distance,
a
c
l = ( y1 x1 )
b
b

EXAMPLE

76

Solution

77

Solution

78

Solution

a2
b2

O(0, 0) = O(x1, y1). Using the formula,


| ax1 + by1 + c |
2

a +b

|0 0+ 4|
2

1 +( 1)

4
4 2
=
= 2 2.
2
2

Find the distance from A(5, 2) to the line 3x 4y + 5 = 0.


A A(x1, y1). Using the formula,
l=

EXAMPLE

1+

Find the distance from the point O(0, 0) to the line x y + 4 = 0.

l=

EXAMPLE

a
c
y1 + x1 +
b
b = ax1 + by1 + c .
=
1 2
a2 + b 2
a + b2
b

| ax1 + by1 + c |
2

a +b

| 3 5 4 2+5|
2

3 +4

12
= 2.4.
5

The distance from A(12, 5) to the line 5x 12y + 5k = 0 is ten units. Find the possible values
of k.
l=
l=

| ax1 + by1 + c |
2

a +b

|5 12 12 5+5 k|
25+144

=10

|5k |
=10, so |5 k|=130, i.e. 5 k= 130, and so k= 26.
13

Check Yourself 18
1. Find the distance from the point P(2, 3) to the line 3x + 4y + 9 = 0.
2. Find the distance from the point A(1, 4) to the line y = 3x 4.
3. The distance between the point P(k, 3) and the line 4x 3y + 5 is 4 units. Find k.
Answers
1. 3
146

2.

10
2

3. k {4, 6}
Geometriy 7

B. DISTANCE BETWEEN TWO PARALLEL LINES


Let d1: a1x + b1y + c1 = 0
d2: a2x + b2y + c2 = 0 be two parallel lines.
a
b
Since d1 d2, we can write 1 1 k , so a1 = k a2 and b1 = k b2.
a2 b2
Now, lets substitute these values into d1:
k a2x + k b2y + c1 = 0
c
k(a2x + b2y + 1 ) = 0.
k

c1
= 0.
k
When we compare d1 with d2, we see that their difference is a constant number.

k 0, so we get d1: a2x + b2y +

In general, we can write two parallel lines d1 and d2 as:


d1: ax + by + c1 = 0
d2: ax + by + c2 = 0.
Proof

The distance of any point A(x, y) on line d1 to the line d2 is


| ax + by + c2 |
.
a2 + b 2
In the equation of d1: ax + by + c = 0

ax + by = c1, and so l =

| c2 c1 |
2

a +b

A(x, y)
l

d1

d2

Remark

It is important to notice that to find the distance between two parallel lines, first of all we
need to equalize the coefficients of x and y.
EXAMPLE

79

Find the distance between the parallel lines x 2y + 5 = 0 and 3x 6y + 9 = 0.


d1: x 2y +5 = 0
3x 6 y +15 = 0

d1 was multiplied by 3.
d2 : 3x 6 y +9 = 0
3x 6 y +9 = 0

Solution

Now, we have c1 =15


| c1 c2 |
15 9
6
6
2 5
=
=
=
=
.
l=
2
2
5
9+ 36
45 3 5
a b
c2 = 9
Theorem

distance between two parallel lines


Let d1: ax+ by + c1 = 0 and d2: ax + by + c2 = 0 be two parallel lines. Then the distance
|c c |
between d1 and d2 is
.
l= 2 1
a2 + b 2

Triangles and Construction

147

EXAMPLE

80

Solution

Find the distance between the parallel lines 3x 2y + 5 = 0 and 3x + 2y + 8 = 0.


d1: 3x 2y +5 = 0

d1: 3x 2 y +5 = 0

d2 : 3x + 2 y +8 = 0
d2: 3 x 2 y 8 = 0
So c1 = 5
| c1 c2 |
|5+8|
13
=
=
= 13.
l=
2
2
2
2
13
a b
3 +(2)
c2 = 8

There is also another way to solve the problem:


The distance between d1 and d2 is the same as the distance of any point on d1 or d2 to the
other line.
For example, A(0, 4) is one point on d2, and the distance of A to d1 is
l=

| 3 x1 2 y1 +5|
2

3 +(2)

| 3 0 2( 4)+5|
13

13
13

13. The solution is the same.

Check Yourself 19
1. Find the distance between the lines 4x 3y 5 = 0 and 12x + 9y + 4 = 0.
2. The lines x + 2y + 1 = 0 and 3x + 6y + k = 0 are parallel and the distance between
them is 5. Find k.
3. Find the area of the square whose two sides are on the parallel lines 2x + y 2 = 0 and
4x + 2y + 6 = 0.
Answers
11
1.
2. k {12, 18}
15

148

3. 5

Geometriy 7

EXERCISES

3.4

A. Distance from a Point to a Line


1. Find the distance from the point A(2, 3) to the
line 8x + 6y 15 = 0.

2. The distance between B(2, 3) and the line


12y 5x = k is

5
. Find k.
13

8. The

distance

between

the

parallel

lines

12x + 9y 2 = 0 and ax + 3y + c = 0 is three


a
units. Find the ratio , if c 0.
c
9. Write the equations of the lines which are four

units away from the line 3x + 4y + 10 = 0.

10 . The

distance

between

the

parallel

lines

3x + 4y 6 = 0 and 4x ky + 4 = 0 is p. Find

3. The distance from a line with equation


y 4 = m(x + 2) to the origin is 2. Find m.

k + p.

4. The distance between P(1, 2) and the line


7x y + k = 0 is 42 units. Find k.

5. The points A(1, 3), B(2, 1) and C(3, 1) are the


vertices of the triangle ABC. Find the length of
the altitude of BC.

1
, k) to the line
2
12x + 9y 1 = 0 is 2 units. Find k.

6. The distance from P(

B. Distance Between Two Parallel


Lines
7. Find the distance between each pair of parallel
lines.
a. 2x + 3y 4 = 0 and 2x + 3y 17 = 0
b. x y 4 = 0 and 2x + 2y 7 = 0
c. y = 2x + 1 and 2y = 4x 3
Triangles and Construction

149

CHAPTER

REVIEW TEST A

1. In the triangle ABC in

5. Two sides of a triangle have lengths 8 and 12.

4x
the figure,
m(A) = 4x,
x
B
m(B) = x and
m(C) = 30. Find the value of x.

30

What is the sum of the minimum and maximum


possible integer values of the length of the third
side?

A) 10

E) 30

B) 15

C) 20

A) 24

D) 25

B) 22

C) 19

6. Which is the longest


2. In a triangle MNP, the interior angle bisectors of
M and P intersect at the point S. Given that
N measures 40, find m(PSM).
A) 95

B) 100

C) 105

D) 110

side in the figure,


according to the given
angle measures?

E) 120

B) 105

?
T

C) 100

D) 95

60

C) BD

D) CD

E) BE

C) 254

C) m(E) < m(D)

D) m(E) = 60

E) m(E) = m(D)

the smallest and


largest possible integer
values of x in the
figure?

x
S

B) m(E) > m(F)

8. What is the sum of

45

25

A) DE < (DF EF)

E) 90

36

D) 248

E) 244

10
x

Chapter 3 Review Test A

60

Which statement is true?

m(P) = 45,
m(N) = 36 and
m(R) = 25. Find
the value of x.

B) 256

61

7. In a triangle DEF, DE = EF and DF > EF.

4. In the figure,

A) 260

62

opposite, N TK and
SN is the interior angle
bisector of S.
If m(T) m(K) = 40,
find m(SNK).
A) 110

B) AB

E) 16

A) BC

3. In the triangle STK

D) 18

10

C
14

A) 26

B) 24

C) 22

D) 20

E) 17
Angles

9. In a triangle ABC, D BC and AD bisects A. If

13. a, b and c are the lengths of the sides of a triangle

AB = 6 cm, BD = 3 cm and DC = 2 cm, find the


length of AD.

ABC. Given that a, b and c are integers and


a2 b2 =17, what is the sum of the minimum and
maximum possible values of c?

B) 43 cm

C) 32 cm

A) 7

E) 3 cm

D) 23 cm

B) 13

14. In the figure,


10. In the figure,

BD = DC,
AD = AE and
m(C) = 20. Find
m(EDC).
A) 70

B) 65

E
?
B

C) 60

20

D) 45

E) 30

D) 18

B) 8

60
D
2x + 3

C) 10

D) 12

15. In the figure,


11. If MNP STK, which of the following
statements is false?
A) MN ST

D) PNM STK

Chapter Review Test 1A

m(BAD) = 12,

?
8

B) 54

C) 58

D) 60

E) 64

ND = DP and
HD
3
.
=
MH
3

D
20

C) 120

16. In the figure,

80

AD = 4 cm. What is m(ABC)?


A) 52

BD bisects B,
BD = BE and
DE = EC.
If m(A) = 80 and,
m(ACD) = 20, what
is m(BDC)?
B) 110

C) NP TK
E) KT PN

12. In the figure,

A) 100

m(BAC) = 90,

E) 14

12

BC = 8 cm and

B) MP TK

E) 23

m(BAC) = 90,
m(C) = 60 and
BD = DC.
Find BC if
AD = 2x + 3 and
AC = 6x 1.
A) 6

C) 17

6x 1

A) 52 cm

D) 140

E) 150

MH
What is
?
NP

A)

3
2

B)

3
3

C)

3
4

D)

3
5

E)

3
6
151

CHAPTER

REVIEW TEST B

1. In the figure,
AB = AD,
AC = BC and
m(DAC) = 15. Find
m(C).

5. In a triangle ABC, points D and E are the

15

midpoints of the sides AB and AC respectively.


DE = (x + 5)/4 and BC = 8x 5 are given. What is
the value of x?

?
D

A) 1

B) 2

C) 3

D) 4

E) 5

A) 40

B) 45

C) 50

D) 60

E) 65

6. ABC is a right triangle with m(A) = 90, and AH


2. In the triangle MNP

A) 4 cm

in the figure,
MS = NS and
KN = KP.
If m(MRP) = 117,
what is m(MNP)?
A) 39

B) 41

is the altitude to the hypotenuse. If m(C) = 30


and BH = 2 cm, find HC.

C) 6 cm

D) 7 cm

117

B) 5 cm
E) 8 cm

?
N

C) 43

D) 45

E) 47

7. In the figure,

AB = AD.
DE
What is
?
EC

E
B

3. In a triangle ABC, D is a point on the side AB and CD


is the interior angle bisector of C. If AB = 15 cm
and 3 AC = 2 BC, find the length of DB.

A)

1
2

B) 1

C)

2
3

D)

3
2

E)

4
3

A) 2 cm B) 3 cm C) 4 cm D) 6 cm E) 9 cm

8. In the figure,
4. In the figure,
MN = MP and
ML = MK.
If m(PLK) = 12,
what is m(LMN)?
A) 18

B) 20

Chapter 3 Review Test B

P
12
K

C) 24

D) 30

E) 36

MS = SN and
MP = PN.
If m(P) = 20,
m(KMP) = 40 and
m(KNP) = 30,
what is m(SKN)?
A) 50

B) 45

M
40
S
?

30

20

C) 40

D) 35

E) 30
Angles

9. In the figure,

13. In the triangle MNP

HK = KN,
m(DAC) = 40 and
m(HKB) = 20.
Find m(BKD).

40
D

?
20

A) 20

B) 30

C) 40

D) 60

bisects angle B. Given


12
m(ADB) = 90,
DE BC,
B
AB = 12 and
BC = 16, find the length of DE.
B)

E) 70

shown opposite, point


O is the center of the
inscribed circle of
K
MNP.
6
If KS NP,
N
KN = 6 and
SP = 8, what is the length of KS?
A) 10

B) 12

C) 14

14. In the figure,

10. In the figure, BD

A) 1

3
2

C) 2

16

D)

5
2

E) 3

8
P

D) 16

E) 18

CH = HB,
AD = 3 and
DB = 8. What is the
sum of the all
possible integer values
of the length AC?
A) 30

B) 34

3 D

C) 40

D) 42

E) 51

15. In the figure,


11. In the figure,

MK = KL,
MN = m,
KN = 2m and
NL = 3m. Find
m(KNL).
A) 45

B) 50

ACDE is a square,
m(ABC) = 60 and
BD = 2 cm. Find the
length of one side of
the square.

N
?
2m

3m

C) 60

D) 70

E) 75

60
B

D
2

A) (3 3) cm B) (3 1) cm C) (3 + 1) cm
D) (4 23) cm

E) (25 3) cm

16. In the figure,

integers a, b and c such that b = c and


(a + b + c) (a + b c ) = 15.
Find the value of a.
A) 1

B) 2

Chapter Review Test 1A

C) 3

D) 5

AE = BD = DC and
AB = AC.
What is m(FDC)?

12. The lengths of the sides of a triangle ABC are

A
F
?
B

E) 7

A) 45

B) 50

C) 60

D) 62.5

E) 67.5
153

CHAPTER

REVIEW TEST C

1. In the figure,

5. In the figure, ABC,

m(KBC) = m(KCA).
and m(LKB) = 80.
What is the measure of
ACB?

80
B

A) 40

B) 60

C) 70

D) 75

E) 80

CDE, and FEG are


equilateral triangles.
If BG = 16, what is
the sum of the
perimeters of the
three triangles?
A) 32

2. Which is the largest

angle in the figure,


according to the given
lengths?

8
222

12
9

6
M

B) 36

C) 42

D) 46

6. In the triangle ABC in

N
8

E) 48

the figure, CD is the


D
F
bisector of C, AE is
50
the median to BC and
B
E
DE AC.
If m(B) = 50, what is m(BAC)?

A) M

B) N

C) S

D) SPK

E) K

A) 30

B) 35

C) 40

7. Which of the

m(X) < m(Y). What is the largest possible


integer measure of the angle Y?

following is a possible
sum of the lengths of
AB and BC in the
figure?

A) 59

A) 11

3. In an isosceles triangle XYZ, m(Y) = m(Z) and

B) 60

C) 89

D) 90

E) 110

B) 12

D) 45

10

13

E
7

C) 13

E) 50

C
6

12

D) 14

E) 37

8. In the figure,
4. In a triangle ABC, points B, C and D are collinear
and AD is the angle bisector of the exterior angle
A. If AC = BC, DB = 12 and AB = 4, find the
length of BC.
A) 2

B) 3

Chapter 3 Review Test C

C) 4

D) 5

E) 8

MP = PS = SN = PT
and ST = TN.
What is m(NMP)?

A) 36

B) 60

C) 72

D) 84

E) 108
Angles

9. In the figure, ABC is

13. In the figure,

an equilateral triangle.
If BD = AE, what is
the measure of EFC?

AF = FB and
AE = EC.
If EH + FH = 12,
what is AB + AC?

E
D F ?
B

A) 45

B) 60

C) 75

D) 90

10. In the figure,

B) 80

B) 18

C
E

12

D) 72

11. In the figure,

D) 18 cm

E) 42

E) 36

B) 7

C) 8

D) 10

E) 12

MK = NK = PK.
What is x + y + z?

x
K
y

5 C

A) 270

B) 180

C) 90

D) 60

E) 45

C) 16 cm
E) 20 cm

16.
12. In the figure, ABC is
an equilateral triangle.
If PB = 16 and
PN = 10, what is the
length of AH?

N
10
P
16

C) 43

A
150

K B

Chapter Review Test 1A

D) 24

15. In the figure,

B) 15 cm

B) 33

C) 22

O is the incenter of
ABC, AB OT and
AC OV.
If BT = 6 cm,
TV = 7 cm and
B 6 T
VC = 5 cm, what is
the perimeter of the triangle OTV?

A) 23

the figure, BH is the


?
exterior angle bisector
of B and
B
AE = EC.
If m(BHC) = 90,
H
BC = 8 and
EH = 7, what is the length of AB?
A) 6

A) 12 cm

14. In the triangle ABC in

C) 78

E) 120

18

A) 82

A) 16

AC = BC and
AB = AD.
If m(CAD) = 18
and m(EBD) = 12,
what is m(AEB)?

D) 53

E) 63

In the figure, m(M) = 90, m(MST) = 150


and PM = MS = ST. What is m(N)?
A) 5

B) 10

C) 15

D) 22.5

E) 30
155

CHAPTER

REVIEW TEST D

1. In the triangle ABC in

5. In the triangle MNP

the figure, BN is the


bisector of ABC and
N
110
H
H is the intersection
point of the altitudes
20
B
of ABC.
If m(AHC) = 110 and m(HBN) = 20,
what is m(BAC)?
A) 50

B) 55

C) 65

D) 75

2. In the figure,

A) 50

B) 55

C) 60

B) 60

B) 35

Chapter 3 Review Test D

C) 60

6. According to the

D) 55

A) 5
T

E) 70

B) 2

1
2

C) 2

E) 50

A
n

(2 m + n ) ?
m

12

1
2

D) 1

E)

1
2

7. The measure of one angle in a triangle is equal to


the sum of the measures of the other two angles.
Which statement about this triangle is always
true?
A) The triangle is equilateral.

B) The triangle is acute.


C) The triangle is a right triangle.

4. In the figure,

A) 30

?
S

figure, what is
the value of

C) 44

D) 30

E) 29

110

D) The triangle is obtuse.


E) The triangle is isosceles.

8. In the triangle XYZ in

AD = BD,
m(DAC) = x and
m(BCE) = 2x.
If m(EAB) = 110,
what is the value of x?

B) 65

50

A) 70

AB = AC = b,
BC = a, and a < b.
What is the largest
possible integer value
of m(A)?

E) 80

D) 65

3. In the figure,

A) 59

opposite, MK = TK,
NS = TS and
m(KTS) = 50.
What is m(MPN)?

MN = MP, KP = KT,
M
m(NMP) = m,
m
m(PKT) = k, and
?
points N, P and T are
N
P
collinear.
If m + k = 130, what is m(MPK)?

A
x
2x

C) 40

D) 45

E) 50

the figure,
m(YZX) = 90,
XZ = PK and
XP = PY.
What is m(PKZ)?
A) 120

B) 135

X
P
?
Y

C) 140

D) 150

E) 160
Angles

9. In the figure, PM is the


angle bisector of NPK,
MN = MP, NS = SP
and m(MKP) = 90.
What is m(STP)?
A) 90

B) 85

13. In the figure,

M
T
?
N

C) 80

D) 75

10. In the figure,

E) 60

D
?

A) 6

B) 8

C) 43

A) 50

B) 55

C) 60

D) 13

A) 55

B) 60

A) 76

B) 72

48
?

36
E

C) 68

K 2x
x
N

C) 65

D) 70

E) 75

A
75
30

?
D

A) 5

B) 10

C) 15

D) 20

E) 25

16. In the figure,

E
D

Chapter Review Test 1A

E) 52

triangle ABC in the


figure, AF = FC and
AH = BD. What is the
measure of EDC?
B) 10

D) 58

12. In the equilateral

A) 5

E) 80

70

AD and CB bisect
angles A and C,
respectively.
If m(AEC) = 75 and
m(B) = 30,
what is m(ADC)?

E) 63

AB = BC,
DE = BE
m(ABD) = 36 and
m(EDC) = 48.
What is m(ACB)?

D) 70

15. In the figure,


11. In the figure,

130

KS = KN,
m(M) = 70,
m(P) = x and
m(MKS) = 2x.
What is the value of x?

63
B

m(DAC) = m(B)
and
?
m(EAB) = m(C).
B
D
If m(AEC) = 130, what is m(ADE)?

14. In the figure,

ABC is an equilateral
triangle and BD = CE.
If AD = 63, what is
the length of DE?

C) 15

D) 20

E) 30

KL = LM and
LH = MH.
If NH = 5 and
m(K) = 30,
what is KM?

N
5
30
K

A) 15

B) 20

C) 25

D) 30

E) 40
157

CHAPTER

REVIEW TEST E

1. In the figure,
DE = DC and
DB = BF.
If m(A) = 45,
what is m(ABC)?
A) 30

5. In the figure, points K,

A
45

F
D

B) 45

C) 50

D) 60

2. In the triangle ABC at

E) 75

A) 6

A
x+13

the right,
AB = AC,
m(A) = x + 13 and
m(B) = y 38.

B) 243

y38

C) 240

D) 233

E) 204

3. According to the
130
105
140

B) 15

C) 20

D) 25

E) 30

m(BAC) = 90,
m(C) = 15 and
BC = 24. What is
the length of AH?

4. In the figure,

A) 2

B) 3

Chapter 3 Review Test E

D) 10

12

E) 12

?
6

4
R

C) 6

D) 7

E) 8

A
?
B

15

B) 5

C) 6

8. According to the

24

A) 4

ABC is an equilateral
triangle and
AD = EC = CF.
If BC = 12,
what is the length of
CF?

B) 5

and KM bisects RKL.


If KL = 6,
KR = 4 and
MS = 8, what is the
length of PK?
A) 4

C) 9

7. In the figure,

figure, what is the


value of x?

A) 10

B) 8

6. In the figure, KL RS

C
What is the sum of the B
minimum integer value of y and the maximum
integer value of x?

A) 248

S, T, M, N and P are
the midpoints of the
sides on which they lie.
If AB = 12,
AC = 8 and
BC = 16, what is
P(MNP)?

figure, what is the


smallest possible value
of a + b + c if a, b
and c are integers?

D) 8

D
5

E) 12

5
C

6
E

C) 4

D) 5

E) 6

A) 7

B) 8

C) 9

D) 10

E) 11
Angles

9. In the figure,
AH = BH = HC.
If AC = 1,
what is HD?

D
?
B

A)

1
2

13. In the figure,

B) 1
3

C)

1
4

D)

1
5

10. In the triangle ABC in

B) 30

3
5

D) 45

C) 70

B) 40

?
E
A

C) 45

D) 50

A) 1

6+33
15

B) 2

C) 3

D) 4

D) 75

E) 80

ABCD is a square and


ABF and BEC are
equilateral triangles.
What is m(FEC)?
A) 5

B) 10

Chapter Review Test 1A

C
F

C) 15

D) 20

E) 22.5

E) 5

M
K
?
9

A) 2 cm B) 3 cm C) 4 cm D) 5 cm E) 6 cm

16.In the figure, CD is the


12. In the figure,

MK = KP,
m(M) = 90,
NS = 9 cm and
SP = 3 cm. Find the
length of KS.

E) 60

15. In the figure,

B) 67.5

E) 50

an equilateral triangle
and DEFH is a square.
Find the measure of
AKD.

A) 30

m(A) = 90,
m(B) = 15 and
AB = 6 + 33.
What is the length
of AC?

140

C) 40

BD = DC,
CE = 3AE and
2AB = AC.
If m(A) = 90, what
is m(DEC)?

14. In the figure,

11. In the figure, ABC is

A) 65

E)

the figure,
CD AB and
BE AC.
If m(BFC) = 140,
what is m(A)?
A) 20

bisector of C.
If m(BAE) = 15,
m(EAC) = 60 and
m(B) = 45, what is
m(DEA)?
A) 10

B) 15

15
D

60

?
45
B

C) 20

D) 22.5

E) 30
159

160

Angles

Objectives

After studying this section you will be able to:


1. Define the concept of a circle and its basic elements.
2. Describe and use the properties of chords.
3. Describe and use the properties of tangents.
4. Describe the possible relative positions of two circles in the same plane.

A. BASIC CONCEPTS
1. Definition
You can see many circular or ring-shaped geometric figures all around you. For example,
wheels, gears, compact discs, clocks, and windmills are all basic examples of circles in the
world around us.

wheels

compact disc

gears

It is easy to recognize a circle, but how can we define it as a shape? Let us look at a
geometric definition.

Note
The word circle is derived from the latin word circus, which means ring or racecourse.
Definition

circle
A circle is the set of all the points in a plane that are at the same distance from a fixed point
in the plane. The distance is called the radius of the circle (plural radii), and the fixed point
is called the center of the circle.
All radii of a circle are congruent. A circle is named by its
center. For example, the circle on the left is named
circle O.
We write a circle with center O and with radius r as
or C(O, r).

center
O
circle

radius

In this book, the point O in a circle is always the center


of the circle.
162

Geometriy 7

To construct a circle, fix a pin on a piece of paper, connect a string of any length
to the pin, tie the other end of the string to your pencil, and turn your pencil
on the paper around the pin for one
complete revolution, keeping the string
taut. You will get a circle.
You can also use a compass to draw a circle.
Mark a point O as the center and set your compass to the
length of the radius. Turn your compass around the
center for one complete revolution. You will get a circle.

r
O

2. Regions Separated by a Circle in a Plane


A circle divides a plane into three separate regions. The set of points whose distance from the center of a circle is less than the radius of the circle is called the
interior of the circle.
interior region

circle

exterior region

For example, if R is a point in the plane and


|OR| < r, then R is in the interior of the
circle.
The set of points whose distance from the
center is greater than the radius of the
circle is called the exterior of the circle.

interior

Q
P

O
R
exterior

circle

For example, if Q is a point in the plane and |OQ| > r, then the point Q is in the exterior of
the circle. The set of points whose distance from the center is equal to the radius is called
the circle itself, and the points are on the circle. For example, if P is a point in the plane and
|OP| = r, the point P is on the circle.

Note
The union of a circle and its interior is called a circular closed region or a disc.

EXAMPLE

Name the points in the figure which are


b. on the circle.
c. in the exterior of the circle.

Solution

a. Since |OA| < r and |OB| < r, points A and B are in


the interior of the circle.

a. in the interior of the circle.

A
D

C
F

b. Since |OC| = |OD| = r, points C and D are on the circle.


c. Since |OE| > r, |OF| > r and |OG| > r, points E, F, and G are in the exterior of the circle.
Circles

163

3. Auxiliary Elements of a Circle


chord

Definition

A line segment which joins two different points on a


circle is called a chord.

B
A
C

chord
O
diameter

For example, [AB] and [CD] in the figure are chords.


diameter

Definition

A chord which passes through the center of a circle is called a diameter of the circle.
In the figure, chord [CD] passes through the center of the circle, so [CD] is a diameter.
We can see that the length of every diameter in a given circle is the same. For this reason,
we usually talk about the diameter of a circle to mean the length of any diameter in the
circle.
The length of the diameter of a circle is twice the radius. For example, if r is the radius of a
d
circle and d is the diameter, then d = 2 r, or r = .
2
The diameter of a circle is the longest chord in the circle.

EXAMPLE

Solution

1. Find the length of the diameter for each


given radius.
a. 3 1 cm
b. 3x cm
2
c. 2x + 5 cm d. 7x 12 cm
1. a. d = 2 r d = 2 3
b. d = 2 (3x) = 6x cm

2. The length of the diameter of a circle is


20 cm and the radius is 2x 4. Find x.

1
= 7 cm
2

c. d = 2 (2x + 5) = 4x + 10 cm
d. d = 2 (7x 12) = 14x 24 cm
2.

164

d=2r
20 = 2 (2x 4)
2x 4 = 10
2x = 14
x = 7 cm
Geometriy 7

4. Relative Position of a Line and a Circle in the Same


Plane
A line and a circle in the same plane can have one of
three different positions relative to each other.

If the distance from the center of the circle to the line


is greater than the radius of the circle, then the line
does not intersect the circle.

In the figure, [OH] l and |OH| > r,


and l C(O, r) = .

tangent

If the distance from the center of the circle to the line


is equal to the radius, then we say that the line is
tangent to the circle. In the figure, |OH| l,
|OH| = r, and l C(O, r) = {H}. H is the only point of
intersection of the line and the circle.

point of
tangency

tangent

Definition

A line which intersects a circle at exactly one point is called a tangent of the circle. The intersection point is called the point of tangency.
If the distance from the center of the circle to the line
is less than the radius, then the line intersects the
circle at two points.

In the figure, [OH] and |OH| < r,


and C(O, r) = {A, B}.
secant

Definition

A line which intersects a circle at two different points is called a secant of the circle.
For example, line is a secant in the figure on the left.
EXAMPLE

Solution

Circles

Name all the radii, diameters, chords, secants, and


tangents of the circle in the figure.
[OF], [OC], and [OB] are radii.
[FC] is a diameter. l is a secant line.
[EF], [ED], and [FC] are chords. GH is a
tangent, and A is a point of tangency.

E
D

G
A

165

Check Yourself 1
1. Define the terms center, radius, chord, diameter, tangent, and secant. Show them in a
figure.

2. How many regions does a circle divide the plane into?

3. Sketch all the possible relative positions of a circle and a line in the same plane.

4. Look at the figure on the right.


a.

Name the tangents.

b.

Name the secants.

c.

Name the chords.

d.

Name the radii.

e.

Name the diameters.

E
B
A
F

O
D

Answers
1. center: a point inside the circle that is equidistant from all the

tangent

points on the circle.


radius: a distance from the center to a point on the circle.
chord: a line segment joining two different points of a circle.

secant

us

di
ra

diameter: a chord passing through the center of a circle

chord
diameter

tangent: a line intersecting a circle at exactly one point.


secant: a line intersecting a circle at two different points.
2. three parts: the interior of the circle, the circle, and the exterior of the circle.
n

3.

m B
C
l

4. a. EF, EB
166

b. BC, DB

c. [AB], [DB], [BC]

d. [OD], [OA], [OB], [OC]

e. [BD]
Geometriy 7

B. CHORDS
Remember that a chord is a line segment which joins two different points on a circle. In this
section we will look at the properties of chords.
Property

A radius that is perpendicular to a chord bisects the chord.

EXAMPLE

Solution

A chord of length 10 cm is 12 cm away from the center


of a circle. Find the length of the radius.

For example, in the figure, if [OH] [AB] then


|AH| = |HB|.

A
r

Look at the figure.

12

| AB|
2
10
5 cm
2
In AHO, r2 = 52 + 122

r2 = 25 + 144
r2 = 169
r = 13 cm.

Property

In the same circle or in congruent circles, two chords which are equidistant from the center
are congruent.
For example, in the figure, if |OM| = |ON|,
then |AB| = |CD|.
The converse of this property is also true:
if |AB| = |CD|, then |OM| = |ON|.

N
M

Circles

167

EXAMPLE

Solution

In the figure, |AB| = 8 cm,


|CN| = 4 cm, and
|OM| = 3 cm. Find |OC| = x.
|CD| = 8 cm, since |CN| = 4 cm. So |AB| = |CD|,
and by the property, |OM| = |ON| = 3 cm.

x
M

3 O
D

Let us use the Pythagorean Theorem to find the length of [OC]:


|OC|2 = |ON|2 + |NC|2
x2 = 32 + 42
x = 5 cm.
Property

In the same circle or in congruent circles, if two chords have different lengths, then the
longer chord is nearer to the center of the circle.
For example, in the figure,
if |CD| > |AB|, then |OF| < |OE|. The converse
of this property is also true: if |OF| < |OE|,
then |CD| > |AB|.

EXAMPLE

Solution

168

In the circle in the figure,


|OM| < |ON| and r = 9 cm.
|AB| = 3x + 2 cm and
|CD| = 5x 2 cm are given.
Find the possible integer
values of x.

O
D

O
N
A

If |OM| < |ON|, then |CD| > |AB|.


5x 2 > 3x + 2
2x > 4
x > 2 (1)
Since the longest chord is the diameter, the greatest possible value of |CD| is the diameter.
d = 2r, d = 2 9 = 18 cm
|CD| 18
5x 2 18
5x 20
x 4 (2)
From (1) and (2), the possible integer values of x are 3 and 4.
Geometriy 7

Check Yourself 2
1. In the figure, the radius of the circle is 5 cm and
|AB| = |CD| = 8 cm. Find|OE|.

O
E
D

2. In the figure, |AB| = |CD|, [OM] [AB], [ON] [CD], and


|ON| = |OM| = 4 cm. Given |AB| = 5x + 1 cm and
|CD| = 4x + 2 cm, find the radius of the circle.

O
M
A

3. In the figure, |AP| = 12 cm, |PB| = 4 cm, and


|OP| = 11 cm. Find the radius of the circle.

D
A

12

O
P 4
B

4. In the figure, |AB| = 12 cm, |DC| = 2 cm,

D
A

[OD] [AB].

Find the radius of the circle.

Answers
1. 3 cm

2. 5 cm

3. 13 cm

4. 10 cm

C. TANGENTS

Remember that a tangent is a line in the plane which


intersects a circle at exactly one point. The point is
called the point of tangency. In this section we will look
at the properties of tangents.
Property

If a line is tangent to a circle, then the line is perpendicular to the radius drawn to the point
of tangency.
H

For example, in the figure, if l is tangent to the circle C


at point H, then [OH] l.

Circles

169

Property

If two segments from the same exterior point are tangent to a circle, then they are congruent.
A

For example, in the figure, if [PA and [PB are tangent to


the circle at points A and B respectively, then
|PA| = |PB|.

O
B

Property

Two tangent line segments from the same external point determine an angle that is bisected by
the ray from the external point through the center of the circle.
A

For example, in the figure, if [PA and [PB are tangent to


the circle then [PO is the angle bisector of APB, i.e.
mAPO = mBPO.

O
B

EXAMPLE

Solution

The circle in the figure is inscribed in the triangle ABC.


|AK| = x + 5 cm,
|BM| = 2x + 3 cm,
|CL| = 2x + 5 cm, and the perimeter of triangle ABC
is 46 cm.
Find |MC|.

A
L
K

O
M

|AK| = |AL|, |BK| = |BM|, and |CM| = |CL|.


P(ABC) = |AB| +|BC| +|AC|
= |AK| + |KB| + |BM| + |MC| + |CL| + |LA|
= 2 |AK| + 2 |BM| + 2 |CL|
= 2 (x + 5) + 2 (2x + 3) + 2 (2x + 5)
= 2x + 10 + 4x + 6 + 4x + 10
= 10x + 26
P(ABC) = 10x + 26 = 46 x = 2 cm
So |MC| = 2x + 5 = 9 cm.

170

Geometriy 7

D. RELATIVE POSITION OF TWO CIRCLES IN THE SAME


PLANE
Definition

nonintersecting circles
Two circles which have no common point are called nonintersecting circles.

If two or more circles


share the same center,
then they are called
concentric circles.

r1
O1

O1

O2

O2

O
r1

C1 C2=
r1 + r2< a

C1 C2=
a=0

C1 C2=
r1 r2> a

concentric circles

Definition

r2

r2

nonintersecting circles

tangent circles
Two circles which have only one common point are called tangent circles.
Tangent circles can be externally tangent or internally tangent, as shown in the figure.
l

O1

Circles

r1
a

r2

r1
O2

O1

a O2 r2

C C2 = {A} and r1 + r2 = a

C1 C2 = {A} and r1 r2 = a

externally tangent circles:


l is the common tangent

internally tangent circles:


l is the common tangent

171

intersecting circles

Definition

Two circles which have two common points are called intersecting circles.
A

r1
O1

r2

O1

O2

O2

a
B

C1 C2 = {A, B} and r1 + r2 > a


intersecting circles

EXAMPLE

Solution

[AB] is the common chord


[O1O2] ^ [AB] and |AH| = |HB|

The circles in the figure with centers A, B, and C are


externally tangent to each other.
|AB| = 16 cm, |BC| = 12 cm, and |AC| = 14 cm are
given. Find the radii of the circles.
Let the radii of circles A, B, and C be r1, r2 and r3
respectively. Then we can write,

C
B
A

|AB| = r1 + r2 = 16
|BC| = r2 + r3 = 12
+ |CA| = r1 + r3 = 14
2 (r1 + r2 + r3) = 42
r1 + r2 + r3 = 21

r1 + r2 + r3 = 21

r1 + 12 = 21
r1 = 9 cm.

172

r1 + r2 = 16
9 + r2 = 16
r2 = 7 cm.

r1 + r2 + r3 = 21

16 + r3 = 21
r3 =5 cm.

Geometriy 7

EXERCISES

4.1

1. Describe each line and

6. In the figure,

E
D

line segment in the


figure as an element of
the circle.

B
A

2. The points in the figure

are in the same plane as


the circle. State the
position of each point
with respect to the
circle.

|AC| = 6 cm and
|AB| = 3 cm.
Find |OB| = r.

8. In the figure,
D

|BC| = 12 cm and
|AD| = 8 cm.
Find the radius of the
circle.

of circle O is 15 cm,
|CD| = 24 cm, and
|OH| = 12 cm.

I
H

O
C

a. Find |OI|.

8
A

12
B

b. Find |AB|.

9. In the figure,

4. Complete each state-

ment about the figure


with a suitable symbol.

F
C

a. If |OE| = |OF|, then


|AB|...|CD|

O
E

|AP| = 63 cm and
mAPB = 60.
Find the radius of the
circle.

A
C

|O1O2| = 3 cm and
r1 + r2 = 11 cm.
Find r1 and r2.

5. In the figure, X, Y, and Z are

r2

r1

O1 O2

11. In the figure,


X
O

10. In the figure,

b. If |OE| > |OF|, then


|AB|...|CD|.

Circles

3. In the figure, the radius

points of tangency.
|AX| = 6 cm,
|CZ| = 4 cm, and
|BY| = 2 cm.
Find the perimeter of
ABC.

2
K

7. In the figure,

[OA] [BC],
|AK| = 2 cm, and
|KC| = 4 cm.
Find |OK|.

|AB| = 3x + 4,
|CD| = 2x + 9, and
|OM| > |ON|.
Find the greatest possible
integer value of x.

O
M
B

173

A. ARCS AND CENTRAL ANGLES


Definition

arc of a circle
An arc of a circle consists of two points on the circle and the unbroken part of the circle
between these two points.

Objectives
After studying this
section you will be
able to:
1. Describe the
concepts of arc and
central angle.
2. Name inscribed
angles and calculate their measure.
3. Use the properties
of arcs, central
angles, and
inscribed angles to
solve problems.

Definition

sign over two or more points to denote the

arc which includes the points. For example, in the fig-

We use the

ure, we write AB to denote the arc between A and B, and

minor arc

ACB to denote the arc ACB.

Notice that any two points of a circle divide the circle into
two arcs. If the arcs are unequal, the smaller arc is called

the minor arc and the larger arc is called the major arc.
O

In the figure on the right, AB is the minor arc and ACB


is the major arc.

central angle of a circle


An angle whose vertex is at the center of a circle is called a central angle of the circle.
We use the
sign over two or more points to denote

the arc which includes the points. For example, in the


figure, we write AB to denote the arc between A and B,
and ACB to denote the arc ACB.
Notice that any two points of a circle divide the circle
into two arcs. If the arcs are unequal, the smaller arc is
called the minor arc and the larger arc is called the
major arc.

In the figure on the right, AB is the minor arc and ACB is the major arc.
174

Geometriy 7

EXAMPLE

Solution

Find the measure of the


indicated central angle of
each circle.

a.

b.
50

Remember that the measure


of a minor arc is equal to the
measure of its central angle.

c.
120

180
D

C
O

a. mAOB = mAB = 50
b. mCOD = mCD = 120
c. mAOB = mAB = 180
Property

In the same circle or in congruent circles, if two chords


are congruent, then their corresponding arcs and central angles are also congruent.

D
r
A

For example, in the figure, if [AB] [CD] then AB CD


and mAOB mCOD.

r
C

r
B

Property

If a line through the center of a circle is perpendicular


to a chord, it bisects the arcs defined by the endpoints
of that chord.

For example, in the figure, if [PK] [AB] then


[AH] [HB]

r
A

AP PB

r
H

AK KB.

B. INSCRIBED ANGLES
Definition

inscribed angle of a circle


An angle whose vertex is on a circle and whose sides
contain chords of the circle is called an inscribed angle.
For example, angle ABC in the figure is an inscribed
angle. [AB] and [BC] are both chords of the circle.
The arc AC in the figure is called the intercepted arc of
the inscribed angle ABC.

Circles

intercepted
arc

O
A

175

Property

The measure of an inscribed angle is half of the measure


of the central angle which intercepts the same arc.

C
a O
b

Proof
In the figure, let mBCO = a and mBAO = b.
Since the triangles BOC and AOB are isosceles
triangles, we can write

a
2a
2b
b
A

mOBC = mOCB = a
mOAB = mOBA = b

mCOE = mCBO + mOCB = 2a


mAOE = mOAB + mOBA = 2b

mABC = mOBA + mOBC


= a + b

mAOC = mAOE + mEOC


= 2 (a + b)

So mABC =

mAOC
.
2

Now remember that the measure of a minor arc is the same as the measure of its central
angle. So we can write the property in a slightly different way:

Property

The measure of an inscribed angle is equal to the half


the measure of its intercepted arc.

mAC

For example, in the figure, mABC =


.
2

EXAMPLE

a O

2a

2a

10 Find the measure of x in each figure.


a.

b.

C
B

50

c.

C
B

B
O
120

O
A

176

Geometriy 7

Solution

a. mABC =

mAC

b. mABC =

50 =

mAC

mx =

mAC

2
120
2

mx = 60

mAC = 100

mAOC
2

c. mABC =

90
2

mx = 45

mx = 100

Property

If two inscribed angles intercept the same arc of a


circle, then the angles are congruent.

For example, in the figure, ABC ADC, because


they both intercept AC.

B
A
D

EXAMPLE

11 Find the value of x and y in each figure.


a.

b.
A

40

40
2

x = y = 20

mCD

20 O

C
B

a. mCAD = mCBD =
x = y =

Circles

Solution

c.

50

b. mBAC =
x =

mBOC
mBC
=
2
2

y 50
=
2
2

x = 25 and y = 50

177

c. mBAC =
20 =

mBOC
2

mBDC =

y
2

x =

mBOC
2

40
2

x = 20

y = 40

Property

An angle inscribed in a semicircle is a right angle.

For example, in the figure,


if mALB = mAMB = mANB = 180, then

mALB = mAMB = mANB = 90 or


mL = mM = mN = 90.

EXAMPLE

12 Find the value of x in each figure.


a.

b.
B
x
A

60

x
O

c.

20

10

Solution

a. Since AC is the diameter, the arc ABC is a semicircle.


So ABC is inscribed in a semicircle, and therefore mABC = x = 90, x = 90.

b. mDAC =

mDC

60
=
2
= 30
178

y = 2 mBAC
= 2 20
= 40

mAD + mDC + mCB = 180


x + 60 + 40 = 180
x = 80

Geometriy 7

c. Let us draw the chord [BD].

mADB = 90

mCAB = mCDB = 10
mADC + mCDB = 90

10

x + 10 = 90

x = 80

Property

The measure of the angle formed by a tangent and a chord is equal to the half of the measure of its intercepted arc.
1
For example, in the figure, mCAB = mAOB.
2

Proof

Let us draw the diameter [AD] and the chord [BD].


[AC] [AD] (a radius is perpendicular to a tangent at
the point of tangency)

O
B

[AB] [BD] (definition of a semicircle)


D

mDAB + mBAC = 90
mDAB + mADB = 90 (in triangle ABD)

mADB = mBAC
mADB =
mBAC =

mAB

(inscribed angle rule)


2

B
O

mAB

(inscribed angle rule)


2

Rule

Let [AB] and [CD] be two chords of a circle.


If [AB] [CD], then

mABC = mBCD (alternate interior angles).


So mAC = mBD.

Circles

179

EXAMPLE

13 Find the value of x and y in each figure.


a.

b.
A
x
O

c.
B

y
B
A

15
C

C
O
30

[AB] || [CD]

Solution

a. Let us draw the radius [OA].

Then [AC] [AO]. AOB is isosceles triangle.


mOAB
mOAB + mBAC
30 + x
x
y
b.

=
=
=
=
=

mCD = 120

15
A 30 C

mC D
mDCE = x =
2
120
=
= 60
2

mAC

mABC =

mAC

2
60
= 30
2

30
r

30 + mCD + 30 = 180

mABC =

B 30
D

mAC + mCD + mDB = 180

mAC = 2 30 = 60

mBD

mBD = 2 15 = 30

c. mADC =

r 30

30 and
90
90
60
2 60 = 120

[AB] [CD] and mBAD =

30

F
C

B
O
30
D

[AD] [BC] and [AF] [AD]. So [AF] [BC]. Therefore, mABC = mBAE and
mEAB = x = 30, x = 30.
180

Geometriy 7

Rule

The measure of an angle formed by two secants, a


secant and a tangent, or two tangents drawn from a
point in the exterior of a circle is equal to half of the
difference of the measures of the intercepted arcs.

B
A

b O
C

mBAC + mBOC = 180


mBAC + mBC = 180
a + b = 180

C
A

A
O

P
D

mP =

mCD mAB
2

angle formed by two secants:

angle formed by a secant


and a tangent:
mP =

angle formed by two


tangents:
mACB mAB
2
mP + mAB = 180

mP =

mCB mAB
2

Rule

The measure of an angle formed by two chords that


intersect in the interior of a circle is equal to half the
sum of the measures of the intercepted arcs.

For example, in the figure, mAED = mBEC =

mBC
+ mAD

mAEB = mCED =

mA B + mC D
2

=
Circles

B
x

a
b

and

x+ y
a+ b
and =
.
2
2
181

EXAMPLE

14 Find the value of x in each figure.


a.

b.
A

30

E
C

Solution

c.
B

70

100

70 =

15

60

a. mBED =

mA C + mBD

2
mA C +60
( mBD
= 2 mBAD )
2

mAC = 140 60 = 80
mADC = mA C = 80 = 40, x = 40
2
2
b. mCAE =

mC E mB D

100 30
=
(mB D = 2 mBCD)
2
2

mCAE = x =

70
= 35, x = 35
2

c. mQPR + mQR = 180


60 + mQR = 180
mQR = 120
mQR + mQTR = 360
mQTR = 240
mQSR = x =

182

mQTR

240
=
= 120, x = 120
2
2

Geometriy 7

EXERCISES

4.2

1. In the figure,
mAOC = 120.
Find mABC.

7. In the figure,

x
120

mBC

mAD =
and
2
mDPC = 75.

D
75
P
x

Find mBAC.

2. In the figure,

8. In the figure,

mBAC = 30 and
mBKC = 70.
Find mOCA.

70
O
K
30

[AE is tangent to the


circle at the point B,
and mEBC = 75.
Find mA.

mAOC = mABC = 3x.


Find the value of x.

4. In the figure,

mCDB = 10
and mABD = 50.
Find mP.

10. In the figure,


mAOC = 100 and
mOAB = 70.
Find mOCB.

20
O 100
x

70

100

11. In the figure,

C
x

mAPD = 30,
mDKA = 60,
mBAC = a, and
mDCA = b.
Find a and b.

40
O

A
B

a
K

60
b
D

30

K
O

50

C
x

mDPA = 50.
Find mBCA.

12. In the figure,

10

Circles

3x 3x O

5. In the figure,

6. In the figure,

120
B
D

[AB] is a diameter and


mOCB = 40.
Find mOAC.

9. In the figure,

3. In the figure,

75

B
C

mBOC = 100 and


mACO = 20.
Find mAOC.

mCBD = 120.
Find AOC.

A
O

50

183

Objectives

After studying this section you will be able to:


1. Describe the concepts of circumference and arc length.
2. Find the area of a circle, an annulus, a sector, and a segment.

A. CIRCUMFERENCE AND ARC LENGTH


1. Circumference of a Circle
Remember from chapter 4 that the distance around a polygon is called the perimeter of the
polygon.

Note
Pi (, pronounced like the English word pie) is a Greek
letter. It is the first letter of a Greek word that means
measure around.

d
r

r
C

Definition

circumference
The distance around a circle is called the circumference of the circle.
If you measure the circumference and diameter of a circle and divide the circumference by
the diameter, you always get the same constant. This constant is approximately equal to
3.14, and denoted by .

Property

For all circles, the ratio of the circumference to the diameter is always the same number. The
number is called (pronounced likepie).
So if the circumference of a circle with a diameter d is C, then we can write
C = 2 r

C
= or C = d or
d

This is the formula for the measure of the circumference of a circle.


184

Geometriy 7

1. Find five different circular objects. Use a piece of string to measure their
circumference (C), and use a ruler to measure their diameter (d). Write
the values in a table.
2. For each circular object calculate the ratio C and then calculate the
d
average of all the ratios.
3. How do the number and the formula C = d relate to this activity?

EXAMPLE

15 a.

Find the diameter of a circle with circumference 24 cm.

b. Find the circumference of a circle with radius 5 cm.


c. Find the circumference of a circle with diameter 9 cm.

Solution

a. Let the diameter of the circle be d, then the circumference of the circle is C = d:
24 = d
d = 24 cm.
b. C = 2 5 = 2 5 = 10cm
c. C = 2r = 2r = d = 9 cm

2. Arc Length
Remember that an arc is a part of a circle. The measure of an arc is equal to the measure of
its central angle.
Rule

In a circle, the ratio of the length of a given arc AB to


the circumference is equal to the ratio of the measure
of the arc to 360.

arc length of A B
mA B
=
,
circumference of the circle
360
arc length of A B

so
=
.
2 r
360

We can rewrite this as

arc length of AB = 2 r

360

In the above formula the measure of the arc is given in degrees. The length of the arc is given
in a linear unit such as centimeters.
Circles

185

EXAMPLE

16 Find the length of each arc.


a.

b.

6 cm

c.

O 10 cm

d.

12 cm
E

210

18 cm

60
F

Solution

a. The length of a semicircle is half


of the circumference.
arc length of AB

180
= 2r
= 2 6
360
360
1
= 2 6
= 6cm
2

b. The length of a 90 arc is a quarter of


the circumference.
arc length of CD

90
= 2r
= 2 10
360
360
1
= 2 10
= 5cm
4

c. arc length of EF

60
= 2r
= 2 12
360
360
1
= 2 12
= 4cm
6

d. arc length of GTH

210
= 2r
= 2 18
360
360
21
= 2 18
= 21cm
36

Check Yourself 3
1. Find the circumference of the circle with the given radius.
a. r = 3 cm
b. r = 5 cm
c. r = 7 cm
d. r = 10 cm
2. Find the radius of the circle with the given circumference.
a. 12 cm
b. 24 cm
c. 36 cm
d. cm
3. Find the length of the minor arc in each figure.
a.

b.

c.
C

3 cm

120
B

O
5 cm

d.

E
8 cm 60
O

G
9 cm

200
L

K
H

Answers
1. a. 6 cm b. 10 cm c. 14 cm d. 20 cm
2. a. 6 cm b. 12 cm c. 18 cm d.
3. a.

186

1
cm
2

3
10
8
cm b.
cm c.
cm d. 8 cm
2
3
3

Geometriy 7

B. AREA OF A CIRCLE, A SECTOR, AND A SEGMENT


1. Area of a Circle
Property

The area of a circle is times the square of the radius.


A =
r2

r
O

To understand why this property is true, let us divide a


circle into 16 equal parts, and rearrange them as
follows:

C = pr
2

As the number of equal parts increases, the area of the circle gets closer and closer to the
area of a parallelogram.
C
2r
= r 2 .
The area of a parallelogram is A = r = r
2
2
So the area of a circle with radius r is A = r2.
EXAMPLE

17 a.

Find the area of a circle with radius r = 6 cm.

b. Find the radius of a circle with area 16 cm2.


c. Find the area of a circle with circumference 10 cm.

Solution

a. Let the area of the


circle be A, then
A = r2
A = 62
A = 36cm2.

Circles

b. Let the radius of


the circle be r, then
A = r2
16 = r2,
r2 = 16
r = 4 cm.

c. The formula for the


circumference of a
circle is C = 2 r:
10 = 2 r
r = 5 cm.
So the area of the
circle is
A = r2
= 52
= 25cm2.
187

2. Area of an Annulus
annulus

Definition

An annulus is a region bounded by two concentric circles.


How can we find the area of an annulus? Look at the
diagram.

R
O

an annulus

area of the big circle area of the small circle = area of the annulus

R
O

r
O

A(annulus) = R2 r2
= (R2 r2)

EXAMPLE

18 Find the area of the annulus bounded by concentric


circles with radii 5 cm and 3 cm long.

Solution

The radius of the big circle is R = 5 cm.

3 cm
5 cm O

The radius of the small circle is r = 3 cm.


A = R2 r2
A = (R2 r2)
A = (52 32)
A = (25 9)
A = 16 cm2
188

Geometriy 7

3. Area of a Sector
Definition

sector of a circle
A sector of a circle is the region bounded by two radii of the circle and their intercepted arc.
For example, in the figure, the smaller region AOB is a sec-

tor of the circle. If the degree measure of arc AB is

a
p r2 .
360
We can also calculate the area of a sector in a different

mAB = a then the area of sector AOB =

a
B

way:
Rule

The area of a sector of a circle is half the product of the length of the arc and the length of
its radius.

In the figure,
A B

a
r
360

| A B |
a
=
r
2
360

r
a

l
B

| A B | r
a
=
r2
2
360
A=

| A B| r
or
2

A=

lr
2

(|AB| = l ).

In the above formula the measure of the arc is given in degrees. The length of the arc is given
in a linear unit such as centimeters.

EXAMPLE

19 Find the area of each shaded sector.


a.

b.
5 cm
O

8 cm

72

6p
A

O
B

Circles

c.

15
6 cm O

189

Solution

a. r = 5 cm and ma = 72.
a
72
1
r2 =
5 2 = 25 = 5 cm 2
360
360
5
b. r = 8 cm and l = 6 cm.
A (sector AOB ) =

A (sector POS ) =

l r 6 8
=
= 24 cm 2
2
2

c. mBOC = 2 mBAC, so
mBOC = 30 and r = 6 cm.
A (sector BOC ) =

mBOC
30
1
r2 =
62 =
36 = 3 cm 2 .
360
360
12

4. Area of a Segment
Definition

segment of a circle
A segment of a circle is a region bounded by a chord and its intercepted arc.
A

a
h
O

A
r

B
O

area of segment =

area of
sector

=
B

area of
triangle

A = A(sector AOB) A(AOB)


A=

EXAMPLE

bh
a
r2
360
2

20 Find the area of each shaded segment.


a.

b.

c.

120
45
12 cm

12

cm

cm

190

Geometriy 7

Solution

a. Since mAOB = 90,


A (sector AOB ) =

90
1
r 2 = 36 = 9 cm 2
360
4

r 2 36
=
=18 cm 2
2
2
A(segment) = 9 18 cm2.
A ( AOB ) =

b. |OH| = 6 cm and
|AB| = 123 cm.
A(sector AOB) =
=

120
12 2
360

60 63
30
12

1
144
3

= 48 cm2.
A(AOB) =

| AB|| OH | 12 3 6

36 3 cm 2
2
2

A(segment) = 48 363 cm .
c. A(sector AOC) =
A(AOC) =

r
36
=
= 9 cm 2
4
4

45
6 O

6 6 36

18 cm 2
2
2

A(segment) = 9 18 cm2.

Check Yourself 4
1. Find the area of a circle with the given radius.
a. r = 3 cm

b. r = 5 cm

c. r = 12 cm

d. r = 16 cm

2. Find the area of a circle with the given circumference.


a. 4 cm

b. 12 cm

c. 20 cm

d. cm

3. Find the circumference of a circle with area 36 cm2.


4. The ratio of the radii of two circles is 5 : 3. What is the ratio of their areas?
R
2

5. The area of the shaded region in the figure is 32 cm and


R = 9 cm. Find r.

Circles

191

6. Find the area of the shaded region in each circle.


A

r
60

120

r
A

B
r = 8 cm

r = 5 cm

r = 5 cm
|AB| = 52 cm

r = 12 cm
A

O
R

r
A

r
O

r
120

O r

r = 5 cm

r = 8 cm

R = 10 cm
r = 8 cm

R = 10 cm
r = 7 cm

Answers
1. a. 9 cm2 b. 25 cm2 c. 144 cm2 d. 256 cm2
2. a. 4 cm2 b. 36 cm2 c. 100 cm2 d.

cm2
4

3. 12 cm
4.

25
9

5. 7 cm
6. a.

192

25
64
25 50
cm2 b.
cm2 c. 36 cm2 d.
cm2 e. 36 cm2 f. 25 cm2 g. 17 cm2 f. 16 cm2
6
3
4

Geometriy 7

EXERCISES

4.3

1. In the figure,
mAOB = 30 and
r = 6 cm.
Find the area of the
shaded region.

30

2. In the figure,

mBAC = 30 and
the radius of the
circle is 6 cm. Find the area
of the shaded region.

mAOB = 45 and
r = 10 cm.
Find the area of the
shaded region.

8. In the figure, ABCD is a

rectangle and
A and B are the
centers of two circles.
Given |AD| = 6 cm,
find the area of the
shaded region.

45
B
O r = 10

3. In the figure,

mAOB = 120 and


r = 6 cm.
Find the length of arc
AXB.

4. In the figure, B is the

7. In the figure,

120
O 6 cm

10.
B

5. In the figure, mAOB = mCOD = mEOF = 20


E

20

C
20

20
A

6. In the figure,

Circles

are the centers of three


congruent tangent circles.
If the sum of the
circumferences of the
circles is 24 cm, find
the area of the shaded
region.

|OB| = 5 cm,
mDOB = 60, and
|BA| = 3 cm.
Find the area of the
shaded region.

30

9. In the figure, A, B, and C

center of a circle and


ABCD is a square with
|AD| = 5 cm.
Find the area of the
shaded region.

and r = 6 cm. Find the


sum of the areas of the
shaded regions.

C
K

B, C, P, and K are the centers of four circles in the


figure. Given |AB| = |BC| = |CD| = 4 cm,
find the area of shaded region.

11. In the figure, ABCD is a

square with perimeter


64 cm. Find the area of
the shaded region.

C
D

60
O

193

12. In the figure, B and D


are the centers of two
circles.
If ABCD is a square
and the shaded area
is 16 cm2, find
|DE|.

13. In the figure,


A(AOB) = 48 cm2,
|OC| = 8 cm, and
[OC] [AB]. Find r.

18. In the figure,

D
P

mOAB = 50 and
mBCO = 35.
Find mAOC.

50
O

35
C

19. In the figure,

8 cm
r

mBCD = 130 and


mOAC = 40.
Find mCBO.

C 130

40
B

14. In the figure,


|AB| = |CD| = 8 cm and
|OH| = 3 cm.
Find the radius of
the circle.

45

x 60

mAOC = 160 and


mABC = x.
Find mABC.

160

B
C
A

mAOB = 60 and
|AB| = 5 cm.
Find the radius of the
circle.

B
C

C
x

40

B is the point of
tangency and
mOAB = 30.
Find mABC.

50

23. In the figure,

22. In the figure,


mOAD = 40 and
mBOC = 50.
Find mCOD.

17. In the figure,

194

B
A

21. In the figure,

16. In the figure,

|AB| = 9 cm,
|BC| = 8 cm, and
|CA| = 5 cm.
Find the radius of
circle A.

mOAB = 45 and
mOCB = 60.
Find mAOC.

20. In the figure,

B
r

15. In the figure,


|CE| = 3x 2,
|FB| = x + 4, and
|OE| = |OF|.
Find x.

8 cm

C
x

30

Geometriy 7

24. A and C are points of tangency on the circle in the


figure.
Given mABC = 60 and
mBCD = 70,
find mBAE.

30. In the figure, the radius

C
D

70

A
x

60

mBAD = 60 and
|AD| = |DC|.
Find mBCD.

60

of the circle is 6 cm and


the length of arc AXB is
4 cm. Find the area of
the shaded region.

r=

6c

31 . In the figure,

25. In the figure,

|OB| = r = 4 cm.
Find the area of the
shaded region.
B

32 . In the figure,

D
A

60

8c

m
6c

mAOE = 60 and
|OA| = |DC|.
Find mACE.

|AB| = 8 cm and
|AC| = 6 cm.
Find the area of the
shaded region.

26. In the figure,

33 . ABCD is a square with sides 10 cm long. Find the


area of each shaded region.
a.

27. In the figure,


mBAD = 30.
Find mACD.

D
E
30

c.

d.

O
20
A

C
B
E

figure,
F
C
40
30
|OA| = 6 cm,
O
mAOB = 50,
50
mCOD = 30,
and mEOF = 40.
A
B
Find the sum of the areas of the shaded regions.
Circles

b.

28. In the figure,

29. In the circle in the

30
C

mBAC = 20 and
mDFE = 30.
Find mCOD.

e.

d.

195

CHAPTER

REVIEW TEST

1. In the figure,
|OA| = 4 cm and
|OC| = 7 cm.
What is |BC|?
A) 2 cm

6. In the figure,

B) 3 cm

C) 4 cm

2. Find |AB|in the figure if

D) 5 cm

D) 5 cm

C) 16 cm

196

B) 12 cm

D) 20 cm

A
C 1 cm
m

B) 50

C) 51

mABD = 60 and
mCED = 80.
Find mCDE.

D) 52

80 x

60

A) 10

B) 20

C) 25

D) 40

mBDC = 70.
Find mACB.

9. In the figure,

32 cm

7c

A) 49

A
70

C) 42 cm

5. In the figure,

A) 13 cm

98
O

8. In the figure,
H

|OC| = 32 cm,
|AC| = 1 cm, and
|BC| = 7 cm.
What is the length
of the radius?

|OK| = |OH| = 5 cm,


|AB| = 2a + 2 cm, and
|CD| = a + 13 cm.
What is the length of
the radius?

4. In the figure,

B) 33 cm

D) 6 cm

C) 6 cm

of the circle is 10 cm
and |OH| = 6 cm.
Find |AB|.

A) 3 cm

C) 53 cm

3. In the figure, the radius

B) 12 cm

A) 8 cm

B) 5 cm

5 A

H
O

B) 7 cm

A) 4 cm

7. Find mABC in the figure.

|CH| = 4 cm.

A) 8 cm

line l is tangent to the


circle at point C and
|OA| = |AB| = 5 cm.
Find |BC| = x.

C
B

l
C

D) 5 cm

K
C

O
H

B) 25

C) 30

D) 40

10. In the figure, line l is

A) 20

tangent to the circle at


point A and
|AB| = |AC|.
Find mCAD.

l
A
B

80

C) 11 cm

D) 10 cm

A) 65

B) 55

C) 50

D) 45
Angles

11. In the figure,


[PE and [PD are
tangent
to
the
circle at the points
A and B, respectively.
Find mACB if
mAPB = 50.
A) 60

50

C) 70

D) 75

A
P

B) 20

35

D) 40

C) 75

14. In the figure,

A) 3 cm

B) 4 cm

A) 5 cm
Chapter Review Test 1A

B) 6 cm

3
cm
2

C) 2 cm

square with sides 6 cm


long. Find the area of
the shaded region.

D)

5
cm
2

D) 80

2 cm
B

C) 6 cm

D) 8 cm

3 cm
H

C) 7 cm

has radius 6 cm and


mABC = 75.
Find the area of the
shaded region.
A) 6 cm

B) 9 cm

D) 8 cm

6 cm

O
75
B

C) 12 cm

20 . In the figure, ABCD is a


square. |BE| = 4 cm,
|DF| = 6 cm, and B and
D are the centers of two
circles. Find the area of
the shaded region.

O
A

C) 36 9 cm2

19. In the figure, the circle


23 cm

9
B) 16
cm2
4
D) 49 12 cm2

A) 9 2 cm

15. In the figure,


the perimeter of the
circle is 10 cm and
|OH| = 3 cm.
Find |AB|.

B)

|AB|= 2 cm and
|AC| = 23 cm.
What is the length of
the circumference?

A) cm

B) 70

60

3 cm

O
30

A) 65

18. In the figure, ABCD is a

mDCE = 30 and
mAB = 80.
Find the value of x.

17. Find the length of the

D) 8 cm2

C
D

C) 30

4 cm

C) 4 cm2

O
x

13. In the figure,

B) 3 cm2

55

35

arc AB in the figure if


the radius is 3 cm and
mACB = 60.

mAPC = 35 and
mBD = 100.
Find mADC.

mABC = 35,
mACB = 55, and
|BC| = 4 cm.
What is the area of the
circle?
A) 2 cm2

12. In the figure,

A) 15

O
B

B) 65

16. In the figure,

D) 15 cm2
6 cm F C

6 cm

4 cm

H
A

E 4 cm B

A) 40 10 cm2

B) 50 13 cm2

C) 36 12 cm2

D) 64 20 cm2
197

EXERCISES

1. 1

1. Because there are no simpler concepts for us to buid on. Therefore, we need to understand these concepts
without a precise definition.
3. A ray has closed enpoint but a half line has an open endpoint.
4. 2 5. 3 7. a. size, length, width, thickness b. line c. plane d. skew lines
8. a. true b. true c. true d. false e. true
9. a. 10 b. 21 c. 210 d. 5050
10. lines: HL, HG rays: [LC, [LH, [HL, [HG, [GH half lines: ]LC, ]LH, ]HL, ]HG, ]GH
11. a. line segment CD b. half open line segment PQ c. open line segment AB d. ray KL e. half line MN
f. line EF
12. a. l (E) = l

b. d (F) = {C}

c. n (G) = 13. M, N and P, R and S, and L and K, are coplanar

points.
14. l

15.

d
l

(D) (E) = m
(D) (F) = l
(E) (F) = d
m d l = {O}

E
D

16. a. 5 b. (P) (Q) = EB, (P) (S) = EA, (P) (T) = AB, (Q) (T) = BC, (Q) (R) = EC, (T) (R) = DC,
(S) (R) = ED, (T) (S) = AD

EXERCISES

c. 3 lines pass through point A, B, C, and D, 4 lines pass through point E.

2. 1

1. a. b. c. {K, O, M} d. e. {N} f. g. h. {P} i. j. k.


2. a. {A} [CD b. ]AC[ ]AD[ c. ]CD[ d. ]CE ]DF e. ]AB] [BC[ ]AH ]DG
5. a. acute angle b. right angle c. obtuse angle d. straight angle e. complete angle
6. a. 20 b. 12 c. 20 7. a. 32 b. 20 c. 10
8. a. 115 b. 65 c. 115 d. 65 e. 115 f. 65 g. 65
9. 130

10. 40 11. 25

20. 90

21. 35

196

12. 50

13. 100

14. 70

15. x = y + z

16. 160

17. 35

18. 140

19. 80

Geometriy 7

3. 1

EXERCISES

1. ADE, DEK, DKF, BDF, CKF, CKE, DEC, ADC, DFC, BDC, CEF, ABC
2.

eight triangles: GDT, DTE, ETF, FTG, GDE, GFE, GDF, DEF 3. 51 cm 4. 10 cm

5. 28.2 cm

B
D

10. a.

r1

C
E

b.

r2

c.

r1

Va
r2

ha
B

d. segment FC without

8. 12 9. 7 cm

c. segment AC and point E

endpoints

7.

6. a. B, E, F, C b. F

11. a. Hint: Construct medians for each

A
na
r2

r2

side. b. Hint: Construct angle bisectors for


C each

angle. c. Hint: Construct altitudes

for each vertex.

d. Hint: Construct perpendicular bisectors for each side.

14. hb =

56
168
; hc =
5
13

15. a. BFC b. CEF, BEF,

ABC c. BFC d. ABF e. ABF 17. a. yes b. no c. yes d. yes e. no 18. a. x {4, 9, 14} b. none 19.

121
cm 2
2

20. a. in the interior b. in the interior c. in the interior d. in the interior e. on the triangle f. in the interior
g. in the interior h. on the triangle i. in the exterior j. in the interior k. in the interior l. in the exterior
21. a. sometimes b. always c. never d. sometimes e. never f. never g. always h. always
22. a.

b.

equilateral

23. a.

Answers to Exercises

c.

scalene right

d.

isosceles

e.

obtuse triangle

f.

equilateral

isosceles

b.

197

EXERCISES

3. 2

d. 6 e. 3 3. A K; D L; E N; AD KL;
11
4. a. 6 b. 20 c. 22 d.
5. a. BC = 3, MN = 8 9. 10 cm 11. 2 12. m(OKM) =
8
5
17. 84 18. 38 19. 9.6 cm 20. 20 21.
m(OLK) = 20 13. 8 14. 15 15. 84 16.
2
24. 33 25. 4 cm 26. 14 27. 15 28. 83 29. 3 cm 30. 16 31. 2 cm 32. 6 cm
2. a. 70

b. 1

c. 60

DE LN; AE KN
10, m(OML) = 60,
1
22. 33 23. 8 cm
4
33. 2 cm 34. 2 cm

35. 43 cm 36. 70 37. 12 38. 99 39. 9 cm 40. 33 3 41. 24 cm 42. yes 43. 200 km 44. 25 45. 7 5
46. 6 47. 70 48. 57

49. 150 50. 8 cm 51. 16 cm 52. 18 53. 8 and 12 54. 8 55. 45 58. 4 cm 59. 10

60. 32 60. 72 62. 6 cm 63. 12 cm 64. 12 cm 65. 8 66. 6 cm 67. 6 cm 68. 6 cm 69. 8 cm 70. 9 cm
3
71.
72. 6 73. 2 74. 18 75. 15
2

EXERCISES

3. 3

1. a. 36 b. 114 c. 54 d. 20 e. 100 f. 50 g. 90 h. 64 2. 40, 60, 80


4. 50 5. 136 6. 60
7. 72 8. 117 9. 12 10. 8 11. 36 12. 106
13. a.
b.
c. not possible d.
E
E
20

150

70
D

40
F

60

E
80

60

80

3. x = 120, acute angles: 85, 5

20

60

e. not possible
40

80

40
M

14. a. no b. no c. no d. yes 15. 55 17. a. 2 < a < 14 b. 4 < p < 20 c. 1 < m < 7 18. a. 4 < x < 12
b. 4 < x < 11 c. 3 < x < 10 19. three 20. 9 21. 12 22. 29 23. 36 24. 9 25. five 26. a. yes b. no c. no
d. no e. yes f. yes 27. x 28. a. 1 = 2 > 3 b. 1 > 2 > 3 c. 3 > 2 > 1 d. 1 > 3 > 2 29. a. false b. false
c. true d. false e. true f. false g. false 30. a. AC b. AC c. DC d. BC 31. a. 5 b. 8 32. 49 33. 25
34. there are no values 35. 11 36. four triangles with side lengths (1, 5, 5), (2, 4, 5), (3, 4, 4), (3, 3, 5) 38. a. A
b. D c. D d. D 39. a. A b. B 40. a. A b. B c. A d. A 41. a. A b. B 42. a. D b. A

EXERCISES
1.

13
10

2. {21, 31}

3. 4
3.

3
4

4. {49, 31}

9. 3x + 4y 10 = 0 ; 3x + 4y + 30 = 0 10.

198

5.

16
29

6. {

35 25
,
}
9
9

7. a. 13 b. 15 2
4

c.

5
2

8.

12
43

53
15
Geometriy 7

EXERCISES

4. 1

1. radii: [OF], [OC], [OA], [OB] diameter: [FC] chords: [ED], [FC], [GB]

tangent: AH secant: GB center: O

2. Points C, O, and D are in the interior region of the circle. Point E is on the circle. Points A, B, G and F are in the
exterior region of the circle 3. a. 9 cm b. 18 cm
9. 6 cm 10. r1 = 4 cm, r2 = 7 cm

EXERCISES
1. 120

2. 10

4. a. = b. >

5. 24 cm

6. 3 cm

11. 4 cm

7.

9
cm
2

8. 46 cm

4. 2

3. 120

4. 30

5. 50

6. 40

7. 70

8. 60

9. 40

10. 60 11. a = 15 b = 45

12. 20

EXERCISES
1. 33 cm2 2.

25
cm2
2

8. (72 18) cm2


13. 10 cm
24. 120

4. 3

14. 5 cm
25. 20

3. 4 cm 4. (25 25 ) cm2
4

9. (163 8) cm2
15. 3
26. 120

16.

5 3
cm
3
27. 80

10. 12 cm2
17. 3 cm
28. 12 cm2

5. 6 cm2 6.

cm2 7. (6 93) cm2

11. (256 64) cm2

18. 170

19. 10

29. 12 cm2

b. (50 100) cm2 c. (100 25) cm2 d. (50 100) cm2 e. (

Answers to Exercises

13
2

20. 150
30. 4 cm2

12. (82 8) cm
21. 100
31.

21. 30

25 - 48
cm2
2

22. 60

23. 50

32. a. 50 cm2

25
25
+ 25) cm2 f. (75
) cm2
4
2

199

TEST
1.
2.
3.
4.
5.
6.
7.
8.

D
A
B
C
C
D
C
B

TEST
1.
2.
3.
4.
5.
6.
7.
8.

200

D
D
A
C
E
B
C
D

TEST
9.
10.
11.
12.

D
D
C
D

2A

E
D
A
C
A
E
B
C

TEST
1.
2.
3.
4.
5.
6.
7.
8.

1.
2.
3.
4.
5.
6.
7.
8.

C
A
D
C
B
B
B
B

TEST
9.
10.
11.
12.
13.
14.
15.
16.

C
C
B
C
D
C
E
C

2D

1.
2.
3.
4.
5.
6.
7.
8.

A
E
E
C
A
A
C
D

1.
2.
3.
4.
5.
6.
7.
8.

9.
10.
11.

D
E
B
C
C
A
C
C

C
C
D

2B

C
A
E
C
A
C
B
A

TEST
9.
10.
11.
12.
13.
14.
15.
16.

TEST
9.
10.
11.
12.
13.
14.
15.
16.

B
C
C
A
C
C
A
E

9.
10.
11.
12.
13.
14.
15.
16.

A
C
D
C
C
C
B
B

1.
2.
3.
4.
5.
6.
7.
8.

E
D
C
B
E
E
D
C

2C
9.
10.
11.
12.
13.
14.
15.
16.

B
A
D
C
D
A
C
C

2E

Geometriy 7

Symbol

Meaning

Symbol

is equal to

is congruent to

is not equal to

is not congruent to

is greater than

is parallel to

is greater than or equal to

is not parallel to

is less than

is perpendicular to

is less than or equal to

is similar to

is approximately equal to

|x|

absolute value of x

pi

square root

angle A

exterior angle of A in a triangle

mA

measure of angle A in degrees

ABC

Meaning

triangle with vertices A, B and C

ha

length of the altitude to side a

is an element of

is not an element of

union

intersection

is contained by

degrees

A B

A is contained by B

minutes

A B

A is not contained by B

seconds

A.S.A

angle-side-angle

right angle

S.A.S

side-angle-side

mABC

measure of angle ABC in degrees

S.S.S

side-side-side

AB

minor arc with endpoints A and B

A.A

angle-angle

mAB

measure of minor arc AB in degrees

(E)

plane E

ACB

major arc with endpoints A and B

mACB

measure of major arc ACB in degrees

AB

line AB, passing through the points A


and B

[AB]

line segment AB or segment AB, with


endpoints A and B

|AB|

length of segment AB

(int ABC)

interior of the triangle ABC

(ext ABC)

exterior of the triangle ABC

A(ABC)

area of the triangle ABC

P(ABC)

perimeter of the triangle ABC

ABCD

quadrilateral ABCD

ABCD

paralelogram ABCD

circle with center O

circumference

[AB

ray AB with initial point A, passing


through B

sin

sine

]AB

half line AB

cos

cosine

]AB]

half-open line segment AB, excluding


point A and including point B

tan

tangent

cot

cotangent

]AB[

open line segment

sec

secant

[AB]

closed line segment

cosec

cosecant

acute angle: An acute angle is


an angle with measure greater
than 0 and less than 90.

angle bisector of a triangle: An angle


bisector of a triangle is a segment that
bisects one of the angles of the
triangle. Its endpoints are points on the
triangle.

85

20

acute triangle: An acute triangle has

by the horizontal and the line of sight to


an object below the horizontal.

50 70

horizon
(horizontal)

angle of depression: The angle formed

60

three acute angles.

angle of
depression

adjacent angles: Two angles


are adjacent if they share a common vertex and side, but have
no common interior points.

adjacent sides: In a triangle or

a b

angle of elevation : The angle formed

b and c are adjacent angles


a and c are not adjacent angles

other polygon, two sides that share a common vertex are


adjacent sides.

alternate exterior angles: Two angles

are alternate exterior angles if they lie


outside l and m on opposite sides of t,
such as b and g.

a b
c d

base: The lower face or side of a geometric shape.

altitude of a triangle: An altitude of a


triangle is a segment from a vertex that
is perpendicular to the opposite side or
to the line containing the opposite side.
An altitude may lie inside or outside the triangle.

ray that divides the angle into two congruent angles.

altitude

O is the center of
the circle

circle: A circle is the set of all points in a plane that are


B

vertex: point A
sides: [AC and [AB

C
B

O
A

[OB is an angle bisector


[AN] is an angle bisector

202

center of a circle : The center of a circle is


the point inside the circle that is
equidistant from all the points on the
circle.

central angle of a circle: A central angle of a circle is an


angle whose vertex is the center of the circle.
C

angle bisector: An angle bisector is a

area: The number of square units that cover a given surface.

alternate interior angles : Two angles are alternate


interior angles if they lie between l and m on opposite sides
of t, such as d and e. (See figure for alternate exterior
angles.)

rays that have the same initial point. The


rays are the sides of the angle and the initial point is the vertex of the angle.

angle of
elevation
horizontal

e f
g h

angle: An angle consists of two different

by the horizontal and the line of sight to


an object above the horizontal.

equidistant from a given point, called the center of the


circle.

circumference of a circle: The circumference of a circle is


the distance around the circle.
collinear : Points, segments,
or rays that are on the same
line are collinear.

A, B, C, and D are collinear

complementary angles: Two angles are complementary if


the sum of their measures is 90. Each angle is a
complement of the other.
concave polygon: See non-convex polygon.

Geometriy 7

concurrent : Two or more lines or segments are


concurrent if they intersect at a single point.
the same measure.

decagon: A decagon is a polygon that has


ten sides.

congruent arcs: On the same circle or on congruent circles,

degree: A unit of angle and arc measure.

two arcs are congruent if they have the same measure.

diameter of a circle: A diameter of a circle is a chord that

congruent polygons : Two polygons are congruent if there is

passes through the center. The diameter, d, is twice the


radius: d = 2r.

congruent angles: Two angles are congruent if they have

a correspondence between their angles and sides such that


corresponding angles are congruent and corresponding sides
are congruent. Congruent polygons have the same size and
the same shape.

diagonal : A line segment joining two

al

gon

dia

non-adjacent vertices of a polygon.

congruent segments: Two segments are congruent if they


have the same length.

consecutive interior angles : Two


angles are consecutive interior angles if
they lie between l and m on the same side
of t, such as b and e.

t
d a
c b

l
m

h e
g f

convex polygons

coplanar: Points, lines, segments or rays that lie in the same


plane.

corresponding angles: Two angles are


corresponding angles if they occupy corresponding positions, such as a and e
in the figure.

t
d a
c b
h e
g f

equilateral triangle: An equilateral triangle has three congruent sides.

convex polygon: A polygon is convex if


no line that contains a side of the
polygon contains a point in the interior of
the polygon.

equiangular triangle: An equiangular triangle has three


congruent angles, each with a measure of 60.

l
m

exterior angles of a triangle :


When the sides of a triangle are
extended, the angles that are
adjacent to the interior angles of the
triangle are the exterior angles. Each
vertex has a pair of exterior angles.

exterior
angle

exterior
angle
interior
angle

exterior of an angle: A point D is in the


exterior of A if it is not on the angle or in
the interior of the angle.

concentric circles : Circles that have


different radii but share the same center
are called concentric circles.

cone: A solid figure that has a circular


base and a point at the top.

half line: A ray without an endpoint


(initial point).

]AB = half line AB

half planes: Two halves of a plane that are

separated by a line

P1

d
P2

P = P 1 P2 P

cube: A square prism that has six equal


square sides.

cylinder: A solid with circular ends and


straight sides.
Answers to Exercises

hexagon: A hexagon is a polygon with six


sides.

hypotenuse: In a right triangle, the side


opposite the right angle is the hypotenuse
of the triangle.

hy

po

ten

us

203

inscribed angle of a circle: An angle is


an inscribed angle of a circle if its vertex is

O
B

on the circle and its sides are chords of the


circle.

AOB is an
inscribed angle
C

interior of an angle: A point D is in the


on each side of the angle.

then the points A and B together with the


points of the circle that lie in the exterior
of mAPB form a major arc of the circle.
Major arcs are denoted by three letters, as
in ACB.

midpoint of a segment: The midpoint

interior of A if it is between points that lie

major arc: On circle P, if mAPB < 180,

of a segment is the point that divides the


segment into two congruent segments.

intersecting lines : Coplanar lines which have only one

minor arc: On circle P, if mAPB < 180,

point in common.

then the points A and B, together with the

intersecting planes: Planes which have one common line.

points of the circle that lie in the interior of

isosceles triangle : An isosceles triangle has at least two

mAPB form a minor arc of the circle.

congruent sides.

isosceles trapezoid : A quadrilateral with one pair of

Minor arcs are denoted by two letters, such


as AB.

ACB is an major
arc of the circle

|RS| = |ST|
R
S
T
midpoint
C
P

AB is a minor
arc of the circle

parallel sides and at least two sides the same length.

noncollinear : Points, segments, or rays that are not


collinear.

kite: A convex quadrilateral with two pairs of

non-cconvex polygon : A polygon is

equal adjacent sides.

legs of a right triangle: Either of the two


sides that form a right angle of a right tri-

non-convex (concave) if at least one


line that contains a side of the polygon
contains a point in the interior of the
polygon.

non-ccoplanar : Not coplanar.

leg
leg

angle.

legs of an isosceles triangle: One of the


two congruent sides in an isosceles

leg

leg

triangle.

oblique lines: Lines are oblique if they


intersect and do not form right angles.

line: A line is an undefined term in geometry. In Euclidean

obtuse angle: An obtuse angle is an angle with measure


greater than 90 and less than 180.

geometry a line is understood to be straight, to contain an

obtuse triangle: An obtuse triangle has exactly one obtuse

infinite number of points, to extend infinitely in two direc-

angle.

tions, and to have no thickness.

octagon: An octagon is a polygon with


eight sides.

line segment : See segment.


204

Geometriy 7

postulate : A postulate is a statement that is accepted as true


without proof.
proof: A proof is an organized series of statements that show
that the statement to be proved follows logically from known
facts (given statements, postulates, and previously proven
theorems).

parallel lines: Two lines are parallel if


they are coplanar and do not intersect.

parallel planes: Two planes are parallel


if they do not intersect.

protractor: A device used to determine the

E
A and B are
parallel planes

measures of angles.

pythagorean triple: A set of three positive


integers a, b, and c that satisfy the equation
a2 + b2 = c2 is a pythagorean triple.

opposite sides parallel, and hence equal in


length.

180

pentagon: A pentagon is a polygon with

perimeter of a polygon: The perimeter of a polygon is the


sum of the length of its sides.

perpendicular lines : Two lines are


perpendicular if they intersect to form a

180

10

a
a2 + b2 = c2

pyramid: A solid figure with a


polygon base and whose other
faces are triangles that share a
common vertex.

quadrilateral: A polygon with four sides. The sum of the

right angle.

k^l

perpendicular line and plane: A line

is perpendicular to a plane if it is

angles is 360.
100
80

135

120
60

45

45
135

P
l^P

plane: A plane is an undefined term in geometry. In


Euclidean geometry it can be thought of as a flat surface that
extends infinitely in all directions.

point: A point is an undefined term in geometry. It can be


thought of as a dot that represents a location in a plane or in
space.

polygon: A polygon is a plane figure formed by three or more


segments called sides, such that the following are true:
1. each side intersects exactly two other sides, once at each

radius of circle: A radius of a circle is a segment that has


the center as one endpoint and a point on the circle as the
other endpoint.
ray: The ray AB, or [AB, consists of the
initial point A and all points on line that
lie on the same side of A as B lies.

[AB
A

[BA
B

rectangle : A rectangle is a parallelogram that has four right


angles.

rectangular prism : A solid figure


that with two bases that are rectangles
and with all other faces that are
parallelograms.

endpoint, and
2. no two sides with a common endpoint are collinear.
Answers to Exercises

prism: A solid figure that has two


bases that are parallel, congruent
polygons and with all other faces
that are parallelograms.

five sides.

perpendicular to each line in the plane.

10
0

parallelogram : A quadrilateral with

80 90 100
70
110
60
120
50
130
90 80 70
110100
140
120
60
50
130
150
40
140
30
150
160
160
20
170
170
10
40

30
20

205

regular polygon : A polygon whose


sides are equal and whose angles are
equal.

square: A square is a parallelogram that


is both a rhombus and a rectangle; that
is, it has four congruent sides and four
right angles.

right prism: A prism that has two

straight angle: A straight angle is an


angle that measures 180.

special characteristics: all lateral


edges are perpendicular to the bases
and all lateral faces are rectangular.

rhombus: A rhombus is a parallelogram that has four


congruent sides.
right triangle: A triangle with exactly one right angle.

scale factor : In two similar polygons or two similar solids,


the scale factor is the ratio of corresponding linear
measures.
scalene triangle: A scalene triangle is a triangle that has no
congruent sides.

segment: A segment AB, or [AB],


consists of the endpoints A and B and
all points on the line AB that lie
between A and B.

AB or BA
A
B
[AB] or [BA]
A

similar polygons : Two polygons are similar if their


corresponding angles are congruent and the lengths of
their corresponding sides are proportional.

sine: The ratio of the length of the side

C
sin a =

opposite an angle to the length of the


hypotenuse in a right triangle.
A

surface area: The sum of all the areas of the surfaces of a


solid figure.

tangent: The ratio of the length of the


side opposite an angle to the length of the
side adjacent to the angle in a right triangle.

C
tan a =

|AC|
|AB|

a
A

tangent to a circle: A line is tangent to


a circle if it intersects the circle at exactly one point.

A
C
B

theorem: A theorem is a statement that must be proved to


be true.

transversal: A transversal is a line that

l
d

intersects two or more coplanar lines at


different points.
k
l is the tranversal

trapezoid : A quadrilateral with exactly


one pair of opposite parallel sides. The
sum of the angles is 360.

skew lines: Two lines are skew if they


do not lie in the same plane.

k
d and k
are skew lines

space: The set of all points.


sphere: A sphere is the set of all points
in space that are a given distance r from
a point called the center. The distance r
is the radius of the sphere.

supplementary angles: Two angles are supplementary if


the sum of their measures is 180. Each angle is a
supplement of the other.

|AC|
|BC|

straight angle

vertex of a polygon: A vertex of a polygon is a common


endpoint of two of its sides.
vertical angles : Two
angles are vertical if their
sides form two pairs of
opposite rays.

b
c

a and c are vertical angles


b and d are vertical angles

volume: The number of cubic units needed to occupy a


given space.

Potrebbero piacerti anche